Эдс Ρ‚ΠΎΠΊΠ°: Π­Π”Π‘. Π—Π°ΠΊΠΎΠ½ Ома для ΠΏΠΎΠ»Π½ΠΎΠΉ Ρ†Π΅ΠΏΠΈ.

Π‘ΠΎΠ΄Π΅Ρ€ΠΆΠ°Π½ΠΈΠ΅

Π£Ρ€ΠΎΠΊ 31. Π·Π°ΠΊΠΎΠ½ ΠΎΠΌΠ° для ΠΏΠΎΠ»Π½ΠΎΠΉ Ρ†Π΅ΠΏΠΈ – Π€ΠΈΠ·ΠΈΠΊΠ° – 10 класс

Ѐизика, 10 класс

Π£Ρ€ΠΎΠΊ 31. Π—Π°ΠΊΠΎΠ½ Ома для ΠΏΠΎΠ»Π½ΠΎΠΉ Ρ†Π΅ΠΏΠΈ

ΠŸΠ΅Ρ€Π΅Ρ‡Π΅Π½ΡŒ вопросов, рассматриваСмых Π½Π° ΡƒΡ€ΠΎΠΊΠ΅:

1) Π·Π°ΠΊΠΎΠ½ Ома для ΠΏΠΎΠ»Π½ΠΎΠΉ Ρ†Π΅ΠΏΠΈ;

2) связь Π­Π”Π‘ с Π²Π½ΡƒΡ‚Ρ€Π΅Π½Π½ΠΈΠΌ сопротивлСниСм;

3) ΠΊΠΎΡ€ΠΎΡ‚ΠΊΠΎΠ΅ Π·Π°ΠΌΡ‹ΠΊΠ°Π½ΠΈΠ΅;

4) Ρ€Π°Π·Π»ΠΈΡ‡ΠΈΠ΅ ΠΌΠ΅ΠΆΠ΄Ρƒ Π­Π”Π‘, напряТСниСм ΠΈ Ρ€Π°Π·Π½ΠΎΡΡ‚ΡŒΡŽ ΠΏΠΎΡ‚Π΅Π½Ρ†ΠΈΠ°Π»ΠΎΠ².

Глоссарий ΠΏΠΎ Ρ‚Π΅ΠΌΠ΅

ЭлСктричСская Ρ†Π΅ΠΏΡŒ – Π½Π°Π±ΠΎΡ€ устройств, ΠΊΠΎΡ‚ΠΎΡ€Ρ‹Π΅ соСдинСны ΠΏΡ€ΠΎΠ²ΠΎΠ΄Π½ΠΈΠΊΠ°ΠΌΠΈ, ΠΏΡ€Π΅Π΄Π½Π°Π·Π½Π°Ρ‡Π΅Π½Π½Ρ‹ΠΉ для протСкания Ρ‚ΠΎΠΊΠ°.

ЭлСктродвиТущая сила – это ΠΎΡ‚Π½ΠΎΡˆΠ΅Π½ΠΈΠ΅ Ρ€Π°Π±ΠΎΡ‚Ρ‹ сторонних сил ΠΏΡ€ΠΈ ΠΏΠ΅Ρ€Π΅ΠΌΠ΅Ρ‰Π΅Π½ΠΈΠΈ заряда ΠΏΠΎ Π·Π°ΠΌΠΊΠ½ΡƒΡ‚ΠΎΠΌΡƒ ΠΊΠΎΠ½Ρ‚ΡƒΡ€Ρƒ ΠΊ Π°Π±ΡΠΎΠ»ΡŽΡ‚Π½ΠΎΠΉ Π²Π΅Π»ΠΈΡ‡ΠΈΠ½Π΅ этого заряда.

Π—Π°ΠΊΠΎΠ½ Ома для ΠΏΠΎΠ»Π½ΠΎΠΉ Ρ†Π΅ΠΏΠΈ: сила Ρ‚ΠΎΠΊΠ° Π² ΠΏΠΎΠ»Π½ΠΎΠΉ Ρ†Π΅ΠΏΠΈ Ρ€Π°Π²Π½Π° ΠΎΡ‚Π½ΠΎΡˆΠ΅Π½ΠΈΡŽ Π­Π”Π‘ Ρ†Π΅ΠΏΠΈ ΠΊ Π΅Π΅ ΠΏΠΎΠ»Π½ΠΎΠΌΡƒ ΡΠΎΠΏΡ€ΠΎΡ‚ΠΈΠ²Π»Π΅Π½ΠΈΡŽ:

Основная ΠΈ Π΄ΠΎΠΏΠΎΠ»Π½ΠΈΡ‚Π΅Π»ΡŒΠ½Π°Ρ Π»ΠΈΡ‚Π΅Ρ€Π°Ρ‚ΡƒΡ€Π° ΠΏΠΎ Ρ‚Π΅ΠΌΠ΅ ΡƒΡ€ΠΎΠΊΠ°:

1. МякишСв Π“. Π―., Π‘ΡƒΡ…ΠΎΠ²Ρ†Π΅Π² Π‘. Π‘., Ботский Н.Н. Π€ΠΈΠ·ΠΈΠΊΠ°. 10 класс. Π£Ρ‡Π΅Π±Π½ΠΈΠΊ для ΠΎΠ±Ρ‰Π΅ΠΎΠ±Ρ€Π°Π·ΠΎΠ²Π°Ρ‚Π΅Π»ΡŒΠ½Ρ‹Ρ… ΠΎΡ€Π³Π°Π½ΠΈΠ·Π°Ρ†ΠΈΠΉ М.

: ΠŸΡ€ΠΎΡΠ²Π΅Ρ‰Π΅Π½ΠΈΠ΅, 2017. Π‘. 348 – 354.

2.Π Ρ‹ΠΌΠΊΠ΅Π²ΠΈΡ‡ А. П. Π‘Π±ΠΎΡ€Π½ΠΈΠΊ Π·Π°Π΄Π°Ρ‡ ΠΏΠΎ Ρ„ΠΈΠ·ΠΈΠΊΠ΅. 10-11 класс. – М.: Π”Ρ€ΠΎΡ„Π°, 2009. Π‘. 106-108.

ВСорСтичСский ΠΌΠ°Ρ‚Π΅Ρ€ΠΈΠ°Π» для ΡΠ°ΠΌΠΎΡΡ‚ΠΎΡΡ‚Π΅Π»ΡŒΠ½ΠΎΠ³ΠΎ изучСния

Π›ΡŽΠ±Ρ‹Π΅ силы, ΠΊΠΎΡ‚ΠΎΡ€Ρ‹Π΅ Π΄Π΅ΠΉΡΡ‚Π²ΡƒΡŽΡ‚ Π½Π° элСктричСски заряТСнныС частицы, ΠΊΡ€ΠΎΠΌΠ΅ сил элСктростатичСского происхоТдСния (Ρ‚.Π΅. кулоновских), Π½Π°Π·Ρ‹Π²Π°ΡŽΡ‚ сторонними силами. Π‘Ρ‚ΠΎΡ€ΠΎΠ½Π½ΠΈΠ΅ силы приводят Π² Π΄Π²ΠΈΠΆΠ΅Π½ΠΈΠ΅ заряТСнныС частицы Π²Π½ΡƒΡ‚Ρ€ΠΈ всСх источников Ρ‚ΠΎΠΊΠ°.

ДСйствиС сторонних сил характСризуСтся Π²Π°ΠΆΠ½ΠΎΠΉ физичСской Π²Π΅Π»ΠΈΡ‡ΠΈΠ½ΠΎΠΉ элСктродвиТущСй силой (Π­Π”Π‘). ЭлСктродвиТущая сила Π² Π·Π°ΠΌΠΊΠ½ΡƒΡ‚ΠΎΠΌ ΠΊΠΎΠ½Ρ‚ΡƒΡ€Π΅ – ΠΎΡ‚Π½ΠΎΡˆΠ΅Π½ΠΈΠ΅ Ρ€Π°Π±ΠΎΡ‚Ρ‹ сторонних сил ΠΏΡ€ΠΈ ΠΏΠ΅Ρ€Π΅ΠΌΠ΅Ρ‰Π΅Π½ΠΈΠΈ заряда вдоль ΠΊΠΎΠ½Ρ‚ΡƒΡ€Π° ΠΊ заряду.

Π’ источникС Ρ‚ΠΎΠΊΠ° ΠΈΠ·-Π·Π° дСйствиСм сторонних сил происходит Ρ€Π°Π·Π΄Π΅Π»Π΅Π½ΠΈΠ΅ зарядов. Π’Π°ΠΊ ΠΊΠ°ΠΊ ΠΎΠ½ΠΈ двиТутся, ΠΎΠ½ΠΈ Π²Π·Π°ΠΈΠΌΠΎΠ΄Π΅ΠΉΡΡ‚Π²ΡƒΡŽΡ‚ с ΠΈΠΎΠ½Π°ΠΌΠΈ кристаллов ΠΈ элСктролитов ΠΈ ΠΎΡ‚Π΄Π°ΡŽΡ‚ ΠΈΠΌ Ρ‡Π°ΡΡ‚ΡŒ своСй энСргии. Π­Ρ‚ΠΎ ΠΏΡ€ΠΈΠ²ΠΎΠ΄ΠΈΡ‚ ΠΊ ΡƒΠΌΠ΅Π½ΡŒΡˆΠ΅Π½ΠΈΡŽ силы Ρ‚ΠΎΠΊΠ°, Ρ‚Π°ΠΊΠΈΠΌ ΠΎΠ±Ρ€Π°Π·ΠΎΠΌ, источник Ρ‚ΠΎΠΊΠ° ΠΎΠ±Π»Π°Π΄Π°Π΅Ρ‚ сопротивлСниСм, ΠΊΠΎΡ‚ΠΎΡ€ΠΎΠ΅ Π½Π°Π·Ρ‹Π²Π°ΡŽΡ‚

Π²Π½ΡƒΡ‚Ρ€Π΅Π½Π½ΠΈΠΌ r.

Π—Π°ΠΊΠΎΠ½ Ома для Π·Π°ΠΌΠΊΠ½ΡƒΡ‚ΠΎΠΉ Ρ†Π΅ΠΏΠΈ связываСт силу Ρ‚ΠΎΠΊΠ° Π² Ρ†Π΅ΠΏΠΈ, Π­Π”Π‘ ΠΈ ΠΏΠΎΠ»Π½ΠΎΠ΅ сопротивлСниС Ρ†Π΅ΠΏΠΈ:

Π‘ΠΈΠ»Π° Ρ‚ΠΎΠΊΠ° Π² ΠΏΠΎΠ»Π½ΠΎΠΉ Ρ†Π΅ΠΏΠΈ Ρ€Π°Π²Π½Π° ΠΎΡ‚Π½ΠΎΡˆΠ΅Π½ΠΈΡŽ Π­Π”Π‘ Ρ†Π΅ΠΏΠΈ ΠΊ Π΅Π΅ ΠΏΠΎΠ»Π½ΠΎΠΌΡƒ ΡΠΎΠΏΡ€ΠΎΡ‚ΠΈΠ²Π»Π΅Π½ΠΈΡŽ

ΠšΠΎΡ€ΠΎΡ‚ΠΊΠΎΠ΅ Π·Π°ΠΌΡ‹ΠΊΠ°Π½ΠΈΠ΅

ΠŸΡ€ΠΈ ΠΊΠΎΡ€ΠΎΡ‚ΠΊΠΎΠΌ Π·Π°ΠΌΡ‹ΠΊΠ°Π½ΠΈΠΈ, ΠΊΠΎΠ³Π΄Π° внСшнСС сопротивлСниС стрСмится ΠΊ Π½ΡƒΠ»ΡŽ , сила Ρ‚ΠΎΠΊΠ° Π² Ρ†Π΅ΠΏΠΈ опрСдСляСтся ΠΈΠΌΠ΅Π½Π½ΠΎ Π²Π½ΡƒΡ‚Ρ€Π΅Π½Π½ΠΈΠΌ сопротивлСниСм ΠΈ ΠΌΠΎΠΆΠ΅Ρ‚ ΠΎΠΊΠ°Π·Π°Ρ‚ΡŒΡΡ ΠΎΡ‡Π΅Π½ΡŒ большой . И Ρ‚ΠΎΠ³Π΄Π° ΠΏΡ€ΠΎΠ²ΠΎΠ΄Π° ΠΌΠΎΠ³ΡƒΡ‚ Ρ€Π°ΡΠΏΠ»Π°Π²ΠΈΡ‚ΡŒΡΡ, Ρ‡Ρ‚ΠΎ ΠΌΠΎΠΆΠ΅Ρ‚ привСсти ΠΊ опасным послСдствиям.

ΠŸΡ€ΠΈΠΌΠ΅Ρ€Ρ‹ ΠΈ Ρ€Π°Π·Π±ΠΎΡ€ Ρ€Π΅ΡˆΠ΅Π½ΠΈΡ Π·Π°Π΄Π°Π½ΠΈΠΉ:

1. К ΠΊΠ°ΠΆΠ΄ΠΎΠΉ ΠΏΠΎΠ·ΠΈΡ†ΠΈΠΈ ΠΏΠ΅Ρ€Π²ΠΎΠ³ΠΎ столбца ΠΏΠΎΠ΄Π±Π΅Ρ€ΠΈΡ‚Π΅ ΡΠΎΠΎΡ‚Π²Π΅Ρ‚ΡΡ‚Π²ΡƒΡŽΡ‰ΡƒΡŽ ΠΏΠΎΠ·ΠΈΡ†ΠΈΡŽ Π²Ρ‚ΠΎΡ€ΠΎΠ³ΠΎ:

Π€Π˜Π—Π˜Π§Π•Π‘ΠšΠ˜Π• Π’Π•Π›Π˜Π§Π˜ΠΠ«

Π€ΠžΠ ΠœΠ£Π›Π«

ЭлСктродвиТущая сила

Π‘ΠΈΠ»Π° Ρ‚ΠΎΠΊΠ°

Π‘ΠΎΠΏΡ€ΠΎΡ‚ΠΈΠ²Π»Π΅Π½ΠΈΠ΅

Π Π°Π·Π½ΠΎΡΡ‚ΡŒ ΠΏΠΎΡ‚Π΅Π½Ρ†ΠΈΠ°Π»ΠΎΠ²

РСшСниС.

ЭлСктродвиТущая сила Π³Π°Π»ΡŒΠ²Π°Π½ΠΈΡ‡Π΅ΡΠΊΠΎΠ³ΠΎ элСмСнта Π΅ΡΡ‚ΡŒ Π²Π΅Π»ΠΈΡ‡ΠΈΠ½Π°, числСнно равная Ρ€Π°Π±ΠΎΡ‚Π΅ сторонних сил ΠΏΡ€ΠΈ ΠΏΠ΅Ρ€Π΅ΠΌΠ΅Ρ‰Π΅Π½ΠΈΠΈ Π΅Π΄ΠΈΠ½ΠΈΡ‡Π½ΠΎΠ³ΠΎ ΠΏΠΎΠ»ΠΎΠΆΠΈΡ‚Π΅Π»ΡŒΠ½ΠΎΠ³ΠΎ заряда Π²Π½ΡƒΡ‚Ρ€ΠΈ элСмСнта ΠΎΡ‚ ΠΎΠ΄Π½ΠΎΠ³ΠΎ полюса ΠΊ Π΄Ρ€ΡƒΠ³ΠΎΠΌΡƒ.

Π Π°Π±ΠΎΡ‚Π° сторонних сил Π½Π΅ ΠΌΠΎΠΆΠ΅Ρ‚ Π±Ρ‹Ρ‚ΡŒ Π²Ρ‹Ρ€Π°ΠΆΠ΅Π½Π° Ρ‡Π΅Ρ€Π΅Π· Ρ€Π°Π·Π½ΠΎΡΡ‚ΡŒ ΠΏΠΎΡ‚Π΅Π½Ρ†ΠΈΠ°Π»ΠΎΠ², Ρ‚Π°ΠΊ ΠΊΠ°ΠΊ сторонниС силы Π½Π΅ΠΏΠΎΡ‚Π΅Π½Ρ†ΠΈΠ°Π»ΡŒΠ½Ρ‹ ΠΈ ΠΈΡ… Ρ€Π°Π±ΠΎΡ‚Π° зависит ΠΎΡ‚ Ρ„ΠΎΡ€ΠΌΡ‹ Ρ‚Ρ€Π°Π΅ΠΊΡ‚ΠΎΡ€ΠΈΠΈ пСрСмСщСния зарядов.

Π­Π”Π‘ опрСдСляСтся ΠΏΠΎ Ρ„ΠΎΡ€ΠΌΡƒΠ»Π΅:

Π‘ΠΈΠ»Π° Ρ‚ΠΎΠΊΠ° опрСдСляСтся ΠΏΠΎ Ρ„ΠΎΡ€ΠΌΡƒΠ»Π΅:

Π‘ΠΎΠΏΡ€ΠΎΡ‚ΠΈΠ²Π»Π΅Π½ΠΈΠ΅ опрСдСляСтся ΠΏΠΎ Ρ„ΠΎΡ€ΠΌΡƒΠ»Π΅:

Π Π°Π·Π½ΠΎΡΡ‚ΡŒ ΠΏΠΎΡ‚Π΅Π½Ρ†ΠΈΠ°Π»ΠΎΠ² опрСдСляСтся ΠΏΠΎ Ρ„ΠΎΡ€ΠΌΡƒΠ»Π΅:

ΠŸΡ€Π°Π²ΠΈΠ»ΡŒΠ½Ρ‹ΠΉ ΠΎΡ‚Π²Π΅Ρ‚:

Π€Π˜Π—Π˜Π§Π•Π‘ΠšΠ˜Π• Π’Π•Π›Π˜Π§Π˜ΠΠ«

Π€ΠžΠ ΠœΠ£Π›Π«

ЭлСктродвиТущая сила

Π‘ΠΈΠ»Π° Ρ‚ΠΎΠΊΠ°

Π‘ΠΎΠΏΡ€ΠΎΡ‚ΠΈΠ²Π»Π΅Π½ΠΈΠ΅

Π Π°Π·Π½ΠΎΡΡ‚ΡŒ ΠΏΠΎΡ‚Π΅Π½Ρ†ΠΈΠ°Π»ΠΎΠ²

2. Π­Π”Π‘ Π±Π°Ρ‚Π°Ρ€Π΅ΠΉΠΊΠΈ ΠΊΠ°Ρ€ΠΌΠ°Π½Π½ΠΎΠ³ΠΎ Ρ„ΠΎΠ½Π°Ρ€ΠΈΠΊΠ° – 3,7 Π’, Π²Π½ΡƒΡ‚Ρ€Π΅Π½Π½Π΅Π΅ сопротивлСниС 1,5 Ом. Π‘Π°Ρ‚Π°Ρ€Π΅ΠΉΠΊΠ° Π·Π°ΠΌΠΊΠ½ΡƒΡ‚Π° Π½Π° сопротивлСниС 11,7 Ом. Каково напряТСниС Π½Π° Π·Π°ΠΆΠΈΠΌΠ°Ρ… Π±Π°Ρ‚Π°Ρ€Π΅ΠΉΠΊΠΈ?

РСшСниС:

НапряТСниС рассчитываСтся ΠΏΠΎ Ρ„ΠΎΡ€ΠΌΡƒΠ»Π΅:

Π§Ρ‚ΠΎΠ±Ρ‹ Π½Π°ΠΉΡ‚ΠΈ силу Ρ‚ΠΎΠΊΠ° ΠΏΡ€ΠΈΠΌΠ΅Π½ΠΈΠΌ Π·Π°ΠΊΠΎΠ½ Ома для ΠΏΠΎΠ»Π½ΠΎΠΉ Ρ†Π΅ΠΏΠΈ:

Π”Π΅Π»Π°Π΅ΠΌ расчёт:

ΠžΡ‚Π²Π΅Ρ‚: U = 3,28 Π’.

ЛСкция 6. НаправлСниС эдс, Ρ‚ΠΎΠΊΠ°, напряТСния. Π’Ρ‚ΠΎΡ€ΠΎΠΉ Π·Π°ΠΊΠΎΠ½ ΠšΠΈΡ€Ρ…Π³ΠΎΡ„Π°.ЭлСктричСскиС Ρ†Π΅ΠΏΠΈ ΠΏΠ΅Ρ€Π΅ΠΌΠ΅Π½Π½ΠΎΠ³ΠΎ Ρ‚ΠΎΠΊΠ°. Π₯арактСристики ΠΏΠ΅Ρ€Π΅ΠΌΠ΅Π½Π½ΠΎΠ³ΠΎ Ρ‚ΠΎΠΊΠ°

Для ΠΎΠ΄Π½ΠΎΠ·Π½Π°Ρ‡Π½ΠΎΠ³ΠΎ описания процСссов Π² элСктричСской Ρ†Π΅ΠΏΠΈ Π½Π΅ΠΎΠ±Ρ…ΠΎΠ΄ΠΈΠΌΠΎ Π·Π½Π°Ρ‚ΡŒ Π½Π΅ Ρ‚ΠΎΠ»ΡŒΠΊΠΎ Π·Π½Π°Ρ‡Π΅Π½ΠΈΠ΅ Π²Π΅Π»ΠΈΡ‡ΠΈΠ½, Π½ΠΎ ΠΈ Π½Π°ΠΏΡ€Π°Π²Π»Π΅Π½ΠΈΠ΅ этих Π²Π΅Π»ΠΈΡ‡ΠΈΠ½.

Π—Π° Π½Π°ΠΏΡ€Π°Π²Π»Π΅Π½ΠΈΠ΅ Ρ‚ΠΎΠΊΠ° принято Π΄Π²ΠΈΠΆΠ΅Π½ΠΈΠ΅ ΠΏΠΎΠ»ΠΎΠΆΠΈΡ‚Π΅Π»ΡŒΠ½Ρ‹Ρ… зарядов ΠΎΡ‚ большСго ΠΏΠΎΡ‚Π΅Π½Ρ†ΠΈΠ°Π»Π° ΠΊ ΠΌΠ΅Π½ΡŒΡˆΠ΅ΠΌΡƒ (φА>Ο†Π’).

НаправлСниС напряТСния Π² элСмСнтС элСктричСской Ρ†Π΅ΠΏΠΈ совпадаСт с Π½Π°ΠΏΡ€Π°Π²Π»Π΅Π½ΠΈΠ΅ΠΌ Ρ‚ΠΎΠΊΠ° Π² Π΄Π°Π½Π½ΠΎΠΌ элСмСнтС (рис. 6.1).

Рис. 6.1. НаправлСниС Ρ‚ΠΎΠΊΠ° ΠΈ напряТСния

Рис.6.2. НаправлСниС Π­Π”Π‘ Π²Π½ΡƒΡ‚Ρ€ΠΈ источника

НаправлСниС Π­Π”Π‘ Π²Π½ΡƒΡ‚Ρ€ΠΈ источника – Π² сторону большСго ΠΏΠΎΡ‚Π΅Π½Ρ†ΠΈΠ°Π»Π° (рис. 6. 2).

Π­Π”Π‘ самоиндукции Π½Π°ΠΏΡ€Π°Π²Π»Π΅Π½Π° Π²Π½ΡƒΡ‚Ρ€ΠΈ ΠΊΠ°Ρ‚ΡƒΡˆΠΊΠΈ Π² сторону большСго ΠΏΠΎΡ‚Π΅Π½Ρ†ΠΈΠ°Π»Π° (рис. 6.3).

Рис. 6.3 – НаправлСниС Π­Π”Π‘ Π²Π½ΡƒΡ‚Ρ€ΠΈ ΠΊΠ°Ρ‚ΡƒΡˆΠΊΠΈ

ΠŸΡ€ΠΈ ΡƒΠΊΠ°Π·Π°Π½Π½ΠΎΠΌ Π½Π°ΠΏΡ€Π°Π²Π»Π΅Π½ΠΈΠΈ Π­Π”Π‘ самоиндукции ΠΏΡ€Π°Π²ΠΈΠ»ΠΎ Π›Π΅Π½Ρ†Π° ΡƒΠΆΠ΅ ΡƒΡ‡Ρ‚Π΅Π½ΠΎ.

. (6.1)

Π•Π΄ΠΈΠ½ΠΈΡ†Π° измСрСния Π­Π”Π‘: [e] = 1 Π’.

Π’Ρ‚ΠΎΡ€ΠΎΠΉ Π·Π°ΠΊΠΎΠ½ ΠšΠΈΡ€Ρ…Π³ΠΎΡ„Π°

АлгСбраичСская сумма ΠΏΠ°Π΄Π΅Π½ΠΈΠΉ напряТСний Π² любом Π·Π°ΠΌΠΊΠ½ΡƒΡ‚ΠΎΠΌ ΠΊΠΎΠ½Ρ‚ΡƒΡ€Π΅ числСнно Ρ€Π°Π²Π½Π° алгСбраичСской суммС Π­Π”Π‘, Π΄Π΅ΠΉΡΡ‚Π²ΡƒΡŽΡ‰ΠΈΡ… Π² этом ΠΊΠΎΠ½Ρ‚ΡƒΡ€Π΅:

, (6.2)

Π³Π΄Π΅ – ΠΏΠ°Π΄Π΅Π½ΠΈΠ΅ напряТСния Π½Π°

i-Ρ‚ΠΎΠΌ элСмСнтС элСктричСской Ρ†Π΅ΠΏΠΈ;n, m – соотвСтствСнно число элСмСнтов ΠΈ источников Π·Π°ΠΌΠΊΠ½ΡƒΡ‚ΠΎΠ³ΠΎ участка элСктричСской Ρ†Π΅ΠΏΠΈ;– Π­Π”Π‘k-Ρ‚ΠΎΠ³ΠΎ источника.

АлгСбраичСская сумма ΠΎΠ·Π½Π°Ρ‡Π°Π΅Ρ‚, Ρ‡Ρ‚ΠΎ Ρ‚ΠΎΠΊΠΈ, напряТСния, Π­Π”Π‘ (I,U,e) ΠΌΠΎΠ³ΡƒΡ‚ Π±Ρ€Π°Ρ‚ΡŒΡΡ со Π·Π½Π°ΠΊΠΎΠΌ Β«+Β» ΠΈΠ»ΠΈ со Π·Π½Π°ΠΊΠΎΠΌ «–».

НаправлСния ΠΎΠ±Ρ…ΠΎΠ΄Π° ΠΊΠΎΠ½Ρ‚ΡƒΡ€Π° ΠΈ Ρ‚ΠΎΠΊΠΎΠ² Π² вСтвях Ρ†Π΅ΠΏΠΈ выбираСтся ΠΏΡ€ΠΎΠΈΠ·Π²ΠΎΠ»ΡŒΠ½ΠΎ.Π­Π”Π‘ ΠΈ падС­ния напряТСния, ΡΠΎΠ²ΠΏΠ°Π΄Π°ΡŽΡ‰ΠΈΠ΅ ΠΏΠΎ Π½Π°ΠΏΡ€Π°Π²Π»Π΅Π½ΠΈΡŽ с Π½Π°ΠΏΡ€Π°Π²Π»Π΅Π½ΠΈΠ΅ΠΌ ΠΎΠ±Ρ…ΠΎΠ΄Π°, бСрутся со Π·Π½Π°ΠΊΠΎΠΌ Β«+Β», ΠΈΠ½Π°Ρ‡Π΅ – со Π·Π½Π°ΠΊΠΎΠΌ «–».

Рассмотрим Π·Π°ΠΌΠΊΠ½ΡƒΡ‚Ρ‹ΠΉ участок элСктричСской Ρ†Π΅ΠΏΠΈ, ΠΊΠΎΡ‚ΠΎΡ€Ρ‹ΠΉ прСдставляСт Ρ‡Π°ΡΡ‚ΡŒ Π±ΠΎΠ»Π΅Π΅ слоТной элСктричСской схСмы (рис. 6.3).

Рис. 6.3. Π—Π°ΠΌΠΊΠ½ΡƒΡ‚Ρ‹ΠΉ участок элСктричСской Ρ†Π΅ΠΏΠΈ

Богласно Π²Ρ‹Ρ€Π°ΠΆΠ΅Π½ΠΈΡŽ (6.2) для рис. (6.3) ΡƒΡ€Π°Π²Π½Π΅Π½ΠΈΠ΅ ΠΈΠΌΠ΅Π΅Ρ‚ Π²ΠΈΠ΄:

. (6.4)

ЭлСктричСскиС Ρ†Π΅ΠΏΠΈ ΠΏΠ΅Ρ€Π΅ΠΌΠ΅Π½Π½ΠΎΠ³ΠΎ Ρ‚ΠΎΠΊΠ°. Π₯арактСристики ΠΏΠ΅Ρ€Π΅ΠΌΠ΅Π½Π½ΠΎΠ³ΠΎ Ρ‚ΠΎΠΊΠ°

ЭлСктричСская энСргия Π² Π±ΠΎΠ»ΡŒΡˆΠΈΠ½ΡΡ‚Π²Π΅ случаСв производится, распрСдСляСтся, потрСбляСтся Π² Π²ΠΈΠ΄Π΅ элСктроэнСргии ΠΏΠ΅Ρ€Π΅ΠΌΠ΅Π½Π½ΠΎΠ³ΠΎ Ρ‚ΠΎΠΊΠ°. Π’ ΠΏΠ΅Ρ€Π²ΡƒΡŽ ΠΎΡ‡Π΅Ρ€Π΅Π΄ΡŒ это обусловлСно Ρ‚Π΅ΠΌ, Ρ‡Ρ‚ΠΎ ΠΏΠ΅Ρ€Π΅ΠΌΠ΅Π½Π½Ρ‹ΠΉ Ρ‚ΠΎΠΊ Π»Π΅Π³ΠΊΠΎ ΠΏΠ΅Ρ€Π΅Π΄Π°Π²Π°Ρ‚ΡŒ с ΠΎΠ΄Π½ΠΎΠ³ΠΎ мСста Π² Π΄Ρ€ΡƒΠ³ΠΎΠ΅.

Π’ цСпях ΠΏΠ΅Ρ€Π΅ΠΌΠ΅Π½Π½ΠΎΠ³ΠΎ Ρ‚ΠΎΠΊΠ° Π·Π½Π°Ρ‡Π΅Π½ΠΈΠ΅ Ρ‚ΠΎΠΊΠ°, напряТСния, Π­Π”Π‘ пСриодичСски ΠΌΠ΅Π½ΡΡŽΡ‚ΡΡ ΠΏΠΎ гармоничСскому Π·Π°ΠΊΠΎΠ½Ρƒ, Π° сами измСнСния Π²Π΅Π»ΠΈΡ‡ΠΈΠ½ Π½Π°Π·Ρ‹Π²Π°ΡŽΡ‚ΡΡ гармоничСскими колСбаниями

;, (6.5)

Π³Π΄Π΅ х– пСрСмСнная функция, Ρ€ΠΎΠ»ΡŒ ΠΊΠΎΡ‚ΠΎΡ€ΠΎΠΉ ΠΌΠΎΠ³ΡƒΡ‚ ΠΈΠ³Ρ€Π°Ρ‚ΡŒi,u,e, ΠΈ Ρ‚.Π΄.;

А – Π°ΠΌΠΏΠ»ΠΈΡ‚ΡƒΠ΄Π° ΠΊΠΎΠ»Π΅Π±Π°Π½ΠΈΠΉ, Ρ‚.Π΅. максимальноС Π·Π½Π°Ρ‡Π΅Π½ΠΈΠ΅ ΠΊΠΎΠ»Π΅Π±Π»ΡŽΡ‰Π΅ΠΉΡΡ Π²Π΅Π»ΠΈΡ‡ΠΈΠ½Ρ‹;

– полная Ρ„Π°Π·Π° ΠΊΠΎΠ»Π΅Π±Π°Π½ΠΈΠΉ;

– цикличСская частота собствСнных гармоничСских ΠΊΠΎΠ»Π΅Π±Π°Π½ΠΈΠΉ.

Π—Π½Π°Ρ‡Π΅Π½ΠΈΠ΅ А удовлСтворяСт ΡΠ»Π΅Π΄ΡƒΡŽΡ‰ΠΈΠΌ условиям:

1);

2) Ρ‚Π°ΠΊ ΠΊΠ°ΠΊ , Ρ‚ΠΎ.

Амплитуда опрСдСляСтся ΠΏΠ΅Ρ€Π²ΠΎΠ½Π°Ρ‡Π°Π»ΡŒΠ½Ρ‹ΠΌ Ρ‚ΠΎΠ»Ρ‡ΠΊΠΎΠΌ энСргии, ΠΊΠΎΡ‚ΠΎΡ€Ρ‹ΠΉ Π²Ρ‹Π²ΠΎΠ΄ΠΈΡ‚ ΠΊΠΎΠ»Π΅Π±Π»ΡŽΡ‰ΡƒΡŽΡΡ (ΡΠ½Π΅Ρ€Π³ΠΈΡŽ) систСму ΠΈΠ· полоТСния равновСсия.

ΠŸΡƒΡΡ‚ΡŒ , Ρ‚ΠΎΠ³Π΄Π° ΠΏΡ€ΠΈ, Ρ‚.Π΅.– Ρ„Π°Π·Π° колСбания Π² Π½Π°Ρ‡Π°Π»ΡŒΠ½Ρ‹ΠΉ ΠΌΠΎΠΌΠ΅Π½Ρ‚ Π²Ρ€Π΅ΠΌΠ΅Π½ΠΈ. Она называСтся Π½Π°Ρ‡Π°Π»ΡŒΠ½ΠΎΠΉ Ρ„Π°Π·ΠΎΠΉ колСбания ΠΈ опрСдСляСтся Π²Ρ‹Π±ΠΎΡ€ΠΎΠΌ Π½Π°Ρ‡Π°Π»Π° отсчСта Π²Ρ€Π΅ΠΌΠ΅Π½ΠΈ.

ΠŸΠ΅Ρ€ΠΈΠΎΠ΄ΠΎΠΌ Π’ называСтся ΠΏΡ€ΠΎΠΌΠ΅ΠΆΡƒΡ‚ΠΎΠΊ Π²Ρ€Π΅ΠΌΠ΅Π½ΠΈ, Π·Π° ΠΊΠΎΡ‚ΠΎΡ€Ρ‹ΠΉ Ρ„Π°Π·Π° ΠΊΠΎΠ»Π΅Π±Π°Π½ΠΈΠΉ измСняСтся Π½Π° 2Ο€, Ρ€Π°Π·ΠΌΠ΅Ρ€Π½ΠΎΡΡ‚ΡŒ ΠΏΠ΅Ρ€ΠΈΠΎΠ΄Π° .

Рассмотрим Π΄Π²Π° ΠΌΠΎΠΌΠ΅Π½Ρ‚Π° Π²Ρ€Π΅ΠΌΠ΅Π½ΠΈ t1ΠΌt2:

; (6.7)

; (6.8)

. (6.9)

Богласно ΠΎΠΏΡ€Π΅Π΄Π΅Π»Π΅Π½ΠΈΡŽ ΠΏΠ΅Ρ€ΠΈΠΎΠ΄Π°

ΠΈΠ»ΠΈ , (6.10)

Π•Π΄ΠΈΠ½ΠΈΡ†Π° цикличСской частоты: .

Частота ν– число ΠΏΠΎΠ»Π½Ρ‹Ρ… ΠΊΠΎΠ»Π΅Π±Π°Π½ΠΈΠΉ Π·Π° 1 сСкунду.

. (6.11)

Π•Π΄ΠΈΠ½ΠΈΡ†Π° частоты:.

1 Π“Ρ† – это частота Ρ‚Π°ΠΊΠΈΡ… ΠΊΠΎΠ»Π΅Π±Π°Π½ΠΈΠΉ, ΠΏΡ€ΠΈ ΠΊΠΎΡ‚ΠΎΡ€Ρ‹Ρ… Π·Π° 1 сСкунду ΡΠΎΠ²Π΅Ρ€ΡˆΠ°Π΅Ρ‚ΡΡ ΠΎΠ΄Π½ΠΎ ΠΏΠΎΠ»Π½ΠΎΠ΅ ΠΊΠΎΠ»Π΅Π±Π°Π½ΠΈΠ΅.

, (6.12)

Π’Π°ΠΊΠΈΠΌ ΠΎΠ±Ρ€Π°Π·ΠΎΠΌ, – физичСская Π²Π΅Π»ΠΈΡ‡ΠΈΠ½Π°, числСнно равная числу ΠΏΠΎΠ»Π½Ρ‹Ρ… ΠΊΠΎΠ»Π΅Π±Π°Π½ΠΈΠΉ Π·Π° врСмя.

Рис. 6.4. Π“Ρ€Π°Ρ„ΠΈΠΊ гармоничСских ΠΊΠΎΠ»Π΅Π±Π°Π½ΠΈΠΉ

Π¦Π΅Π»Π΅ΡΠΎΠΎΠ±Ρ€Π°Π·Π½ΠΎΡΡ‚ΡŒ использования гармоничСских Π·Π°ΠΊΠΎΠ½ΠΎΠ² ΠΏΠΎ ΡΡ€Π°Π²Π½Π΅Π½ΠΈΡŽ с нСгармоничСскими обусловлСна ΡΠ»Π΅Π΄ΡƒΡŽΡ‰ΠΈΠΌΠΈ Ρ„Π°ΠΊΡ‚ΠΎΡ€Π°ΠΌΠΈ:

  • большими значСниями ΠšΠŸΠ” Π³Π΅Π½Π΅Ρ€Π°Ρ‚ΠΎΡ€ΠΎΠ², Π΄Π²ΠΈΠ³Π°Ρ‚Π΅Π»Π΅ΠΉ, трансформаторов;

1.3. Π˜ΡΡ‚ΠΎΡ‡Π½ΠΈΠΊΠΈ Π­Π”Π‘ ΠΈ Ρ‚ΠΎΠΊΠ°

К Π°ΠΊΡ‚ΠΈΠ²Π½Ρ‹ΠΌ элСмСнтам элСктричСских Ρ†Π΅ΠΏΠ΅ΠΉ относятся источники Π­Π”Π‘ ΠΈ источники Ρ‚ΠΎΠΊΠ°.

ЭлСктродвиТущая сила (Π­Π”Π‘) – это количСство энСргии, Π·Π°Ρ‚Ρ€Π°Ρ‡Π΅Π½Π½ΠΎΠ΅ сторонними силами Π½Π° пСрСнос Π΅Π΄ΠΈΠ½ΠΈΡ‡Π½ΠΎΠ³ΠΎ ΠΏΠΎΠ»ΠΎΠΆΠΈΡ‚Π΅Π»ΡŒΠ½ΠΎΠ³ΠΎ заряда ΠΎΡ‚ мСньшСго ΠΏΠΎΡ‚Π΅Π½Ρ†ΠΈΠ°Π»Π° ΠΊ Π±ΠΎΠ»ΡŒΡˆΠ΅ΠΌΡƒ

Π—Π° ΠΏΠΎΠ»ΠΎΠΆΠΈΡ‚Π΅Π»ΡŒΠ½ΠΎΠ΅ Π½Π°ΠΏΡ€Π°Π²Π»Π΅Π½ΠΈΠ΅ э.Π΄.с. принимаСтся Π½Π°ΠΏΡ€Π°Π²Π»Π΅Π½ΠΈΠ΅ возрастания ΠΏΠΎΡ‚Π΅Π½Ρ†ΠΈΠ°Π»Π° (рис. 1.6).

Β 

Π’Π°ΠΊΠΈΠΌ ΠΎΠ±Ρ€Π°Π·ΠΎΠΌ, ΠΏΠΎΠ»ΠΎΠΆΠΈΡ‚Π΅Π»ΡŒΠ½Ρ‹Π΅ направлСния Β Π­Π”Π‘ ΠΈ напряТСния всСгда ΠΏΡ€ΠΎΡ‚ΠΈΠ²ΠΎΠΏΠΎΠ»ΠΎΠΆΠ½Ρ‹.

ЧислСнно Π­Π”Π‘ Ρ€Π°Π²Π½Π° разности ΠΏΠΎΡ‚Π΅Π½Ρ†ΠΈΠ°Π»ΠΎΠ² ΠΌΠ΅ΠΆΠ΄Ρƒ Π²Ρ‹Π²ΠΎΠ΄Π°ΠΌΠΈ источника ΠΏΡ€ΠΈ Ρ€Π°Π·ΠΎΠΌΠΊΠ½ΡƒΡ‚ΠΎΠΉ Ρ†Π΅ΠΏΠΈ.

Если Π²Π½ΡƒΡ‚Ρ€ΠΈ источника Β Π­Π”Π‘ Π½Π΅ содСрТится пассивных элСмСнтов, Ρ‚ΠΎ Π΅Π³ΠΎ Π²Π½ΡƒΡ‚Ρ€Π΅Π½Π½Π΅Π΅ сопротивлСниС r0Β Ρ€Π°Π²Π½ΠΎ Π½ΡƒΠ»ΡŽ. Π’Π°ΠΊΠΎΠΉ источник  являСтся ΠΈΠ΄Π΅Π°Π»ΡŒΠ½Ρ‹ΠΌ.

На ΠΏΡ€Π°ΠΊΡ‚ΠΈΠΊΠ΅ ΠΎΠ±Ρ‹Ρ‡Π½ΠΎ приходится ΠΈΠΌΠ΅Ρ‚ΡŒ Π΄Π΅Π»ΠΎ с Ρ€Π΅Π°Π»ΡŒΠ½Ρ‹ΠΌΠΈ источниками Β Π­Π”Π‘, ΠΎΠ±Π»Π°Π΄Π°ΡŽΡ‰ΠΈΠΌΠΈ Π½Π΅ΠΊΠΎΡ‚ΠΎΡ€Ρ‹ΠΌ Π²Π½ΡƒΡ‚Ρ€Π΅Π½Π½ΠΈΠΌ сопротивлСниСм (рис. 1.7).

Π’ Ρ‚Π°ΠΊΠΈΡ… источниках напряТСниС Π½Π° Π·Π°ΠΆΠΈΠΌΠ°Ρ… зависит ΠΎΡ‚ Ρ‚ΠΎΠΊΠ° Π² Π½Π°Π³Ρ€ΡƒΠ·ΠΊΠ΅.

НапряТСниС Π½Π° Π·Π°ΠΆΠΈΠΌΠ°Ρ… Ρ€Π΅Π°Π»ΡŒΠ½ΠΎΠ³ΠΎ источника Π² Ρ€Π°Π±ΠΎΡ‚Π°ΡŽΡ‰Π΅ΠΉ Ρ†Π΅ΠΏΠΈ опрСдСляСтся ΡΠΎΠΎΡ‚Π½ΠΎΡˆΠ΅Π½ΠΈΠ΅ΠΌ

Π­Ρ‚ΠΎ Π²Ρ‹Ρ€Π°ΠΆΠ΅Π½ΠΈΠ΅ Π½Π°Π·Ρ‹Π²Π°ΡŽΡ‚ внСшнСй характСристикой источника Β Π­Π”Π‘.

Анализируя внСшнюю характСристику источника, ΠΌΠΎΠΆΠ½ΠΎ ΡΠ΄Π΅Π»Π°Ρ‚ΡŒ Π²Ρ‹Π²ΠΎΠ΄, Ρ‡Ρ‚ΠΎ напряТСниС Π½Π° Π·Π°ΠΆΠΈΠΌΠ°Ρ… источника Π² Ρ€Π΅ΠΆΠΈΠΌΠ΅ Π½Π°Π³Ρ€ΡƒΠ·ΠΊΠΈ всСгда мСньшС Β Π­Π”Π‘ Π½Π° Π²Π΅Π»ΠΈΡ‡ΠΈΠ½Ρƒ падСния напряТСния Π½Π° Π²Π½ΡƒΡ‚Ρ€Π΅Π½Π½Π΅ΠΌ сопротивлСнии источника. Π—Π°Π²ΠΈΡΠΈΠΌΠΎΡΡ‚ΡŒ напряТСния ΠΎΡ‚ Ρ‚ΠΎΠΊΠ° Π½Π°Π³Ρ€ΡƒΠ·ΠΊΠΈ ΠΏΠΎΠΊΠ°Π·Π°Π½Π° Π½Π° рис. 1.8 ΠΏΡƒΠ½ΠΊΡ‚ΠΈΡ€Π½ΠΎΠΉ Π»ΠΈΠ½ΠΈΠ΅ΠΉ. Π’ свою ΠΎΡ‡Π΅Ρ€Π΅Π΄ΡŒ Π²Π΅Π»ΠΈΡ‡ΠΈΠ½Π° Ρ‚ΠΎΠΊΠ° Π½Π°Π³Ρ€ΡƒΠ·ΠΊΠΈ зависит ΠΎΡ‚ сопротивлСния внСшнСй Ρ†Π΅ΠΏΠΈ, поэтому ΠΌΠΎΠΆΠ½ΠΎ ΡΡ‡ΠΈΡ‚Π°Ρ‚ΡŒ, Ρ‡Ρ‚ΠΎ напряТСниС Π½Π° Π·Π°ΠΆΠΈΠΌΠ°Ρ… Ρ€Π΅Π°Π»ΡŒΠ½ΠΎΠ³ΠΎ источника зависит ΠΎΡ‚ сопротивлСния внСшнСй Ρ†Π΅ΠΏΠΈ.

Π’ случаС идСального источника Π²Π½ΡƒΡ‚Ρ€Π΅Π½Π½Π΅ сопротивлСниС Ρ€Π°Π²Π½ΠΎ Π½ΡƒΠ»ΡŽ. НапряТСниС Π½Π° Π·Π°ΠΆΠΈΠΌΠ°Ρ… Ρ‚Π°ΠΊΠΎΠ³ΠΎ источника Π½Π΅ зависит ΠΎΡ‚ Ρ‚ΠΎΠΊΠ° Π½Π°Π³Ρ€ΡƒΠ·ΠΊΠΈ ΠΈ Ρ€Π°Π²Π½ΠΎ Β Π­Π”Π‘ источника UΒ =Β E. Β Π—Π°Π²ΠΈΡΠΈΠΌΠΎΡΡ‚ΡŒ напряТСния ΠΎΡ‚ Ρ‚ΠΎΠΊΠ° Π² идСальном источникС ΠΏΠΎΠΊΠ°Π·Π°Π½Π° Π½Π° рис. 1.8 сплошной Π»ΠΈΠ½ΠΈΠ΅ΠΉ.

Β Π˜ΡΡ‚ΠΎΡ‡Π½ΠΈΠΊΠΈ Ρ‚ΠΎΠΊΠ°

Π˜Π΄Π΅Π°Π»ΠΈΠ·ΠΈΡ€ΠΎΠ²Π°Π½Π½Ρ‹ΠΉ источник Ρ‚ΠΎΠΊΠ° – это Π°ΠΊΡ‚ΠΈΠ²Π½Ρ‹ΠΉ элСмСнт, Ρ‚ΠΎΠΊ ΠΊΠΎΡ‚ΠΎΡ€ΠΎΠ³ΠΎ Π½Π΅ зависит ΠΎΡ‚ напряТСния Π½Π° Π΅Π³ΠΎ Π·Π°ΠΆΠΈΠΌΠ°Ρ….

БчитаСтся, Ρ‡Ρ‚ΠΎ Π²Π½ΡƒΡ‚Ρ€Π΅Π½Π½Π΅Π΅ сопротивлСниС идСального источника бСсконСчно Π²Π΅Π»ΠΈΠΊΠΎ, поэтому ΠΏΠ°Ρ€Π°ΠΌΠ΅Ρ‚Ρ€Ρ‹ внСшнСй Ρ†Π΅ΠΏΠΈ Π½Π΅ Π±ΡƒΠ΄ΡƒΡ‚ ΠΎΠΊΠ°Π·Ρ‹Π²Π°Ρ‚ΡŒ влияния Π½Π° Ρ‚ΠΎΠΊ Π² источникС Ρ‚ΠΎΠΊΠ°. На элСктричСских схСмах источник Ρ‚ΠΎΠΊΠ° обозначаСтся Ρ‚Π°ΠΊ, ΠΊΠ°ΠΊ ΠΏΠΎΠΊΠ°Π·Π°Π½ΠΎ Π½Π° рис. 1.9.


Β Π Π΅Π°Π»ΡŒΠ½Ρ‹ΠΉ источник Ρ‚ΠΎΠΊΠ° ΠΎΠ±Π»Π°Π΄Π°Π΅Ρ‚ ΠΊΠΎΠ½Π΅Ρ‡Π½Ρ‹ΠΌ Π²Π½ΡƒΡ‚Ρ€Π΅Π½Π½ΠΈΠΌ сопротивлСниСм ΠΈΠ»ΠΈ ΠΎΡ‚Π»ΠΈΡ‡Π½ΠΎΠΉ ΠΎΡ‚ нуля ΠΏΡ€ΠΎΠ²ΠΎΠ΄ΠΈΠΌΠΎΡΡ‚ΡŒΡŽ. Π‘Ρ…Π΅ΠΌΠ° Ρ€Π΅Π°Π»ΡŒΠ½ΠΎΠ³ΠΎ источника прСдставлСна Π½Π° рис. 1.10. Π’ΠΎΠΊ Ρ€Π΅Π°Π»ΡŒΠ½ΠΎΠ³ΠΎ источника опрСдСляСтся Ρ€Π°Π·Π½ΠΎΡΡ‚ΡŒΡŽ Ρ‚ΠΎΠΊΠ° идСального источника JΒ ΠΈ Π²Π½ΡƒΡ‚Ρ€Π΅Π½Π½Π΅Π³ΠΎ Ρ‚ΠΎΠΊΠ°Β I0:

Π³Π΄Π΅Β U – напряТСниС, ΠΏΡ€ΠΈΠ»ΠΎΠΆΠ΅Π½Π½ΠΎΠ΅ ΠΊ Π·Π°ΠΆΠΈΠΌΠ°ΠΌ источника. ΠŸΠΎΠ»ΡƒΡ‡Π΅Π½Π½ΠΎΠ΅ Π²Ρ‹Ρ€Π°ΠΆΠ΅Π½ΠΈΠ΅ Π½Π°Π·Ρ‹Π²Π°ΡŽΡ‚ внСшнСй характСристикой источника Ρ‚ΠΎΠΊΠ°.

Π—Π°Π²ΠΈΡΠΈΠΌΠΎΡΡ‚ΡŒ Ρ‚ΠΎΠΊΠ° источника ΠΎΡ‚ напряТСния Π½Π° Π΅Π³ΠΎ Π·Π°ΠΆΠΈΠΌΠ°Ρ… ΠΏΠΎΠΊΠ°Π·Π°Π½ΠΎ Π½Π° рис. 1.11. Π’ случаС идСального источника внутрСнняя ΠΏΡ€ΠΎΠ²ΠΎΠ΄ΠΈΠΌΠΎΡΡ‚ΡŒ Ρ€Π°Π²Π½Π° Π½ΡƒΠ»ΡŽ ΠΈ, исходя ΠΈΠ· уравнСния внСшнСй характСристики, ΠΌΠΎΠΆΠ½ΠΎ Π·Π°ΠΊΠ»ΡŽΡ‡ΠΈΡ‚ΡŒ, Ρ‡Ρ‚ΠΎ Ρ‚ΠΎΠΊ, ΠΈΠ΄ΡƒΡ‰ΠΈΠΉ ΠΎΡ‚ источника Ρ€Π°Π²Π΅Π½ Ρ‚ΠΎΠΊΡƒ ΠΊΠΎΡ€ΠΎΡ‚ΠΊΠΎΠ³ΠΎ замыкания источника. Π­Ρ‚Π° Π·Π°Π²ΠΈΡΠΈΠΌΠΎΡΡ‚ΡŒ ΠΏΠΎΠΊΠ°Π·Π°Π½Π° Π½Π° рис. 1.11 сплошной Π»ΠΈΠ½ΠΈΠ΅ΠΉ.

Π’ случаС Ρ€Π΅Π°Π»ΡŒΠ½ΠΎΠ³ΠΎ источника Β Β g0Β β‰  0 ΠΈ Ρ‡Π°ΡΡ‚ΡŒ Ρ‚ΠΎΠΊΠ° Π±ΡƒΠ΄Π΅Ρ‚ ΠΎΡ‚Π²Π΅Ρ‚Π²Π»ΡΡ‚ΡŒΡΡ Ρ‡Π΅Ρ€Π΅Π· Π²Π½ΡƒΡ‚Ρ€Π΅Π½Π½ΡŽΡŽ ΠΏΡ€ΠΎΠ²ΠΎΠ΄ΠΈΠΌΠΎΡΡ‚ΡŒ. Π§Π΅ΠΌ большС напряТСниС, ΠΏΡ€ΠΈΠ»ΠΎΠΆΠ΅Π½Π½ΠΎΠ΅ ΠΊ источнику, Ρ‚Π΅ΠΌ больший Ρ‚ΠΎΠΊ отвСтвляСтся ΠΈ Ρ‚Π΅ΠΌ мСньший Ρ‚ΠΎΠΊ поступаСт Π² Π½Π°Π³Ρ€ΡƒΠ·ΠΊΡƒ. Π’ΠΎΠ»ΡŒΡ‚-ампСрная характСристика Ρ€Π΅Π°Π»ΡŒΠ½ΠΎΠ³ΠΎ источника ΠΏΠΎΠΊΠ°Π·Π°Π½Π° Π½Π° рис. 1.11 ΠΏΡƒΠ½ΠΊΡ‚ΠΈΡ€Π½ΠΎΠΉ Π»ΠΈΠ½ΠΈΠ΅ΠΉ.Β Π˜ΡΡ‚ΠΎΡ‡Π½ΠΈΠΊ Ρ‚ΠΎΠΊΠ° – это тСорСтичСскоС понятиС, Π½ΠΎ ΠΎΠ½ΠΎ часто примСняСтся для расчСта элСктричСских Ρ†Π΅ΠΏΠ΅ΠΉ. ΠŸΡ€ΠΈΠΌΠ΅Ρ€ΠΎΠΌ источника Ρ‚ΠΎΠΊΠ° ΠΌΠΎΠΆΠ΅Ρ‚ ΡΠ»ΡƒΠΆΠΈΡ‚ΡŒ ΠΏΠ΅Π½Ρ‚ΠΎΠ΄.

Π­ΠΊΠ²ΠΈΠ²Π°Π»Π΅Π½Ρ‚Π½ΠΎΠ΅ Β Β ΠΏΡ€Π΅ΠΎΠ±Ρ€Π°Π·ΠΎΠ²Π°Π½ΠΈΠ΅   источников Β Β ΠΊΠΎΠ½Π΅Ρ‡Π½ΠΎΠΉ    мощности

ΠŸΡ€Π΅ΠΎΠ±Ρ€Π°Π·ΠΎΠ²Π°Π½ΠΈΠ΅ ΠΊΠ°ΠΊΠΎΠ³ΠΎ-Π»ΠΈΠ±ΠΎ участка Ρ†Π΅ΠΏΠΈ ΠΏΠΎ ΠΎΡ‚Π½ΠΎΡˆΠ΅Π½ΠΈΡŽ ΠΊ внСшним Π·Π°ΠΆΠΈΠΌΠ°ΠΌ Π½Π°Π·Ρ‹Π²Π°ΡŽΡ‚ эквивалСнтным, Ссли напряТСниС uΒ ΠΈ Ρ‚ΠΎΠΊΒ iΒ Π½Π° Π²Π½Π΅ΡˆΠ½ΠΈΡ… Π·Π°ΠΆΠΈΠΌΠ°Ρ… ΠΏΡ€ΠΈ этом Π½Π΅ ΠΈΠ·ΠΌΠ΅Π½ΡΡŽΡ‚ΡΡ.

 Рассмотрим условиС эквивалСнтности Ρ€Π΅Π°Π»ΡŒΠ½Ρ‹Ρ… источников напряТСния ΠΈ Ρ‚ΠΎΠΊΠ°, прСдставлСнных Π½Π° рис. 1.12, Π°,Π±. Β Π’ΠΎΡΠΏΠΎΠ»ΡŒΠ·ΡƒΠ΅ΠΌΡΡ ΡƒΡ€Π°Π²Π½Π΅Π½ΠΈΠ΅ΠΌ внСшнСй характСристики источника  Π­Π”Π‘

ПодСлим ΠΏΠΎΡ‡Π»Π΅Π½Π½ΠΎ это ΡƒΡ€Π°Π²Π½Π΅Π½ΠΈΠ΅ Π½Π°Β r0


Π—Π΄Π΅ΡΡŒΒ I – Ρ‚ΠΎΠΊ, ΠΏΡ€ΠΎΡ‚Π΅ΠΊΠ°ΡŽΡ‰ΠΈΠΉ Ρ‡Π΅Ρ€Π΅Π· Π½Π°Π³Ρ€ΡƒΠ·ΠΊΡƒ;

JΠΊΠ· = E/r0 – Ρ‚ΠΎΠΊ ΠΊΠΎΡ€ΠΎΡ‚ΠΊΠΎΠ³ΠΎ замыкания источника  Π­Π”Π‘;Β 

I0 = U/r0  – Ρ‚ΠΎΠΊ, ΠΏΡ€ΠΎΡ‚Π΅ΠΊΠ°ΡŽΡ‰ΠΈΠΉ Ρ‡Π΅Ρ€Π΅Π· Π²Π½ΡƒΡ‚Ρ€Π΅Π½Π½Π΅Π΅ сопротивлСниС.

ΠžΡ‚ΡΡŽΠ΄Π°Β  ΠΌΠΎΠΆΠ½ΠΎ Π·Π°ΠΊΠ»ΡŽΡ‡ΠΈΡ‚ΡŒ, Ρ‡Ρ‚ΠΎΒ  I0 = JΠΊΠ· – IΒ  Β ΠΈΠ»ΠΈΒ I = JΠΊΠ· – I0, Ρ‚ΠΎ Π΅ΡΡ‚ΡŒ ΠΏΠΎΠ»ΡƒΡ‡ΠΈΠ»ΠΈ внСшнюю характСристику источника Ρ‚ΠΎΠΊΠ°.

Π‘Π»Π΅Π΄ΠΎΠ²Π°Ρ‚Π΅Π»ΡŒΠ½ΠΎ, схСму источника Β Π­Π”Π‘ ΠΌΠΎΠΆΠ½ΠΎ Π·Π°ΠΌΠ΅Π½ΠΈΡ‚ΡŒ схСмой источника Ρ‚ΠΎΠΊΠ° ΠΏΡ€ΠΈ условии, Ρ‡Ρ‚ΠΎ Ρ‚ΠΎΠΊ ΠΊΠΎΡ€ΠΎΡ‚ΠΊΠΎΠ³ΠΎ замыкания источника ΠΈ внутрСнняя ΠΏΡ€ΠΎΠ²ΠΎΠ΄ΠΈΠΌΠΎΡΡ‚ΡŒ опрСдСлятся выраТСниями:

Π’ свою ΠΎΡ‡Π΅Ρ€Π΅Π΄ΡŒ, схСму источника Ρ‚ΠΎΠΊΠ° ΠΌΠΎΠΆΠ½ΠΎ Π·Π°ΠΌΠ΅Π½ΠΈΡ‚ΡŒ схСмой источника Β Π­Π”Π‘ ΠΏΡ€ΠΈ условии, Ρ‡Ρ‚ΠΎ Π²Π½ΡƒΡ‚Ρ€Π΅Π½Π½Π΅Π΅ сопротивлСниС ΠΈ э.Π΄.с. источника опрСдСлятся выраТСниями:

ΠœΠΎΡ‰Π½ΠΎΡΡ‚ΡŒ источника Π­Π”Π‘ опрСдСляСтся ΠΏΡ€ΠΎΠΈΠ·Π²Π΅Π΄Π΅Π½ΠΈΠ΅ΠΌ элСктродвиТущСй силы источника ΠΈ Ρ‚ΠΎΠΊΠ° Π² Π½Π°Π³Ρ€ΡƒΠ·ΠΊΠ΅

ΠœΠΎΡ‰Π½ΠΎΡΡ‚ΡŒ источника Ρ‚ΠΎΠΊΠ° опрСдСляСтся ΠΏΡ€ΠΎΠΈΠ·Π²Π΅Π΄Π΅Π½ΠΈΠ΅ΠΌ Ρ‚ΠΎΠΊΠ° ΠΊΠΎΡ€ΠΎΡ‚ΠΊΠΎΠ³ΠΎ замыкания ΠΈ напряТСния Π½Π° Π·Π°ΠΆΠΈΠΌΠ°Ρ… источника:

Π­Π”Π‘. Π—Π°ΠΊΠΎΠ½ Ома для ΠΏΠΎΠ»Π½ΠΎΠΉ Ρ†Π΅ΠΏΠΈ.

Если свободныС заряды ΠΏΠ΅Ρ€Π΅ΠΌΠ΅Ρ‰Π°ΡŽΡ‚ΡΡ Π² элСктричСской Ρ†Π΅ΠΏΠΈ ΠΏΠΎ Π·Π°ΠΌΠΊΠ½ΡƒΡ‚ΠΎΠΉ Ρ‚Ρ€Π°Π΅ΠΊΡ‚ΠΎΡ€ΠΈΠΈ, Ρ‚ΠΎ Ρ‚Π°ΠΊΡƒΡŽ Ρ†Π΅ΠΏΡŒ Π½Π°Π·Ρ‹Π²Π°ΡŽΡ‚ ΠΏΠΎΠ»Π½ΠΎΠΉ ΠΈΠ»ΠΈ Π·Π°ΠΌΠΊΠ½ΡƒΡ‚ΠΎΠΉ.

ΠŸΡ€ΠΈ этом Π½Π° ΠΊΠ°ΠΆΠ΄ΠΎΠΌ ΠΈΠ· участков Ρ‚Π°ΠΊΠΎΠΉ Ρ†Π΅ΠΏΠΈ Ρ€Π°Π±ΠΎΡ‚Π° элСктростатичСских сил ΠΏΠ΅Ρ€Π΅Ρ…ΠΎΠ΄ΠΈΡ‚ Π² Ρ‚Π΅ΠΏΠ»ΠΎΠ²ΡƒΡŽ, ΠΌΠ΅Ρ…Π°Π½ΠΈΡ‡Π΅ΡΠΊΡƒΡŽ ΠΈΠ»ΠΈ ΡΠ½Π΅Ρ€Π³ΠΈΡŽ химичСских связСй. Π’Π°ΠΊ ΠΊΠ°ΠΊ Ρ€Π°Π±ΠΎΡ‚Π° элСктростатичСских сил, ΠΏΠ΅Ρ€Π΅ΠΌΠ΅Ρ‰Π°ΡŽΡ‰ΠΈΡ… заряд ΠΏΠΎ Π·Π°ΠΌΠΊΠ½ΡƒΡ‚ΠΎΠΉ Ρ‚Ρ€Π°Π΅ΠΊΡ‚ΠΎΡ€ΠΈΠΈ, всСгда Ρ€Π°Π²Π½Π° Π½ΡƒΠ»ΡŽ, Ρ‚ΠΎ Ρ‚ΠΎΠ»ΡŒΠΊΠΎ силы элСктростатичСского поля Π½Π΅ ΠΌΠΎΠ³ΡƒΡ‚ ΠΎΠ±Π΅ΡΠΏΠ΅Ρ‡ΠΈΡ‚ΡŒ постоянноС Π΄Π²ΠΈΠΆΠ΅Π½ΠΈΠ΅ зарядов ΠΏΠΎ Π·Π°ΠΌΠΊΠ½ΡƒΡ‚ΠΎΠΉ Ρ‚Ρ€Π°Π΅ΠΊΡ‚ΠΎΡ€ΠΈΠΈ.

Π§Ρ‚ΠΎΠ±Ρ‹ элСктричСский Ρ‚ΠΎΠΊ Π² Π·Π°ΠΌΠΊΠ½ΡƒΡ‚ΠΎΠΉ Ρ†Π΅ΠΏΠΈ Π½Π΅ прСкращался, Π½Π΅ΠΎΠ±Ρ…ΠΎΠ΄ΠΈΠΌΠΎ Π²ΠΊΠ»ΡŽΡ‡ΠΈΡ‚ΡŒ Π² Π½Π΅Ρ‘ источник Ρ‚ΠΎΠΊΠ° (см. рис. Π°), Π²Π½ΡƒΡ‚Ρ€ΠΈ ΠΊΠΎΡ‚ΠΎΡ€ΠΎΠ³ΠΎ ΠΏΠ΅Ρ€Π΅ΠΌΠ΅Ρ‰Π΅Π½ΠΈΠ΅ свободных зарядов происходило Π±Ρ‹ Π½Π΅ ΠΏΠΎΠ΄ дСйствиСм элСктростатичСских сил, Π° ΠΏΡ€ΠΈ участии Π»ΡŽΠ±Ρ‹Ρ… Π΄Ρ€ΡƒΠ³ΠΈΡ… сил, Π½Π°Π·Ρ‹Π²Π°Π΅ΠΌΡ‹Ρ… сторонними. Π‘Ρ‚ΠΎΡ€ΠΎΠ½Π½ΠΈΠ΅ силы – силы нСэлСктростатичСского происхоТдСния, Π΄Π΅ΠΉΡΡ‚Π²ΡƒΡŽΡ‰ΠΈΡ… Π½Π° заряды со стороны источника Ρ‚ΠΎΠΊΠ°. ΠŸΡ€ΠΈΡ€ΠΎΠ΄Π° сторонних сил ΠΌΠΎΠΆΠ΅Ρ‚ Π±Ρ‹Ρ‚ΡŒ Ρ€Π°Π·Π»ΠΈΡ‡Π½ΠΎΠΉ (ΠΊΡ€ΠΎΠΌΠ΅ Π½Π΅ΠΏΠΎΠ΄Π²ΠΈΠΆΠ½Ρ‹Ρ… зарядов):

1) химичСскиС Ρ€Π΅Π°ΠΊΡ†ΠΈΠΈ – Π² Π³Π°Π»ΡŒΠ²Π°Π½ΠΈΡ‡Π΅ΡΠΊΠΈΡ… элСмСнтах (Π±Π°Ρ‚Π°Ρ€Π΅ΠΉΠΊΠ°Ρ…), аккумуляторах (сторонниС силы Π²ΠΎΠ·Π½ΠΈΠΊΠ°ΡŽΡ‚ Π² Ρ€Π΅Π·ΡƒΠ»ΡŒΡ‚Π°Ρ‚Π΅ химичСских Ρ€Π΅Π°ΠΊΡ†ΠΈΠΉ ΠΌΠ΅ΠΆΠ΄Ρƒ элСктродами ΠΈ ΠΆΠΈΠ΄ΠΊΠΈΠΌ элСктролитом),

2) элСктромагнитной – Π² Π³Π΅Π½Π΅Ρ€Π°Ρ‚ΠΎΡ€Π°Ρ…. ΠŸΡ€ΠΈ этом Π³Π΅Π½Π΅Ρ€Π°Ρ‚ΠΎΡ€Ρ‹ ΠΌΠΎΠ³ΡƒΡ‚ ΠΈΡΠΏΠΎΠ»ΡŒΠ·ΠΎΠ²Π°Ρ‚ΡŒ Π°) ΠΌΠ΅Ρ…Π°Π½ΠΈΡ‡Π΅ΡΠΊΡƒΡŽ ΡΠ½Π΅Ρ€Π³ΠΈΡŽ – Π“Π­Π‘, Π±) ΡΠ΄Π΅Ρ€Π½ΡƒΡŽ – АЭБ, Π²) Ρ‚Π΅ΠΏΠ»ΠΎΠ²ΡƒΡŽ – Π’Π­Π‘, Π³) ΠΏΡ€ΠΈΠ»ΠΈΠ²ΠΎΠ² ΠΈ ΠΎΡ‚Π»ΠΈΠ²ΠΎΠ² – ПЭБ, Π΄) Π²Π΅Ρ‚Ρ€ΠΎΠ²ΡƒΡŽ – Π’Π­Π‘ ΠΈ Ρ‚.Π΄. (силы, Π΄Π΅ΠΉΡΡ‚Π²ΡƒΡŽΡ‰ΠΈΠ΅ Π½Π° свободныС заряды, ΠΏΠ΅Ρ€Π΅ΠΌΠ΅Ρ‰Π°ΡŽΡ‰ΠΈΠ΅ΡΡ Π² ΠΌΠ°Π³Π½ΠΈΡ‚Π½ΠΎΠΌ ΠΏΠΎΠ»Π΅).

3) использованиС фотоэффСкта – Ρ„ΠΎΡ‚ΠΎ-Π­Π”Π‘ Π² ΠΊΠ°Π»ΡŒΠΊΡƒΠ»ΡΡ‚ΠΎΡ€Π°Ρ… ΠΈ солнСчных батарСях (Π² фотоэлСмСнтах сторонниС силы Π²ΠΎΠ·Π½ΠΈΠΊΠ°ΡŽΡ‚ ΠΏΡ€ΠΈ дСйствии свСта Π½Π° элСктроны Π°Ρ‚ΠΎΠΌΠΎΠ², входящих Π² состав Π½Π΅ΠΊΠΎΡ‚ΠΎΡ€Ρ‹Ρ… вСщСств),

4) ΠΏΡŒΠ΅Π·ΠΎΡΡ„Ρ„Π΅ΠΊΡ‚ – пьСзо-Π­Π”Π‘, Π½Π°ΠΏΡ€ΠΈΠΌΠ΅Ρ€, Π² ΠΏΡŒΠ΅Π·ΠΎΠ·Π°ΠΆΠΈΠ³Π°Π»ΠΊΠ°Ρ…,

5) контактная Ρ€Π°Π·Π½ΠΎΡΡ‚ΡŒ ΠΏΠΎΡ‚Π΅Π½Ρ†ΠΈΠ°Π»ΠΎΠ² – Ρ‚Π΅Ρ€ΠΌΠΎ-Π­Π”Π‘ Π² Ρ‚Π΅Ρ€ΠΌΠΎΠΏΠ°Ρ€Π°Ρ… ΠΈ Ρ‚.Π΄.

НапримСр, Π² Ρ†Π΅ΠΏΠΈ Π½Π° рис. Π°, свободныС заряды, ΠΏΠ΅Ρ€Π΅ΠΌΠ΅Ρ‰Π°ΡŽΡ‚ΡΡ ΠΎΡ‚ Ρ‚Π΅Π»Π° А ΠΊ Ρ‚Π΅Π»Ρƒ Π‘ ΠΏΠΎΠ΄ дСйствиСм элСктростатичСских сил, Π° сторонниС силы источника питания Π·Π°ΡΡ‚Π°Π²Π»ΡΡŽΡ‚ ΠΈΡ… Π²ΠΎΠ·Π²Ρ€Π°Ρ‰Π°Ρ‚ΡŒΡΡ ΠΎΠ±Ρ€Π°Ρ‚Π½ΠΎ – ΠΎΡ‚ Π‘ ΠΊ А.

Π‘Ρ‚ΠΎΡ€ΠΎΠ½Π½ΠΈΠ΅ силы Π² источникС Ρ‚ΠΎΠΊΠ° Ρ€Π°Π·Π΄Π΅Π»ΡΡŽΡ‚ Ρ€Π°Π·Π½ΠΎΠΈΠΌΡ‘Π½Π½Ρ‹Π΅ элСктричСскиС заряды Π΄Ρ€ΡƒΠ³ ΠΎΡ‚ Π΄Ρ€ΡƒΠ³Π°, ΡΠΎΠ²Π΅Ρ€ΡˆΠ°Ρ Ρ€Π°Π±ΠΎΡ‚Ρƒ ΠΏΡ€ΠΎΡ‚ΠΈΠ² элСктростатичСских (кулоновских сил). ΠšΠΎΠ½Ρ‚Π°ΠΊΡ‚ (полюс) источника Ρ‚ΠΎΠΊΠ°, Π³Π΄Π΅ Π² Ρ€Π΅Π·ΡƒΠ»ΡŒΡ‚Π°Ρ‚Π΅ дСйствия сторонних сил накапливаСтся ΠΏΠΎΠ»ΠΎΠΆΠΈΡ‚Π΅Π»ΡŒΠ½Ρ‹ΠΉ заряд, Π½Π°Π·Ρ‹Π²Π°ΡŽΡ‚ ΠΏΠΎΠ»ΠΎΠΆΠΈΡ‚Π΅Π»ΡŒΠ½Ρ‹ΠΌ, Π° ΠΏΡ€ΠΎΡ‚ΠΈΠ²ΠΎΠΏΠΎΠ»ΠΎΠΆΠ½ΠΎ заряТСнный полюс – ΠΎΡ‚Ρ€ΠΈΡ†Π°Ρ‚Π΅Π»ΡŒΠ½Ρ‹ΠΌ, обозначая ΠΈΡ… Ρ‚Π°ΠΊ, ΠΊΠ°ΠΊ ΠΈΠ·ΠΎΠ±Ρ€Π°ΠΆΠ΅Π½ΠΎ Π½Π° рис. Π±. ΠžΡ‡Π΅Π²ΠΈΠ΄Π½ΠΎ, Ρ‡Ρ‚ΠΎ Ρ‡Π΅ΠΌ больший заряд накопится Π½Π° полюсС источника Ρ‚ΠΎΠΊΠ°, Ρ‚Π΅ΠΌ большС Ρ€Π°Π±ΠΎΡ‚Ρ‹ ΡΠΎΠ²Π΅Ρ€ΡˆΠΈΠ»ΠΈ сторонниС силы ΠΏΠΎ Ρ€Π°Π·Π΄Π΅Π»Π΅Π½ΠΈΡŽ зарядов, Ρ‚.ΠΊ. Ρ€Π°Π±ΠΎΡ‚Π° ΠΏΡ€ΠΎΡ‚ΠΈΠ² кулоновских сил прямо ΠΏΡ€ΠΎΠΏΠΎΡ€Ρ†ΠΈΠΎΠ½Π°Π»ΡŒΠ½Π° Π²Π΅Π»ΠΈΡ‡ΠΈΠ½Π΅ заряда. ΠŸΠΎΡΡ‚ΠΎΠΌΡƒΒ  ΠΎΡ‚Π½ΠΎΡˆΠ΅Π½ΠΈΠ΅ Ρ€Π°Π±ΠΎΡ‚Ρ‹, Аст, сторонних сил, ΠΏΠ΅Ρ€Π΅ΠΌΠ΅Ρ‰Π°ΡŽΡ‰ΠΈΡ… заряд q Π²Π½ΡƒΡ‚Ρ€ΠΈ источника Ρ‚ΠΎΠΊΠ° ΠΎΡ‚ ΠΎΡ‚Ρ€ΠΈΡ†Π°Ρ‚Π΅Π»ΡŒΠ½ΠΎΠ³ΠΎ полюса ΠΊ ΠΏΠΎΠ»ΠΎΠΆΠΈΡ‚Π΅Π»ΡŒΠ½ΠΎΠΌΡƒ, Π½Π΅ зависит ΠΎΡ‚ Π²Π΅Π»ΠΈΡ‡ΠΈΠ½Ρ‹ заряда ΠΈ слуТит характСристикой источника Ρ‚ΠΎΠΊΠ°, Π½Π°Π·Ρ‹Π²Π°Π΅ΠΌΠΎΠΉ элСктродвиТущСй силой (Π­Π”Π‘) источника,

.

Β 

Как ΠΈ Ρ€Π°Π·Π½ΠΎΡΡ‚ΡŒ ΠΏΠΎΡ‚Π΅Π½Ρ†ΠΈΠ°Π»ΠΎΠ², Π­Π”Π‘ Π² БИ ΠΈΠ·ΠΌΠ΅Ρ€ΡΡŽΡ‚ Π² Π²ΠΎΠ»ΡŒΡ‚Π°Ρ….

Π‘ΠΎΠΏΡ€ΠΎΡ‚ΠΈΠ²Π»Π΅Π½ΠΈΠ΅ источника Ρ‚ΠΎΠΊΠ° ΠΈΠ»ΠΈ Π²Π½ΡƒΡ‚Ρ€Π΅Π½Π½Π΅Π΅ сопротивлСниС Ρ‚ΠΎΠΆΠ΅ являСтся Π΅Π³ΠΎ Π²Π°ΠΆΠ½ΠΎΠΉ характСристикой. Π’Π½ΡƒΡ‚Ρ€Π΅Π½Π½ΠΈΠΌ сопротивлСниСм Π³Π°Π»ΡŒΠ²Π°Π½ΠΈΡ‡Π΅ΡΠΊΠΎΠ³ΠΎ элСмСнта, Π½Π°ΠΏΡ€ΠΈΠΌΠ΅Ρ€, являСтся сопротивлСниС элСктродов ΠΈ элСктролита, находящСгося ΠΌΠ΅ΠΆΠ΄Ρƒ Π½ΠΈΠΌΠΈ. Π’Π½Π΅ΡˆΠ½ΠΈΠΌ участком Π·Π°ΠΌΠΊΠ½ΡƒΡ‚ΠΎΠΉ Ρ†Π΅ΠΏΠΈ Π½Π°Π·Ρ‹Π²Π°ΡŽΡ‚ Π΅Ρ‘ участок, подсоСдинённый снаруТи ΠΊ источнику Ρ‚ΠΎΠΊΠ° (см. рис. Π°).

Π§Ρ‚ΠΎΠ±Ρ‹ ΠΎΠΏΡ€Π΅Π΄Π΅Π»ΠΈΡ‚ΡŒ, ΠΊΠ°ΠΊ зависит сила Ρ‚ΠΎΠΊΠ° ΠΎΡ‚ Π­Π”Π‘ источника Π² Ρ†Π΅ΠΏΠΈ, ΠΈΠ·ΠΎΠ±Ρ€Π°ΠΆΡ‘Π½Π½ΠΎΠΉ Π½Π° рис. Π°, нарисуСм ΡΠΊΠ²ΠΈΠ²Π°Π»Π΅Π½Ρ‚Π½ΡƒΡŽ схСму (см. рис. Π²), Π³Π΄Π΅ R соотвСтствуСт ΡΠΎΠΏΡ€ΠΎΡ‚ΠΈΠ²Π»Π΅Π½ΠΈΡŽ ΠΏΡ€ΠΎΠ²ΠΎΠ΄Π½ΠΈΠΊΠ° ΠΌΠ΅ΠΆΠ΄Ρƒ А ΠΈ Π‘, (внСшняя Ρ†Π΅ΠΏΡŒ), Π° r – Π²Π½ΡƒΡ‚Ρ€Π΅Π½Π½Π΅ΠΌΡƒ ΡΠΎΠΏΡ€ΠΎΡ‚ΠΈΠ²Π»Π΅Π½ΠΈΡŽ источника Ρ‚ΠΎΠΊΠ°. Богласно Π·Π°ΠΊΠΎΠ½Ρƒ ДТоуля-Π›Π΅Π½Ρ†Π° Ρ€Π°Π±ΠΎΡ‚Π°Β  Аполн Ρ‚ΠΎΠΊΠ°, ΠΏΡ€ΠΎΡ‚Π΅ΠΊΠ°ΡŽΡ‰Π΅Π³ΠΎ ΠΏΠΎ Π·Π°ΠΌΠΊΠ½ΡƒΡ‚ΠΎΠΉ Ρ†Π΅ΠΏΠΈ, Π·Π° ΠΈΠ½Ρ‚Π΅Ρ€Π²Π°Π» Π²Ρ€Π΅ΠΌΠ΅Π½ΠΈ t Ρ€Π°Π²Π½Π°: Аполн = I2.R.t + I2.r.t .Β  Из Π·Π°ΠΊΠΎΠ½Π° сохранСния энСргии слСдуСт, Ρ‡Ρ‚ΠΎ Ρ€Π°Π±ΠΎΡ‚Π° Ρ‚ΠΎΠΊΠ° Π΄ΠΎΠ»ΠΆΠ½Π° Π±Ρ‹Ρ‚ΡŒ Ρ€Π°Π²Π½Π° Ρ€Π°Π±ΠΎΡ‚Π΅ сторонних сил Астор = Ɛ.q = Ɛ.It . ΠŸΡ€ΠΈΡ€Π°Π²Π½ΡΠ² Аполн ΠΈ Астор, ΠΏΠΎΠ»ΡƒΡ‡Π°Π΅ΠΌ ΡΠ»Π΅Π΄ΡƒΡŽΡ‰Π΅Π΅ Π²Ρ‹Ρ€Π°ΠΆΠ΅Π½ΠΈΠ΅ для 

ΠΊΠΎΡ‚ΠΎΡ€ΠΎΠ΅ Π½Π°Π·Ρ‹Π²Π°ΡŽΡ‚ Π·Π°ΠΊΠΎΠ½ΠΎΠΌ Ома для ΠΏΠΎΠ»Π½ΠΎΠΉ Ρ†Π΅ΠΏΠΈ.

1) НапряТСниС Π½Π° Π·Π°ΠΆΠΈΠΌΠ°Ρ… источника, Π° соотвСтствСнно ΠΈ Π²ΠΎ внСшнСй Ρ†Π΅ΠΏΠΈ

Π³Π΄Π΅ Π²Π΅Π»ΠΈΡ‡ΠΈΠ½Π° IrΠΏΠ°Π΄Π΅Π½ΠΈΠ΅ напряТСния Π²Π½ΡƒΡ‚Ρ€ΠΈ источника Ρ‚ΠΎΠΊΠ°.

2) Если внСшнСС сопротивлСниС Π·Π°ΠΌΠΊΠ½ΡƒΡ‚ΠΎΠΉ Ρ†Π΅ΠΏΠΈ Ρ€Π°Π²Π½ΠΎ Π½ΡƒΠ»ΡŽ, Ρ‚ΠΎ Ρ‚Π°ΠΊΠΎΠΉ Ρ€Π΅ΠΆΠΈΠΌ источника Ρ‚ΠΎΠΊΠ° называСтся ΠΊΠΎΡ€ΠΎΡ‚ΠΊΠΈΠΌ Π·Π°ΠΌΡ‹ΠΊΠ°Π½ΠΈΠ΅ΠΌ.

3) Для ΠΏΠΎΠ»Π½ΠΎΠΉ Ρ†Π΅ΠΏΠΈ Π·Π°ΠΊΠΎΠ½ ДТоуля-Π›Π΅Π½Ρ†Π°

Π›Π΅Π³ΠΊΠΎ ΠΏΠΎΠΊΠ°Π·Π°Ρ‚ΡŒ, Ρ‡Ρ‚ΠΎ, Ссли полная Ρ†Π΅ΠΏΡŒ содСрТит нСсколько ΠΏΠΎΡΠ»Π΅Π΄ΠΎΠ²Π°Ρ‚Π΅Π»ΡŒΠ½ΠΎ соСдинённых источников Ρ‚ΠΎΠΊΠ°, Ρ‚ΠΎ для вычислСния силы Ρ‚ΠΎΠΊΠ° слСдуСт вмСсто Ɛ Π²Π·ΡΡ‚ΡŒ Π°Π»Π³Π΅Π±Ρ€Π°ΠΈΡ‡Π΅ΡΠΊΡƒΡŽ сумму Π­Π”Π‘ всСх этих источников, Π²Ρ‹Π±Ρ€Π°Π² ΠΊΠ°ΠΊΠΎΠ΅-Π½ΠΈΠ±ΡƒΠ΄ΡŒ Π½Π°ΠΏΡ€Π°Π²Π»Π΅Π½ΠΈΠ΅ ΠΎΠ±Ρ…ΠΎΠ΄Π° Ρ†Π΅ΠΏΠΈ, Π½Π°ΠΏΡ€ΠΈΠΌΠ΅Ρ€, ΠΏΠΎ часовой стрСлкС (рис. Π³). Если ΠΏΡ€ΠΈ Ρ‚Π°ΠΊΠΎΠΌ ΠΎΠ±Ρ…ΠΎΠ΄Π΅ ΠΌΡ‹ ΠΈΠ΄Ρ‘ΠΌ ΠΎΡ‚ ΠΏΠΎΠ»ΠΎΠΆΠΈΡ‚Π΅Π»ΡŒΠ½ΠΎΠ³ΠΎ полюса источника Ρ‚ΠΎΠΊΠ° ΠΊ ΠΎΡ‚Ρ€ΠΈΡ†Π°Ρ‚Π΅Π»ΡŒΠ½ΠΎΠΌΡƒ, Ρ‚ΠΎ Π­Π”Π‘ Π΄Π°Π½Π½ΠΎΠ³ΠΎ источника слСдуСт ΡΡƒΠΌΠΌΠΈΡ€ΠΎΠ²Π°Ρ‚ΡŒ со Π·Π½Π°ΠΊΠΎΠΌ минус. Β 

Β  Π‘ΠΎΠ»Π΅Π΅ ΠΏΠΎΠ΄Ρ€ΠΎΠ±Π½ΡƒΡŽ ΠΈΠ½Ρ„ΠΎΡ€ΠΌΠ°Ρ†ΠΈΡŽ смотри Π—Π”Π•Π‘Π¬.

элСктродвиТущая сила – это… Π§Ρ‚ΠΎ Ρ‚Π°ΠΊΠΎΠ΅ элСктродвиТущая сила?

(эдс), Π²Π΅Π»ΠΈΡ‡ΠΈΠ½Π°, Ρ…Π°Ρ€Π°ΠΊΡ‚Π΅Ρ€ΠΈΠ·ΡƒΡŽΡ‰Π°Ρ источник энСргии нСэлСктростатичСской ΠΏΡ€ΠΈΡ€ΠΎΠ΄Ρ‹ Π² элСктричСской Ρ†Π΅ΠΏΠΈ, Π½Π΅ΠΎΠ±Ρ…ΠΎΠ΄ΠΈΠΌΡ‹ΠΉ для поддСрТания Π² Π½Π΅ΠΉ элСктричСского Ρ‚ΠΎΠΊΠ°. Эдс числСнно Ρ€Π°Π²Π½Π° Ρ€Π°Π±ΠΎΡ‚Π΅ ΠΏΠΎ ΠΏΠ΅Ρ€Π΅ΠΌΠ΅Ρ‰Π΅Π½ΠΈΡŽ Π΅Π΄ΠΈΠ½ΠΈΡ‡Π½ΠΎΠ³ΠΎ ΠΏΠΎΠ»ΠΎΠΆΠΈΡ‚Π΅Π»ΡŒΠ½ΠΎΠ³ΠΎ заряда вдоль Π·Π°ΠΌΠΊΠ½ΡƒΡ‚ΠΎΠΉ Ρ†Π΅ΠΏΠΈ. Полная эдс Π² Ρ†Π΅ΠΏΠΈ постоянного Ρ‚ΠΎΠΊΠ° Ρ€Π°Π²Π½Π° разности ΠΏΠΎΡ‚Π΅Π½Ρ†ΠΈΠ°Π»ΠΎΠ² Π½Π° ΠΊΠΎΠ½Ρ†Π°Ρ… Ρ€Π°Π·ΠΎΠΌΠΊΠ½ΡƒΡ‚ΠΎΠΉ Ρ†Π΅ΠΏΠΈ. Эдс ΠΈΠ½Π΄ΡƒΠΊΡ†ΠΈΠΈ создаётся Π²ΠΈΡ…Ρ€Π΅Π²Ρ‹ΠΌ элСктричСским ΠΏΠΎΠ»Π΅ΠΌ, ΠΏΠΎΡ€ΠΎΠΆΠ΄Π°Π΅ΠΌΡ‹ΠΌ ΠΏΠ΅Ρ€Π΅ΠΌΠ΅Π½Π½Ρ‹ΠΌ ΠΌΠ°Π³Π½ΠΈΡ‚Π½Ρ‹ΠΌ ΠΏΠΎΠ»Π΅ΠΌ. Π’Β Π‘Π˜ измСряСтся Π² Π²ΠΎΠ»ΡŒΡ‚Π°Ρ….

Π­Π›Π•ΠšΠ’Π ΠžΠ”Π’Π˜ΜΠ–Π£Π©ΠΠ― Π‘Π˜ΜΠ›Π (эдс; e) β€” Π²Π΅Π»ΠΈΡ‡ΠΈΠ½Π°, Ρ…Π°Ρ€Π°ΠΊΡ‚Π΅Ρ€ΠΈΠ·ΡƒΡŽΡ‰Π°Ρ источник энСргии нСэлСктростатичСской ΠΏΡ€ΠΈΡ€ΠΎΠ΄Ρ‹ Π² элСктричСской Ρ†Π΅ΠΏΠΈ, Π½Π΅ΠΎΠ±Ρ…ΠΎΠ΄ΠΈΠΌΡ‹ΠΉ для поддСрТания Π² Π½Π΅ΠΉ элСктричСского Ρ‚ΠΎΠΊΠ° (см. Π­Π›Π•ΠšΠ’Π Π˜Π§Π•Π‘ΠšΠ˜Π™ ВОК). ΠŸΠΎΡ‚Π΅Π½Ρ†ΠΈΠ°Π»ΡŒΠ½Ρ‹Π΅ силы элСктростатичСского (ΠΈΠ»ΠΈ стационарного) поля Π½Π΅ ΠΌΠΎΠ³ΡƒΡ‚ ΠΏΠΎΠ΄Π΄Π΅Ρ€ΠΆΠΈΠ²Π°Ρ‚ΡŒ постоянный Ρ‚ΠΎΠΊ Π² Ρ†Π΅ΠΏΠΈ. Для поддСрТания Π² Ρ†Π΅ΠΏΠΈ Π½Π΅ΠΏΡ€Π΅Ρ€Ρ‹Π²Π½ΠΎΠ³ΠΎ Ρ‚ΠΎΠΊΠ° Π½Π΅ΠΎΠ±Ρ…ΠΎΠ΄ΠΈΠΌ источник Ρ‚ΠΎΠΊΠ° (см. ИБВОЧНИКИ ВОКА), ΠΈΠ»ΠΈ Π³Π΅Π½Π΅Ρ€Π°Ρ‚ΠΎΡ€ (см. Π“Π•ΠΠ•Π ΠΠ’ΠžΠ ) элСктричСского Ρ‚ΠΎΠΊΠ°, ΠΎΠ±Π΅ΡΠΏΠ΅Ρ‡ΠΈΠ²Π°ΡŽΡ‰ΠΈΠΉ дСйствиС сторонних сил (см. Π‘Π’ΠžΠ ΠžΠΠΠ˜Π• Π‘Π˜Π›Π«). Π‘Ρ‚ΠΎΡ€ΠΎΠ½Π½ΠΈΠ΅ силы ΠΈΠΌΠ΅ΡŽΡ‚ нСэлСктростатичСскоС происхоТдСниС ΠΈ Π΄Π΅ΠΉΡΡ‚Π²ΡƒΡŽΡ‚ Π²Π½ΡƒΡ‚Ρ€ΠΈ источников Ρ‚ΠΎΠΊΠ°, (Π³Π΅Π½Π΅Ρ€Π°Ρ‚ΠΎΡ€ΠΎΠ², Π³Π°Π»ΡŒΠ²Π°Π½ΠΈΡ‡Π΅ΡΠΊΠΈΡ… элСмСнтов, аккумуляторов ΠΈ Ρ‚. Π΄.), создавая Ρ€Π°Π·Π½ΠΎΡΡ‚ΡŒ ΠΏΠΎΡ‚Π΅Π½Ρ†ΠΈΠ°Π»ΠΎΠ² ΠΌΠ΅ΠΆΠ΄Ρƒ ΠΊΠΎΠ½Ρ†Π°ΠΌΠΈ ΠΎΡΡ‚Π°Π»ΡŒΠ½ΠΎΠΉ части Ρ†Π΅ΠΏΠΈ ΠΈ приводя Π² Π΄Π²ΠΈΠΆΠ΅Π½ΠΈΠ΅ заряТСнныС частицы Π²Π½ΡƒΡ‚Ρ€ΠΈ источников Ρ‚ΠΎΠΊΠ°.
Π’Π°ΠΊ ΠΊΠ°ΠΊ ΠΏΡ€ΠΈ ΠΏΠ΅Ρ€Π΅ΠΌΠ΅Ρ‰Π΅Π½ΠΈΠΈ элСктричСского заряда ΠΏΠΎ Π·Π°ΠΌΠΊΠ½ΡƒΡ‚ΠΎΠΉ Ρ†Π΅ΠΏΠΈ Ρ€Π°Π±ΠΎΡ‚Π°, ΡΠΎΠ²Π΅Ρ€ΡˆΠ°Π΅ΠΌΠ°Ρ элСктростатичСскими силами, Ρ€Π°Π²Π½Π° Π½ΡƒΠ»ΡŽ, Ρ‚ΠΎ заряд пСрСмСщаСтся лишь ΠΏΠΎΠ΄ дСйствиСм сторонних сил. ΠŸΠΎΡΡ‚ΠΎΠΌΡƒ элСктродвиТущая сила источника Ρ‚ΠΎΠΊΠ° Π±ΡƒΠ΄Π΅Ρ‚ числСнно Ρ€Π°Π²Π½Π° Ρ€Π°Π±ΠΎΡ‚Π΅ сторонних сил А Π² источниках постоянного ΠΈΠ»ΠΈ ΠΏΠ΅Ρ€Π΅ΠΌΠ΅Π½Π½ΠΎΠ³ΠΎ Ρ‚ΠΎΠΊΠ° ΠΏΠΎ ΠΏΠ΅Ρ€Π΅ΠΌΠ΅Ρ‰Π΅Π½ΠΈΡŽ Π΅Π΄ΠΈΠ½ΠΈΡ‡Π½ΠΎΠ³ΠΎ ΠΏΠΎΠ»ΠΎΠΆΠΈΡ‚Π΅Π»ΡŒΠ½ΠΎΠ³ΠΎ заряда Q вдоль Π·Π°ΠΌΠΊΠ½ΡƒΡ‚ΠΎΠΉ Ρ†Π΅ΠΏΠΈ. Π­Π”Π‘, Π΄Π΅ΠΉΡΡ‚Π²ΡƒΡŽΡ‰Π°Ρ Π² Ρ†Π΅ΠΏΠΈ, опрСдСляСтся ΠΊΠ°ΠΊ циркуляция Π²Π΅ΠΊΡ‚ΠΎΡ€Π° напряТСнности сторонних сил.
ΠŸΡ€ΠΎΠΈΡΡ…ΠΎΠΆΠ΄Π΅Π½ΠΈΠ΅ сторонних сил ΠΌΠΎΠΆΠ΅Ρ‚ Π±Ρ‹Ρ‚ΡŒ Ρ€Π°Π·Π»ΠΈΡ‡Π½Ρ‹ΠΌ. Π’ качСствС ΠΌΠ΅Ρ€Ρ‹ элСктродвиТущСй силы, Π΄Π΅ΠΉΡΡ‚Π²ΡƒΡŽΡ‰Π΅ΠΉ Π² Π³Π΅Π½Π΅Ρ€Π°Ρ‚ΠΎΡ€Π΅, ΠΏΡ€ΠΈΠ½ΠΈΠΌΠ°ΡŽΡ‚ Ρ€Π°Π·Π½ΠΎΡΡ‚ΡŒ ΠΏΠΎΡ‚Π΅Π½Ρ†ΠΈΠ°Π»ΠΎΠ², ΡΠΎΠ·Π΄Π°Π²Π°Π΅ΠΌΡƒΡŽ Π½Π° Π·Π°ΠΆΠΈΠΌΠ°Ρ… Ρ€Π°Π·ΠΎΠΌΠΊΠ½ΡƒΡ‚ΠΎΠ³ΠΎ Π³Π΅Π½Π΅Ρ€Π°Ρ‚ΠΎΡ€Π°. Один ΠΈ Ρ‚ΠΎΡ‚ ΠΆΠ΅ источник Ρ‚ΠΎΠΊΠ°, Π² зависимости ΠΎΡ‚ силы ΠΎΡ‚Π±ΠΈΡ€Π°Π΅ΠΌΠΎΠ³ΠΎ Ρ‚ΠΎΠΊΠ°, ΠΌΠΎΠΆΠ΅Ρ‚ ΠΎΠ±Π»Π°Π΄Π°Ρ‚ΡŒ Ρ€Π°Π·Π»ΠΈΡ‡Π½Ρ‹ΠΌ напряТСниСм Π½Π° элСктродах. Π˜ΡΡ‚ΠΎΡ‡Π½ΠΈΠΊΠΈ Ρ‚ΠΎΠΊΠ° β€” аккумуляторы, тСрмоэлСмСнты, элСктричСскиС Π³Π΅Π½Π΅Ρ€Π°Ρ‚ΠΎΡ€Ρ‹ – ΠΎΠ΄Π½ΠΎΠ²Ρ€Π΅ΠΌΠ΅Π½Π½ΠΎ Π·Π°ΠΌΡ‹ΠΊΠ°ΡŽΡ‚ ΡΠ»Π΅ΠΊΡ‚Ρ€ΠΈΡ‡Π΅ΡΠΊΡƒΡŽ Ρ†Π΅ΠΏΡŒ. Π’ΠΎΠΊ Ρ‚Π΅Ρ‡Π΅Ρ‚ ΠΏΠΎ внСшнСй части Ρ†Π΅ΠΏΠΈ β€” ΠΏΡ€ΠΎΠ²ΠΎΠ΄Π½ΠΈΠΊΡƒ ΠΈ ΠΏΠΎ Π²Π½ΡƒΡ‚Ρ€Π΅Π½Π½Π΅ΠΉ β€” источнику Ρ‚ΠΎΠΊΠ°. Π˜ΡΡ‚ΠΎΡ‡Π½ΠΈΠΊ Ρ‚ΠΎΠΊΠ° ΠΈΠΌΠ΅Π΅Ρ‚ Π΄Π²Π° полюса: ΠΏΠΎΠ»ΠΎΠΆΠΈΡ‚Π΅Π»ΡŒΠ½Ρ‹ΠΉ (с Π±ΠΎΠ»Π΅Π΅ высоким ΠΏΠΎΡ‚Π΅Π½Ρ†ΠΈΠ°Π»ΠΎΠΌ) ΠΈ ΠΎΡ‚Ρ€ΠΈΡ†Π°Ρ‚Π΅Π»ΡŒΠ½Ρ‹ΠΉ (с Π±ΠΎΠ»Π΅Π΅ Π½ΠΈΠ·ΠΊΠΈΠΌ ΠΏΠΎΡ‚Π΅Π½Ρ†ΠΈΠ°Π»ΠΎΠΌ). Π‘Ρ‚ΠΎΡ€ΠΎΠ½Π½ΠΈΠ΅ силы, ΠΏΡ€ΠΈΡ€ΠΎΠ΄Π° ΠΊΠΎΡ‚ΠΎΡ€Ρ‹Ρ… ΠΌΠΎΠΆΠ΅Ρ‚ Π±Ρ‹Ρ‚ΡŒ Ρ€Π°Π·Π»ΠΈΡ‡Π½ΠΎΠΉ (химичСской, мСханичСской, Ρ‚Π΅ΠΏΠ»ΠΎΠ²ΠΎΠΉ), Ρ€Π°Π·Π΄Π΅Π»ΡΡŽΡ‚ заряды Π² источникС Ρ‚ΠΎΠΊΠ°. Полная Π­Π”Π‘ Π² Ρ†Π΅ΠΏΠΈ постоянного Ρ‚ΠΎΠΊΠ° (максимальноС ΠΈΠ· этих напряТСний, ΡΡƒΡ‰Π΅ΡΡ‚Π²ΡƒΡŽΡ‰Π΅Π΅ ΠΏΡ€ΠΈ Ρ€Π°Π·ΠΎΠΌΠΊΠ½ΡƒΡ‚ΠΎΠΉ Ρ†Π΅ΠΏΠΈ), Ρ€Π°Π²Π½Π° разности ΠΏΠΎΡ‚Π΅Π½Ρ†ΠΈΠ°Π»ΠΎΠ² Π½Π° ΠΊΠΎΠ½Ρ†Π°Ρ… Ρ€Π°Π·ΠΎΠΌΠΊΠ½ΡƒΡ‚ΠΎΠΉ Ρ†Π΅ΠΏΠΈ ΠΈ ΠΏΠΎΠΊΠ°Π·Ρ‹Π²Π°Π΅Ρ‚ Π­Π”Π‘ источника.
Π­Π”Π‘ опрСдСляСт силу Ρ‚ΠΎΠΊΠ° Π² Ρ†Π΅ΠΏΠΈ ΠΏΡ€ΠΈ Π·Π°Π΄Π°Π½Π½ΠΎΠΌ Π΅Π΅ сопротивлСнии (Ома Π·Π°ΠΊΠΎΠ½ (см. ОМА Π—ΠΠšΠžΠ)). Π˜Π·ΠΌΠ΅Ρ€ΡΠ΅Ρ‚ΡΡ Π­Π”Π‘, ΠΊΠ°ΠΊ ΠΈ напряТСниС, Π² Π²ΠΎΠ»ΡŒΡ‚Π°Ρ… (см. Π’ΠžΠ›Π¬Π’). Для поддСрТания Π½Π΅ΠΏΡ€Π΅Ρ€Ρ‹Π²Π½ΠΎΠ³ΠΎ элСктричСского Ρ‚ΠΎΠΊΠ° ΠΈΡΠΏΠΎΠ»ΡŒΠ·ΡƒΡŽΡ‚ΡΡ Π³Π΅Π½Π΅Ρ€Π°Ρ‚ΠΎΡ€Ρ‹, ΡΠ²Π»ΡΡŽΡ‰ΠΈΠ΅ΡΡ источником элСктродвиТущСй силы. Π’ Π³Π΅Π½Π΅Ρ€Π°Ρ‚ΠΎΡ€Π°Ρ… сторонниС силы β€” это силы со стороны Π²ΠΈΡ…Ρ€Π΅Π²ΠΎΠ³ΠΎ элСктричСского поля, Π²ΠΎΠ·Π½ΠΈΠΊΠ°ΡŽΡ‰Π΅Π³ΠΎ ΠΏΡ€ΠΈ ΠΈΠ·ΠΌΠ΅Π½Π΅Π½ΠΈΠΈ ΠΌΠ°Π³Π½ΠΈΡ‚Π½ΠΎΠ³ΠΎ поля со Π²Ρ€Π΅ΠΌΠ΅Π½Π΅ΠΌ, ΠΈΠ»ΠΈ Π›ΠΎΡ€Π΅Π½Ρ†Π° сила (см. Π›ΠžΠ Π•ΠΠ¦Π Π‘Π˜Π›Π), Π΄Π΅ΠΉΡΡ‚Π²ΡƒΡŽΡ‰Π°Ρ со стороны ΠΌΠ°Π³Π½ΠΈΡ‚Π½ΠΎΠ³ΠΎ поля Π½Π° элСктроны Π² двиТущСмся ΠΏΡ€ΠΎΠ²ΠΎΠ΄Π½ΠΈΠΊΠ΅; Π² Π³Π°Π»ΡŒΠ²Π°Π½ΠΈΡ‡Π΅ΡΠΊΠΈΡ… элСмСнтах (см. Π“ΠΠ›Π¬Π’ΠΠΠ˜Π§Π•Π‘ΠšΠ˜Π™ Π­Π›Π•ΠœΠ•ΠΠ’) ΠΈ аккумуляторах β€” это химичСскиС силы.

ΠŸΠΎΡΡ‚ΠΎΡΠ½Π½Ρ‹ΠΉ Ρ‚ΠΎΠΊ – Π­Π”Π‘ – элСктродвиТущая сила

Β 

Π­Π”Π‘ – элСктродвиТущая сила

Β 

Β 

ΠžΠΏΡ€Π΅Π΄Π΅Π»Π΅Π½ΠΈΠ΅ 1. Π‘Ρ‚ΠΎΡ€ΠΎΠ½Π½ΠΈΠ΅ силы – это Π»ΡŽΠ±Ρ‹Π΅ силы нСэлСктричСского происхоТдСния, Π΄Π΅ΠΉΡΡ‚Π²ΡƒΡŽΡ‰ΠΈΠ΅ Π½Π° элСктричСскиС заряды.

ΠžΠΏΡ€Π΅Π΄Π΅Π»Π΅Π½ΠΈΠ΅ 2. Π­Π”Π‘ (элСктродвиТущая сила) – это ΠΎΡ‚Π½ΠΎΡˆΠ΅Π½ΠΈΠ΅ Ρ€Π°Π±ΠΎΡ‚Ρ‹ сторонних сил ΠΏΠΎ пСрСносу ΠΏΠΎΠ»ΠΎΠΆΠΈΡ‚Π΅Π»ΡŒΠ½ΠΎΠ³ΠΎ заряда q вдоль Π·Π°ΠΌΠΊΠ½ΡƒΡ‚ΠΎΠ³ΠΎ ΠΊΠΎΠ½Ρ‚ΡƒΡ€Π° ΠΊ этому заряду.

{\cal E}=\frac{A_{ст}}{q}

A_{ст}={\cal E} q

Β 

Π—Π°ΠΊΠΎΠ½ Ома для ΠΏΠΎΠ»Π½ΠΎΠΉ Ρ†Π΅ΠΏΠΈ

Β 

r – Π²Π½ΡƒΡ‚Ρ€Π΅Π½Π½Π΅Π΅ сопротивлСниС источника

I=\frac{{\cal E}}{R+r}

\left[{\cal E}\right]=1Π’

Β 

НапряТСниС Π½Π° ΠΊΠ»Π΅ΠΌΠΌΠ°Ρ… источника

Β 

Β 

\cases{U=IR\cr I=\frac{{\cal E}}{R+r}}

I=\frac{{\cal E}}{R+r}\Rightarrow R+r=\frac{{\cal E}}{I}

R=\frac{{\cal E}}{I}-r

U=IR=I\frac{{\cal E}}{I}-r={\cal E} -Ir

НапряТСниС Π½Π° ΠΊΠ»Π΅ΠΌΠΌΠ°Ρ… источника:

U={\cal E} -Ir

Β 

Π’ΠΎΠΊ ΠΊΠΎΡ€ΠΎΡ‚ΠΊΠΎΠ³ΠΎ замыкания

Β 

ΠŸΡ€ΠΈ Π½Π΅ΠΊΠΎΡ‚ΠΎΡ€Ρ‹Ρ… условиях Ρ‚ΠΎΠΊ ΠΌΠΎΠΆΠ΅Ρ‚ Π½Π΅ ΠΏΠΎΠΉΡ‚ΠΈ Ρ‡Π΅Ρ€Π΅Π· Π½Π°Π³Ρ€ΡƒΠ·ΠΊΡƒ, ΠΈ Ρ†Π΅ΠΏΡŒ Π±ΡƒΠ΄Π΅Ρ‚ Π·Π°ΠΌΠΊΠ½ΡƒΡ‚Π° Π½Π°ΠΊΠΎΡ€ΠΎΡ‚ΠΊΠΎ.

Β 

Β 

I_{ΠΊ.Π·.}=\frac{{\cal E}}{r}

Π’Π°ΠΊ ΠΊΠ°ΠΊ Π²Π½ΡƒΡ‚Ρ€Π΅Π½Π½Π΅Π΅ сопротивлСниС Ρ†Π΅ΠΏΠΈ, ΠΊΠ°ΠΊ ΠΏΡ€Π°Π²ΠΈΠ»ΠΎ, ΠΌΠ½ΠΎΠ³ΠΎ мСньшС Π½Π°Π³Ρ€ΡƒΠ·ΠΊΠΈ, Ρ‚ΠΎΠΊΠΈ ΠΊΠΎΡ€ΠΎΡ‚ΠΊΠΎΠ³ΠΎ замыкания ΠΎΡ‡Π΅Π½ΡŒ Π²Π΅Π»ΠΈΠΊΠΈ, Ρ‡Ρ‚ΠΎ ΠΏΡ€ΠΈΠ²ΠΎΠ΄ΠΈΡ‚ ΠΊ Π±ΠΎΠ»ΡŒΡˆΠΎΠΌΡƒ ΡΠ½Π΅Ρ€Π³ΠΎΠ²Ρ‹Π΄Π΅Π»Π΅Π½ΠΈΡŽ ΠΈ ΠΏΠΎΠΆΠ°Ρ€Π°ΠΌ.

Β 

Π‘ΠΎΠ΅Π΄ΠΈΠ½Π΅Π½ΠΈΠ΅ {\cal E}

Β 

Если Π½Π°ΠΏΡ€Π°Π²Π»Π΅Π½ΠΈΠ΅ Ρ‚ΠΎΠΊΠ° совпадаСт с Π½Π°ΠΏΡ€Π°Π²Π»Π΅Π½ΠΈΠ΅ΠΌ ΠΎΠ±Ρ…ΠΎΠ΄Π°, Ρ‚ΠΎ Ρ‚ΠΎΠΊ Π±Π΅Ρ€ΡƒΡ‚ с плюсом.

Если Π½Π°ΠΏΡ€Π°Π²Π»Π΅Π½ΠΈΠ΅ Ρ‚ΠΎΠΊΠ° ΠΏΡ€ΠΎΡ‚ΠΈΠ²ΠΎΠΏΠΎΠ»ΠΎΠΆΠ½ΠΎ Π½Π°ΠΏΡ€Π°Π²Π»Π΅Π½ΠΈΡŽ ΠΎΠ±Ρ…ΠΎΠ΄Π°, Ρ‚ΠΎ Ρ‚ΠΎΠΊ Π±Π΅Ρ€ΡƒΡ‚ с минусом.

Если ΠΌΡ‹ ΠΎΠ±Ρ…ΠΎΠ΄ΠΈΠΌ {\cal E} ΠΎΡ‚ «–» ΠΊ Β«+Β», Ρ‚ΠΎ {\cal E} Π±Π΅Ρ€ΡƒΡ‚ с Β«+Β».

Если ΠΌΡ‹ ΠΎΠ±Ρ…ΠΎΠ΄ΠΈΠΌ {\cal E} ΠΎΡ‚ Β«+Β» ΠΊ «–», Ρ‚ΠΎ {\cal E} Π±Π΅Ρ€ΡƒΡ‚ с Β«-Β».

IR+Ir_{1}+Ir_{2}+Ir_{3}={\cal E} _{1}-{\cal E} _{3}-{\cal E} _{3}

I=\frac{{\cal E} _{1}-{\cal E} _{3}-{\cal E} _{3}}{R+r_{1}+r_{2}+r_{3}}

Β 

ΠŸΠΎΡΠ»Π΅Π΄ΠΎΠ²Π°Ρ‚Π΅Π»ΡŒΠ½ΠΎΠ΅ ΠΈ ΠΏΠ°Ρ€Π°Π»Π»Π΅Π»ΡŒΠ½ΠΎΠ΅ соСдинСниС n ΠΎΠ΄ΠΈΠ½Π°ΠΊΠΎΠ²Ρ‹Ρ… {\cal E}

Β 

Π°) ΠŸΠΎΡΠ»Π΅Π΄ΠΎΠ²Π°Ρ‚Π΅Π»ΡŒΠ½ΠΎΠ΅ соСдинСниС

Β 

I=\frac{n{\cal E}}{R+nr}

Π±) ΠŸΠ°Ρ€Π°Π»Π»Π΅Π»ΡŒΠ½ΠΎΠ΅ соСдинСниС

I=\frac{{\cal E}}{R+\frac{r}{n}}

ΠΎΠΏΡ€Π΅Π΄Π΅Π»Π΅Π½ΠΈΠ΅ ΠΈ Ρ„ΠΎΡ€ΠΌΡƒΠ»Π°, Π² Ρ‡Ρ‘ΠΌ измСряСтся, Ρ€Π°Π±ΠΎΡ‚Π° источника элСктродвиТущСй силы

ЭлСктричСский Ρ‚ΠΎΠΊ Π½Π΅ ΠΏΡ€ΠΎΡ‚Π΅ΠΊΠ°Π΅Ρ‚ Π² ΠΌΠ΅Π΄Π½ΠΎΠΌ ΠΏΡ€ΠΎΠ²ΠΎΠ΄Π΅ ΠΏΠΎ Ρ‚ΠΎΠΉ ΠΆΠ΅ ΠΏΡ€ΠΈΡ‡ΠΈΠ½Π΅, ΠΏΠΎ ΠΊΠΎΡ‚ΠΎΡ€ΠΎΠΉ остаётся Π½Π΅ΠΏΠΎΠ΄Π²ΠΈΠΆΠ½ΠΎΠΉ Π²ΠΎΠ΄Π° Π² Π³ΠΎΡ€ΠΈΠ·ΠΎΠ½Ρ‚Π°Π»ΡŒΠ½ΠΎΠΉ Ρ‚Ρ€ΡƒΠ±Π΅. Если ΠΎΠ΄ΠΈΠ½ ΠΊΠΎΠ½Π΅Ρ† Ρ‚Ρ€ΡƒΠ±Ρ‹ ΡΠΎΠ΅Π΄ΠΈΠ½ΠΈΡ‚ΡŒ с Ρ€Π΅Π·Π΅Ρ€Π²ΡƒΠ°Ρ€ΠΎΠΌ Ρ‚Π°ΠΊΠΈΠΌ ΠΎΠ±Ρ€Π°Π·ΠΎΠΌ, Ρ‡Ρ‚ΠΎΠ±Ρ‹ ΠΎΠ±Ρ€Π°Π·ΠΎΠ²Π°Π»Π°ΡΡŒ Ρ€Π°Π·Π½ΠΎΡΡ‚ΡŒ Π΄Π°Π²Π»Π΅Π½ΠΈΠΉ, ΠΆΠΈΠ΄ΠΊΠΎΡΡ‚ΡŒ Π±ΡƒΠ΄Π΅Ρ‚ Π²Ρ‹Ρ‚Π΅ΠΊΠ°Ρ‚ΡŒ ΠΈΠ· ΠΎΠ΄Π½ΠΎΠ³ΠΎ ΠΊΠΎΠ½Ρ†Π°. Аналогичным ΠΎΠ±Ρ€Π°Π·ΠΎΠΌ, для поддСрТания постоянного Ρ‚ΠΎΠΊΠ° Π½Π΅ΠΎΠ±Ρ…ΠΎΠ΄ΠΈΠΌΠΎ внСшнСС воздСйствиС, ΠΏΠ΅Ρ€Π΅ΠΌΠ΅Ρ‰Π°ΡŽΡ‰Π΅Π΅ заряды. Π­Ρ‚ΠΎ воздСйствиС называСтся элСктродвиТущая сила ΠΈΠ»ΠΈ Π­Π”Π‘.

ΠžΡ‚ элСктростатики ΠΊ элСктрокинСтикС

ΠœΠ΅ΠΆΠ΄Ρƒ ΠΊΠΎΠ½Ρ†ΠΎΠΌ XVIII ΠΈ Π½Π°Ρ‡Π°Π»ΠΎΠΌ XIX Π²Π΅ΠΊΠ° Ρ€Π°Π±ΠΎΡ‚Ρ‹ Ρ‚Π°ΠΊΠΈΡ… ΡƒΡ‡Ρ‘Π½Ρ‹Ρ…, ΠΊΠ°ΠΊ ΠšΡƒΠ»ΠΎΠ½, Π›Π°Π³Ρ€Π°Π½ΠΆ ΠΈ ΠŸΡƒΠ°ΡΡΠΎΠ½, Π·Π°Π»ΠΎΠΆΠΈΠ»ΠΈ матСматичСскиС основы опрСдСлСния элСктростатичСских Π²Π΅Π»ΠΈΡ‡ΠΈΠ½. ΠŸΡ€ΠΎΠ³Ρ€Π΅ΡΡ Π² ΠΏΠΎΠ½ΠΈΠΌΠ°Π½ΠΈΠΈ элСктричСства Π½Π° этом историчСском этапС ΠΎΡ‡Π΅Π²ΠΈΠ΄Π΅Π½. Π€Ρ€Π°Π½ΠΊΠ»ΠΈΠ½ ΡƒΠΆΠ΅ Π²Π²Ρ‘Π» понятиС «количСство элСктричСской субстанции», Π½ΠΎ ΠΏΠΎΠΊΠ° Π΅Ρ‰Ρ‘ ΠΈ ΠΎΠ½, Π½ΠΈ Π΅Π³ΠΎ ΠΏΡ€Π΅Π΅ΠΌΠ½ΠΈΠΊΠΈ Π½Π΅ смогли Π΅Π³ΠΎ ΠΈΠ·ΠΌΠ΅Ρ€ΠΈΡ‚ΡŒ.

БлСдуя Π·Π° экспСримСнтами Π“Π°Π»ΡŒΠ²Π°Π½ΠΈ, Π’ΠΎΠ»ΡŒΡ‚Π° пытался Π½Π°ΠΉΡ‚ΠΈ подтвСрТдСния Ρ‚ΠΎΠ³ΠΎ, Ρ‡Ρ‚ΠΎ Β«Π³Π°Π»ΡŒΠ²Π°Π½ΠΈΡ‡Π΅ΡΠΊΠΈΠ΅ Тидкости» ΠΆΠΈΠ²ΠΎΡ‚Π½ΠΎΠ³ΠΎ Π±Ρ‹Π»ΠΈ ΠΎΠ΄Π½ΠΎΠΉ ΠΏΡ€ΠΈΡ€ΠΎΠ΄Ρ‹ со статичСским элСктричСством. Π’ поисках истины ΠΎΠ½ ΠΎΠ±Π½Π°Ρ€ΡƒΠΆΠΈΠ», Ρ‡Ρ‚ΠΎ ΠΊΠΎΠ³Π΄Π° Π΄Π²Π° элСктрода ΠΈΠ· Ρ€Π°Π·Π½Ρ‹Ρ… ΠΌΠ΅Ρ‚Π°Π»Π»ΠΎΠ² ΠΊΠΎΠ½Ρ‚Π°ΠΊΡ‚ΠΈΡ€ΡƒΡŽΡ‚ Ρ‡Π΅Ρ€Π΅Π· элСктролит, ΠΎΠ±Π° Π·Π°Ρ€ΡΠΆΠ°ΡŽΡ‚ΡΡ ΠΈ ΠΎΡΡ‚Π°ΡŽΡ‚ΡΡ заряТСнными нСсмотря Π½Π° Π·Π°ΠΌΡ‹ΠΊΠ°Π½ΠΈΠ΅ ΠΊΠΎΠ½Ρ‚ΡƒΡ€Π° Π½Π°Π³Ρ€ΡƒΠ·ΠΊΠΎΠΉ. Π­Ρ‚ΠΎ явлСниС Π½Π΅ соотвСтствовало ΡΡƒΡ‰Π΅ΡΡ‚Π²ΡƒΡŽΡ‰ΠΈΠΌ прСдставлСниям ΠΎΠ± элСктричСствС ΠΏΠΎΡ‚ΠΎΠΌΡƒ, Ρ‡Ρ‚ΠΎ элСктростатичСскиС заряды Π² ΠΏΠΎΠ΄ΠΎΠ±Π½ΠΎΠΌ случаС Π΄ΠΎΠ»ΠΆΠ½Ρ‹ Π±Ρ‹Π»ΠΈ Ρ€Π΅ΠΊΠΎΠΌΠ±ΠΈΠ½ΠΈΡ€ΠΎΠ²Π°Ρ‚ΡŒ.

Π’ΠΎΠ»ΡŒΡ‚Π° Π²Π²Ρ‘Π» Π½ΠΎΠ²ΠΎΠ΅ ΠΎΠΏΡ€Π΅Π΄Π΅Π»Π΅Π½ΠΈΠ΅ силы, Π΄Π΅ΠΉΡΡ‚Π²ΡƒΡŽΡ‰Π΅ΠΉ Π² Π½Π°ΠΏΡ€Π°Π²Π»Π΅Π½ΠΈΠΈ раздСлСния зарядов ΠΈ ΠΏΠΎΠ΄Π΄Π΅Ρ€ΠΆΠ°Π½ΠΈΠΈ ΠΈΡ… Π² Ρ‚Π°ΠΊΠΎΠΌ состоянии. Он Π½Π°Π·Π²Π°Π» Π΅Ρ‘ элСктродвиТущСй. ПодобноС объяснСниС описания Ρ€Π°Π±ΠΎΡ‚Ρ‹ Π±Π°Ρ‚Π°Ρ€Π΅ΠΈ Π½Π΅ Π²ΠΏΠΈΡΡ‹Π²Π°Π»ΠΎΡΡŒ Π² тСорСтичСскиС основы Ρ„ΠΈΠ·ΠΈΠΊΠΈ Ρ‚ΠΎΠ³ΠΎ Π²Ρ€Π΅ΠΌΠ΅Π½ΠΈ. Π’ ΠšΡƒΠ»ΠΎΠ½ΠΎΠ²ΡΠΊΠΎΠΉ ΠΏΠ°Ρ€Π°Π΄ΠΈΠ³ΠΌΠ΅ ΠΏΠ΅Ρ€Π²ΠΎΠΉ Ρ‚Ρ€Π΅Ρ‚ΠΈ XIX Π²Π΅ΠΊΠ° э. Π΄. с. Π’ΠΎΠ»ΡŒΡ‚Π° ΠΎΠΏΡ€Π΅Π΄Π΅Π»ΡΠ»Π°ΡΡŒ ΡΠΏΠΎΡΠΎΠ±Π½ΠΎΡΡ‚ΡŒΡŽ ΠΎΠ΄Π½ΠΈΡ… Ρ‚Π΅Π» Π²Ρ‹Ρ€Π°Π±Π°Ρ‚Ρ‹Π²Π°Ρ‚ΡŒ элСктричСство Π² Π΄Ρ€ΡƒΠ³ΠΈΡ….

Π’Π°ΠΆΠ½Π΅ΠΉΡˆΠΈΠΉ Π²ΠΊΠ»Π°Π΄ Π² объяснСниС Ρ€Π°Π±ΠΎΡ‚Ρ‹ элСктричСских Ρ†Π΅ΠΏΠ΅ΠΉ внёс Ом. Π Π΅Π·ΡƒΠ»ΡŒΡ‚Π°Ρ‚Ρ‹ ряда экспСримСнтов ΠΏΡ€ΠΈΠ²Π΅Π»ΠΈ Π΅Π³ΠΎ ΠΊ ΠΏΠΎΡΡ‚Ρ€ΠΎΠ΅Π½ΠΈΡŽ Ρ‚Π΅ΠΎΡ€ΠΈΠΈ элСктропроводности. Он Π²Π²Ρ‘Π» Π²Π΅Π»ΠΈΡ‡ΠΈΠ½Ρƒ «напряТСниС» ΠΈ ΠΎΠΏΡ€Π΅Π΄Π΅Π»ΠΈΠ» Π΅Ρ‘ ΠΊΠ°ΠΊ Ρ€Π°Π·Π½ΠΎΡΡ‚ΡŒ ΠΏΠΎΡ‚Π΅Π½Ρ†ΠΈΠ°Π»ΠΎΠ² Π½Π° ΠΊΠΎΠ½Ρ‚Π°ΠΊΡ‚Π°Ρ…. Подобно Π€ΡƒΡ€ΡŒΠ΅, ΠΊΠΎΡ‚ΠΎΡ€Ρ‹ΠΉ Π² своСй Ρ‚Π΅ΠΎΡ€ΠΈΠΈ Ρ€Π°Π·Π»ΠΈΡ‡Π°Π» количСство Ρ‚Π΅ΠΏΠ»Π° ΠΈ Ρ‚Π΅ΠΌΠΏΠ΅Ρ€Π°Ρ‚ΡƒΡ€Ρƒ Π² Ρ‚Π΅ΠΏΠ»ΠΎΠΏΠ΅Ρ€Π΅Π΄Π°Ρ‡Π΅, Ом создал модСль ΠΏΠΎ Π°Π½Π°Π»ΠΎΠ³ΠΈΠΈ, ΡΠ²ΡΠ·Ρ‹Π²Π°ΡŽΡ‰ΡƒΡŽ количСство ΠΏΠ΅Ρ€Π΅ΠΌΠ΅Ρ‰Π°Π΅ΠΌΠΎΠ³ΠΎ заряда, напряТСниС ΠΈ ΡΠ»Π΅ΠΊΡ‚Ρ€ΠΎΠΏΡ€ΠΎΠ²ΠΎΠ΄Π½ΠΎΡΡ‚ΡŒ. Π—Π°ΠΊΠΎΠ½ Ома Π½Π΅ ΠΏΡ€ΠΎΡ‚ΠΈΠ²ΠΎΡ€Π΅Ρ‡ΠΈΠ» Π½Π°ΠΊΠΎΠΏΠ»Π΅Π½Π½Ρ‹ΠΌ знаниям ΠΎΠ± элСктростатичСском элСктричСствС.

Π—Π°Ρ‚Π΅ΠΌ, благодаря ΠœΠ°ΠΊΡΠ²Π΅Π»Π»Ρƒ ΠΈ Π€Π°Ρ€Π°Π΄Π΅ΡŽ, ΠΏΠΎΡΡΠ½ΠΈΡ‚Π΅Π»ΡŒΠ½Ρ‹Π΅ ΠΌΠΎΠ΄Π΅Π»ΠΈ Ρ‚ΠΎΠΊΠ° ΠΏΠΎΠ»ΡƒΡ‡ΠΈΠ»ΠΈ Π½ΠΎΠ²ΡƒΡŽ Ρ‚Π΅ΠΎΡ€ΠΈΡŽ поля. Π­Ρ‚ΠΎ ΠΏΠΎΠ·Π²ΠΎΠ»ΠΈΠ»ΠΎ Ρ€Π°Π·Ρ€Π°Π±ΠΎΡ‚Π°Ρ‚ΡŒ ΡΠ²ΡΠ·Π°Π½Π½ΡƒΡŽ с ΠΏΠΎΠ»Π΅ΠΌ ΠΊΠΎΠ½Ρ†Π΅ΠΏΡ†ΠΈΡŽ энСргии ΠΊΠ°ΠΊ для статичСских ΠΏΠΎΡ‚Π΅Π½Ρ†ΠΈΠ°Π»ΠΎΠ², Ρ‚Π°ΠΊ ΠΈ для элСктродвиТущСй силы. ΠžΡΠ½ΠΎΠ²Π½Ρ‹Π΅ Π΄Π°Ρ‚Ρ‹ ΡΠ²ΠΎΠ»ΡŽΡ†ΠΈΠΈ понятия Π­Π”Π‘:

  • 1800 Π³. β€” созданиС Π’ΠΎΠ»ΡŒΡ‚ΠΎΠΉ Π³Π°Π»ΡŒΠ²Π°Π½ΠΈΡ‡Π΅ΡΠΊΠΎΠΉ Π±Π°Ρ‚Π°Ρ€Π΅ΠΈ;
  • 1826 Π³. β€” Ом Ρ„ΠΎΡ€ΠΌΡƒΠ»ΠΈΡ€ΡƒΠ΅Ρ‚ свой Π·Π°ΠΊΠΎΠ½ для ΠΏΠΎΠ»Π½ΠΎΠΉ Ρ†Π΅ΠΏΠΈ;
  • 1831 Π³. β€” ΠΎΠ±Π½Π°Ρ€ΡƒΠΆΠ΅Π½ΠΈΠ΅ элСктромагнитной ΠΈΠ½Π΄ΡƒΠΊΡ†ΠΈΠΈ Π€Π°Ρ€Π°Π΄Π΅Π΅ΠΌ.

ΠžΠΏΡ€Π΅Π΄Π΅Π»Π΅Π½ΠΈΠ΅ ΠΈ физичСский смысл

ΠŸΡ€ΠΈΠ»ΠΎΠΆΠ΅Π½ΠΈΠ΅ Π½Π΅ΠΊΠΎΡ‚ΠΎΡ€ΠΎΠΉ разности ΠΏΠΎΡ‚Π΅Π½Ρ†ΠΈΠ°Π»ΠΎΠ² ΠΌΠ΅ΠΆΠ΄Ρƒ двумя ΠΊΠΎΠ½Ρ†Π°ΠΌΠΈ ΠΏΡ€ΠΎΠ²ΠΎΠ΄Π½ΠΈΠΊΠ° создаст ΠΏΠ΅Ρ€Π΅Ρ‚Π΅ΠΊΠ°Π½ΠΈΠ΅ элСктронов ΠΎΡ‚ ΠΎΠ΄Π½ΠΎΠ³ΠΎ ΠΊΠΎΠ½Ρ†Π° ΠΊ Π΄Ρ€ΡƒΠ³ΠΎΠΌΡƒ. Но этого нСдостаточно для поддСрТания ΠΏΠΎΡ‚ΠΎΠΊΠ° зарядов Π² ΠΏΡ€ΠΎΠ²ΠΎΠ΄Π½ΠΈΠΊΠ΅. Π”Ρ€Π΅ΠΉΡ„ элСктронов ΠΏΡ€ΠΈΠ²ΠΎΠ΄ΠΈΡ‚ ΠΊ ΡƒΠΌΠ΅Π½ΡŒΡˆΠ΅Π½ΠΈΡŽ ΠΏΠΎΡ‚Π΅Π½Ρ†ΠΈΠ°Π»Π° Π΄ΠΎ ΠΌΠΎΠΌΠ΅Π½Ρ‚Π° Π΅Π³ΠΎ ΡƒΡ€Π°Π²Π½ΠΎΠ²Π΅ΡˆΠΈΠ²Π°Π½ΠΈΡ (ΠΏΡ€Π΅ΠΊΡ€Π°Ρ‰Π΅Π½ΠΈΠ΅ Ρ‚ΠΎΠΊΠ°). Π’Π°ΠΊΠΈΠΌ ΠΎΠ±Ρ€Π°Π·ΠΎΠΌ, для создания постоянного Ρ‚ΠΎΠΊΠ° Π½Π΅ΠΎΠ±Ρ…ΠΎΠ΄ΠΈΠΌΡ‹ ΠΌΠ΅Ρ…Π°Π½ΠΈΠ·ΠΌΡ‹, Π½Π΅ΠΏΡ€Π΅Ρ€Ρ‹Π²Π½ΠΎ Π²ΠΎΠ·Π²Ρ€Π°Ρ‰Π°ΡŽΡ‰ΠΈΠ΅ ΠΎΠΏΠΈΡΠ°Π½Π½ΡƒΡŽ систСму Π² ΠΏΠ΅Ρ€Π²ΠΎΠ½Π°Ρ‡Π°Π»ΡŒΠ½ΡƒΡŽ ΠΊΠΎΠ½Ρ„ΠΈΠ³ΡƒΡ€Π°Ρ†ΠΈΡŽ, Ρ‚ΠΎ Π΅ΡΡ‚ΡŒ, ΠΏΡ€Π΅ΠΏΡΡ‚ΡΡ‚Π²ΡƒΡŽΡ‰ΠΈΠ΅ Π°Π³Ρ€Π΅Π³Π°Ρ†ΠΈΠΈ зарядов Π² Ρ€Π΅Π·ΡƒΠ»ΡŒΡ‚Π°Ρ‚Π΅ ΠΈΡ… двиТСния. Для этой Ρ†Π΅Π»ΠΈ ΠΈΡΠΏΠΎΠ»ΡŒΠ·ΡƒΡŽΡ‚ΡΡ ΡΠΏΠ΅Ρ†ΠΈΠ°Π»ΡŒΠ½Ρ‹Π΅ устройства, Π½Π°Π·Ρ‹Π²Π°Π΅ΠΌΡ‹Π΅ источники питания.

Π’ качСствС ΠΈΠ»Π»ΡŽΡΡ‚Ρ€Π°Ρ†ΠΈΠΈ ΠΈΡ… Ρ€Π°Π±ΠΎΡ‚Ρ‹ ΡƒΠ΄ΠΎΠ±Π½ΠΎ Ρ€Π°ΡΡΠΌΠ°Ρ‚Ρ€ΠΈΠ²Π°Ρ‚ΡŒ Π·Π°ΠΌΠΊΠ½ΡƒΡ‚Ρ‹ΠΉ ΠΊΠΎΠ½Ρ‚ΡƒΡ€ ΠΈΠ· сопротивлСния ΠΈ Π³Π°Π»ΡŒΠ²Π°Π½ΠΈΡ‡Π΅ΡΠΊΠΎΠ³ΠΎ источника питания (Π±Π°Ρ‚Π°Ρ€Π΅ΠΈ). Если ΠΏΡ€Π΅Π΄ΠΏΠΎΠ»ΠΎΠΆΠΈΡ‚ΡŒ, Ρ‡Ρ‚ΠΎ Π²Π½ΡƒΡ‚Ρ€ΠΈ Π±Π°Ρ‚Π°Ρ€Π΅ΠΈ Ρ‚ΠΎΠΊΠ° Π½Π΅Ρ‚, Ρ‚ΠΎ описанная ΠΏΡ€ΠΎΠ±Π»Π΅ΠΌΠ° объСдинСния зарядов остаётся Π½Π΅Ρ€Π°Π·Ρ€Π΅ΡˆΡ‘Π½Π½ΠΎΠΉ. Но Π² Ρ†Π΅ΠΏΠΈ с Ρ€Π΅Π°Π»ΡŒΠ½Ρ‹ΠΌ источником питания элСктроны ΠΏΠ΅Ρ€Π΅ΠΌΠ΅Ρ‰Π°ΡŽΡ‚ΡΡ постоянно. Π­Ρ‚ΠΎ происходит благодаря Ρ‚ΠΎΠΌΡƒ, Ρ‡Ρ‚ΠΎ ΠΏΠΎΡ‚ΠΎΠΊ ΠΈΠΎΠ½ΠΎΠ² ΠΏΡ€ΠΎΡ‚Π΅ΠΊΠ°Π΅Ρ‚ ΠΈ Π²Π½ΡƒΡ‚Ρ€ΠΈ Π±Π°Ρ‚Π°Ρ€Π΅ΠΈ ΠΎΡ‚ ΠΎΡ‚Ρ€ΠΈΡ†Π°Ρ‚Π΅Π»ΡŒΠ½ΠΎΠ³ΠΎ элСктрода ΠΊ ΠΏΠΎΠ»ΠΎΠΆΠΈΡ‚Π΅Π»ΡŒΠ½ΠΎΠΌΡƒ. Π˜ΡΡ‚ΠΎΡ‡Π½ΠΈΠΊ энСргии, ΠΏΠ΅Ρ€Π΅ΠΌΠ΅Ρ‰Π°ΡŽΡ‰ΠΈΠΉ эти заряды Π² Π±Π°Ρ‚Π°Ρ€Π΅Π΅ β€” химичСскиС Ρ€Π΅Π°ΠΊΡ†ΠΈΠΈ. Вакая энСргия называСтся элСктродвиТущСй силой.

Π­Π”Π‘ являСтся характСристикой любого источника энСргии, способного ΡƒΠΏΡ€Π°Π²Π»ΡΡ‚ΡŒ Π΄Π²ΠΈΠΆΠ΅Π½ΠΈΠ΅ΠΌ элСктричСских зарядов Π² Ρ†Π΅ΠΏΠΈ. Π’ Π°Π½Π°Π»ΠΎΠ³ΠΈΠΈ с Π·Π°ΠΌΠΊΠ½ΡƒΡ‚Ρ‹ΠΌ гидравличСским ΠΊΠΎΠ½Ρ‚ΡƒΡ€ΠΎΠΌ Ρ€Π°Π±ΠΎΡ‚Π° источника э. Π΄. с. соотвСтствуСт Ρ€Π°Π±ΠΎΡ‚Π΅ насоса для создания давлСния Π²ΠΎΠ΄Ρ‹. ΠŸΠΎΡΡ‚ΠΎΠΌΡƒ Π·Π½Π°Ρ‡ΠΎΠΊ, ΠΎΠ±ΠΎΠ·Π½Π°Ρ‡Π°ΡŽΡ‰ΠΈΠΉ эти устройства, Π½Π΅ΠΎΡ‚Π»ΠΈΡ‡ΠΈΠΌ Π½Π° гидравличСских ΠΈ элСктричСских схСмах.

НСсмотря Π½Π° Π½Π°Π·Π²Π°Π½ΠΈΠ΅, элСктродвиТущая сила Π½Π° самом Π΄Π΅Π»Π΅ Π½Π΅ являСтся силой ΠΈ измСряСтся Π² Π²ΠΎΠ»ΡŒΡ‚Π°Ρ…. Π•Ρ‘ числСнноС Π·Π½Π°Ρ‡Π΅Π½ΠΈΠ΅ Ρ€Π°Π²Π½ΠΎ Ρ€Π°Π±ΠΎΡ‚Π΅ ΠΏΠΎ ΠΏΠ΅Ρ€Π΅ΠΌΠ΅Ρ‰Π΅Π½ΠΈΡŽ заряда ΠΏΠΎ Π·Π°ΠΌΠΊΠ½ΡƒΡ‚ΠΎΠΉ Ρ†Π΅ΠΏΠΈ. Π­Π”Π‘ источника выраТаСтся Ρ„ΠΎΡ€ΠΌΡƒΠ»ΠΎΠΉ E=A/q, Π² ΠΊΠΎΡ‚ΠΎΡ€ΠΎΠΉ:

  • E β€” элСктродвиТущая сила Π² Π²ΠΎΠ»ΡŒΡ‚Π°Ρ…;
  • A β€” Ρ€Π°Π±ΠΎΡ‚Π° сторонних сил ΠΏΠΎ ΠΏΠ΅Ρ€Π΅ΠΌΠ΅Ρ‰Π΅Π½ΠΈΡŽ заряда Π² дТоулях;
  • q β€” ΠΏΠ΅Ρ€Π΅ΠΌΠ΅Ρ‰Ρ‘Π½Π½Ρ‹ΠΉ заряд Π² ΠΊΡƒΠ»ΠΎΠ½Π°Ρ….

Из этой Ρ„ΠΎΡ€ΠΌΡƒΠ»Ρ‹ Π­Π”Π‘ слСдуСт, Ρ‡Ρ‚ΠΎ элСктродвиТущая сила Π½Π΅ являСтся свойством Ρ†Π΅ΠΏΠΈ ΠΈΠ»ΠΈ Π½Π°Π³Ρ€ΡƒΠ·ΠΊΠΈ, Π° Π΅ΡΡ‚ΡŒ ΡΠΏΠΎΡΠΎΠ±Π½ΠΎΡΡ‚ΡŒ Π³Π΅Π½Π΅Ρ€Π°Ρ‚ΠΎΡ€Π° элСктроэнСргии ΠΊ Ρ€Π°Π·Π΄Π΅Π»Π΅Π½ΠΈΡŽ зарядов.

Π‘Ρ€Π°Π²Π½Π΅Π½ΠΈΠ΅ с Ρ€Π°Π·Π½ΠΎΡΡ‚ΡŒΡŽ ΠΏΠΎΡ‚Π΅Π½Ρ†ΠΈΠ°Π»ΠΎΠ²

ЭлСктродвиТущая сила ΠΈ Ρ€Π°Π·Π½ΠΎΡΡ‚ΡŒ ΠΏΠΎΡ‚Π΅Π½Ρ†ΠΈΠ°Π»ΠΎΠ² Π² Ρ†Π΅ΠΏΠΈ ΠΎΡ‡Π΅Π½ΡŒ ΠΏΠΎΡ…ΠΎΠΆΠΈΠ΅ физичСскиС Π²Π΅Π»ΠΈΡ‡ΠΈΠ½Ρ‹, Ρ‚Π°ΠΊ ΠΊΠ°ΠΊ ΠΎΠ±Π° ΠΈΠ·ΠΌΠ΅Ρ€ΡΡŽΡ‚ΡΡ Π² Π²ΠΎΠ»ΡŒΡ‚Π°Ρ… ΠΈ ΠΎΠΏΡ€Π΅Π΄Π΅Π»ΡΡŽΡ‚ΡΡ Ρ€Π°Π±ΠΎΡ‚ΠΎΠΉ ΠΏΠΎ ΠΏΠ΅Ρ€Π΅ΠΌΠ΅Ρ‰Π΅Π½ΠΈΡŽ заряда. Одно ΠΈΠ· основных смысловых Ρ€Π°Π·Π»ΠΈΡ‡ΠΈΠΉ Π·Π°ΠΊΠ»ΡŽΡ‡Π°Π΅Ρ‚ΡΡ Π² Ρ‚ΠΎΠΌ, Ρ‡Ρ‚ΠΎ э. Π΄. с. (E) вызываСтся ΠΏΡƒΡ‚Ρ‘ΠΌ прСобразования ΠΊΠ°ΠΊΠΎΠΉ-Π»ΠΈΠ±ΠΎ энСргии Π² ΡΠ»Π΅ΠΊΡ‚Ρ€ΠΈΡ‡Π΅ΡΠΊΡƒΡŽ, Ρ‚ΠΎΠ³Π΄Π° ΠΊΠ°ΠΊ Ρ€Π°Π·Π½ΠΎΡΡ‚ΡŒ ΠΏΠΎΡ‚Π΅Π½Ρ†ΠΈΠ°Π»ΠΎΠ² (U) Ρ€Π΅Π°Π»ΠΈΠ·ΡƒΠ΅Ρ‚ ΡΠ»Π΅ΠΊΡ‚Ρ€ΠΈΡ‡Π΅ΡΠΊΡƒΡŽ ΡΠ½Π΅Ρ€Π³ΠΈΡŽ Π² Π΄Ρ€ΡƒΠ³ΠΈΠ΅ Π²ΠΈΠ΄Ρ‹. Π”Ρ€ΡƒΠ³ΠΈΠ΅ различия выглядят Ρ‚Π°ΠΊ:

  • E ΠΏΠ΅Ρ€Π΅Π΄Π°Ρ‘Ρ‚ ΡΠ½Π΅Ρ€Π³ΠΈΡŽ всСй Ρ†Π΅ΠΏΠΈ. U являСтся ΠΌΠ΅Ρ€ΠΎΠΉ энСргии ΠΌΠ΅ΠΆΠ΄Ρƒ двумя Ρ‚ΠΎΡ‡ΠΊΠ°ΠΌΠΈ Π½Π° схСмС.
  • Π• являСтся ΠΏΡ€ΠΈΡ‡ΠΈΠ½ΠΎΠΉ U, Π½ΠΎ Π½Π΅ Π½Π°ΠΎΠ±ΠΎΡ€ΠΎΡ‚.
  • Π• индуцируСтся Π² элСктричСском, ΠΌΠ°Π³Π½ΠΈΡ‚Π½ΠΎΠΌ ΠΈ Π³Ρ€Π°Π²ΠΈΡ‚Π°Ρ†ΠΈΠΎΠ½Π½ΠΎΠΌ ΠΏΠΎΠ»Π΅.
  • ΠšΠΎΠ½Ρ†Π΅ΠΏΡ†ΠΈΡ э. Π΄. с. ΠΏΡ€ΠΈΠΌΠ΅Π½ΠΈΠΌΠ° Ρ‚ΠΎΠ»ΡŒΠΊΠΎ ΠΊ элСктричСскому полю, Π² Ρ‚ΠΎ врСмя ΠΊΠ°ΠΊ Ρ€Π°Π·Π½ΠΎΡΡ‚ΡŒ ΠΏΠΎΡ‚Π΅Π½Ρ†ΠΈΠ°Π»ΠΎΠ² ΠΏΡ€ΠΈΠΌΠ΅Π½ΠΈΠΌΠ° ΠΊ ΠΌΠ°Π³Π½ΠΈΡ‚Π½Ρ‹ΠΌ, Π³Ρ€Π°Π²ΠΈΡ‚Π°Ρ†ΠΈΠΎΠ½Π½Ρ‹ΠΌ ΠΈ элСктричСским полям.

НапряТСниС Π½Π° ΠΊΠ»Π΅ΠΌΠΌΠ°Ρ… источника питания, ΠΊΠ°ΠΊ ΠΏΡ€Π°Π²ΠΈΠ»ΠΎ, отличаСтся ΠΎΡ‚ Π­Π”Π‘ источника. Π­Ρ‚ΠΎ происходит ΠΈΠ·-Π·Π° наличия Π²Π½ΡƒΡ‚Ρ€Π΅Π½Π½Π΅Π³ΠΎ сопротивлСния источника (элСктролита ΠΈ элСктродов, ΠΎΠ±ΠΌΠΎΡ‚ΠΎΠΊ Π³Π΅Π½Π΅Ρ€Π°Ρ‚ΠΎΡ€Π°). Π‘Π²ΡΠ·Ρ‹Π²Π°ΡŽΡ‰Π°Ρ Ρ€Π°Π·Π½ΠΎΡΡ‚ΡŒ ΠΏΠΎΡ‚Π΅Π½Ρ†ΠΈΠ°Π»ΠΎΠ² ΠΈ Π­Π”Π‘ источника Ρ‚ΠΎΠΊΠ° Ρ„ΠΎΡ€ΠΌΡƒΠ»Π° выглядит ΠΊΠ°ΠΊ U=E-Ir. Π’ этом Π²Ρ‹Ρ€Π°ΠΆΠ΅Π½ΠΈΠΈ:

  • U β€” напряТСниС Π½Π° ΠΊΠ»Π΅ΠΌΠΌΠ°Ρ… источника;
  • r β€” Π²Π½ΡƒΡ‚Ρ€Π΅Π½Π½Π΅Π΅ сопротивлСниС источника;
  • I β€” Ρ‚ΠΎΠΊ Π² Ρ†Π΅ΠΏΠΈ.

Из этой Ρ„ΠΎΡ€ΠΌΡƒΠ»Ρ‹ элСктродвиТущСй силы слСдуСт, Ρ‡Ρ‚ΠΎ э. Π΄. с. Ρ€Π°Π²Π½Π° Π½Π°ΠΏΡ€ΡΠΆΠ΅Π½ΠΈΡŽ ΠΊΠΎΠ³Π΄Π° Ρ‚ΠΎΠΊ Π² Ρ†Π΅ΠΏΠΈ Π½Π΅ Ρ‚Π΅Ρ‡Ρ‘Ρ‚. Π˜Π΄Π΅Π°Π»ΡŒΠ½Ρ‹ΠΉ источник Π­Π”Π‘ создаёт Ρ€Π°Π·Π½ΠΎΡΡ‚ΡŒ ΠΏΠΎΡ‚Π΅Π½Ρ†ΠΈΠ°Π»ΠΎΠ² нСзависимо ΠΎΡ‚ Π½Π°Π³Ρ€ΡƒΠ·ΠΊΠΈ (ΠΏΡ€ΠΎΡ‚Π΅ΠΊΠ°ΡŽΡ‰Π΅Π³ΠΎ Ρ‚ΠΎΠΊΠ°) ΠΈ Π½Π΅ ΠΎΠ±Π»Π°Π΄Π°Π΅Ρ‚ Π²Π½ΡƒΡ‚Ρ€Π΅Π½Π½ΠΈΠΌ сопротивлСниСм.

Π’ ΠΏΡ€ΠΈΡ€ΠΎΠ΄Π΅ Π½Π΅ ΠΌΠΎΠΆΠ΅Ρ‚ ΡΡƒΡ‰Π΅ΡΡ‚Π²ΠΎΠ²Π°Ρ‚ΡŒ источника с бСсконСчной ΠΌΠΎΡ‰Π½ΠΎΡΡ‚ΡŒΡŽ ΠΏΡ€ΠΈ Π·Π°ΠΌΡ‹ΠΊΠ°Π½ΠΈΠΈ Π½Π° ΠΊΠ»Π΅ΠΌΠΌΠ°Ρ…, ΠΊΠ°ΠΊ ΠΈ ΠΌΠ°Ρ‚Π΅Ρ€ΠΈΠ°Π»Π° с бСсконСчной ΠΏΡ€ΠΎΠ²ΠΎΠ΄ΠΈΠΌΠΎΡΡ‚ΡŒΡŽ. Π˜Π΄Π΅Π°Π»ΡŒΠ½Ρ‹ΠΉ источник ΠΈΡΠΏΠΎΠ»ΡŒΠ·ΡƒΠ΅Ρ‚ΡΡ ΠΊΠ°ΠΊ абстрактная матСматичСская модСль.

Π˜ΡΡ‚ΠΎΡ‡Π½ΠΈΠΊΠΈ элСктродвиТущСй силы

Π‘ΡƒΡ‚ΡŒ источника Π­Π”Π‘ Π·Π°ΠΊΠ»ΡŽΡ‡Π°Π΅Ρ‚ΡΡ Π² ΠΏΡ€Π΅ΠΎΠ±Ρ€Π°Π·ΠΎΠ²Π°Π½ΠΈΠΈ Π΄Ρ€ΡƒΠ³ΠΈΡ… Π²ΠΈΠ΄ΠΎΠ² энСргии Π² ΡΠ»Π΅ΠΊΡ‚Ρ€ΠΈΡ‡Π΅ΡΠΊΡƒΡŽ с ΠΏΠΎΠΌΠΎΡ‰ΡŒΡŽ сторонних сил. Π‘ Ρ‚ΠΎΡ‡ΠΊΠΈ зрСния Ρ„ΠΈΠ·ΠΈΠΊΠΈ обСспСчСния э. Π΄. с Ρ€Π°Π·Π»ΠΈΡ‡Π°ΡŽΡ‚ ΡΠ»Π΅Π΄ΡƒΡŽΡ‰ΠΈΠ΅ Π΄Π²Π° основных Π²ΠΈΠ΄Π° источников:

  • Π³Π°Π»ΡŒΠ²Π°Π½ΠΈΡ‡Π΅ΡΠΊΠΈΠ΅;
  • элСктромагнитныС.

ΠŸΠ΅Ρ€Π²Ρ‹Π΅ ΠΏΡ€Π΅Π΄ΡΡ‚Π°Π²Π»ΡΡŽΡ‚ собой элСктрохимичСскиС источники, основанныС Π½Π° Π²ΠΎΠ²Π»Π΅Ρ‡Π΅Π½ΠΈΠ΅ Π² Ρ…ΠΈΠΌΠΈΡ‡Π΅ΡΠΊΡƒΡŽ Ρ€Π΅Π°ΠΊΡ†ΠΈΡŽ процСсса пСрСноса элСктронов. Π’ ΠΎΠ±Ρ‹Ρ‡Π½Ρ‹Ρ… условиях химичСскиС взаимодСйствия ΡΠΎΠΏΡ€ΠΎΠ²ΠΎΠΆΠ΄Π°ΡŽΡ‚ΡΡ Π²Ρ‹Π΄Π΅Π»Π΅Π½ΠΈΠ΅ΠΌ ΠΈΠ»ΠΈ ΠΏΠΎΠ³Π»ΠΎΡ‰Π΅Π½ΠΈΠ΅ΠΌ Ρ‚Π΅ΠΏΠ»Π°, Π½ΠΎ сущСствуСт Π½Π΅ΠΌΠ°Π»ΠΎ Ρ€Π΅Π°ΠΊΡ†ΠΈΠΉ, Π² Ρ€Π΅Π·ΡƒΠ»ΡŒΡ‚Π°Ρ‚Π΅ ΠΊΠΎΡ‚ΠΎΡ€Ρ‹Ρ… гСнСрируСтся элСктричСская энСргия.

ЭлСктрохимичСскиС процСссы Π² Π±ΠΎΠ»ΡŒΡˆΠΈΠ½ΡΡ‚Π²Π΅ случаСв ΠΎΠ±Ρ€Π°Ρ‚ΠΈΠΌΡ‹, ΠΏΠΎΡΠΊΠΎΠ»ΡŒΠΊΡƒ энСргия элСктричСского Ρ‚ΠΎΠΊΠ° ΠΌΠΎΠΆΠ΅Ρ‚ Π±Ρ‹Ρ‚ΡŒ использована, Ρ‡Ρ‚ΠΎΠ±Ρ‹ Π·Π°ΡΡ‚Π°Π²ΠΈΡ‚ΡŒ Ρ€Π΅Π°Π³ΠΈΡ€ΠΎΠ²Π°Ρ‚ΡŒ вСщСства ΠΌΠ΅ΠΆΠ΄Ρƒ собой. Π­Ρ‚Π° Π²ΠΎΠ·ΠΌΠΎΠΆΠ½ΠΎΡΡ‚ΡŒ позволяСт ΡΠΎΠ·Π΄Π°Π²Π°Ρ‚ΡŒ возобновляСмыС Π³Π°Π»ΡŒΠ²Π°Π½ΠΈΡ‡Π΅ΡΠΊΠΈΠ΅ источники β€” аккумуляторы.

Π’ Π³Π΅Π½Π΅Ρ€Π°Ρ‚ΠΎΡ€Π°Ρ… Ρ‚ΠΎΠΊΠ° э. Π΄. с. создаётся Π΄Ρ€ΡƒΠ³ΠΈΠΌ способом. Π Π°Π·Π΄Π΅Π»Π΅Π½ΠΈΠ΅ зарядов происходит с ΠΏΠΎΠΌΠΎΡ‰ΡŒΡŽ явлСния элСктромагнитной ΠΈΠ½Π΄ΡƒΠΊΡ†ΠΈΠΈ, ΠΊΠΎΡ‚ΠΎΡ€ΠΎΠ΅ Π·Π°ΠΊΠ»ΡŽΡ‡Π°Π΅Ρ‚ΡΡ Π² Ρ‚ΠΎΠΌ, Ρ‡Ρ‚ΠΎ ΠΈΠ·ΠΌΠ΅Π½Π΅Π½ΠΈΠ΅ Π²Π΅Π»ΠΈΡ‡ΠΈΠ½Ρ‹ ΠΈΠ»ΠΈ направлСния ΠΌΠ°Π³Π½ΠΈΡ‚Π½ΠΎΠ³ΠΎ поля создаёт Π­Π”Π‘. Богласно Π·Π°ΠΊΠΎΠ½Ρƒ ЀарадСя, Π½Π°Ρ…ΠΎΠΆΠ΄Π΅Π½ΠΈΠ΅ э. Π΄. с. ΠΈΠ½Π΄ΡƒΠΊΡ†ΠΈΠΈ Π²ΠΎΠ·ΠΌΠΎΠΆΠ½ΠΎ ΠΈΠ· выраТСния E=β€”dΠ€/dt. Π’ этой Ρ„ΠΎΡ€ΠΌΡƒΠ»Π΅:

  • Π€ β€” ΠΌΠ°Π³Π½ΠΈΡ‚Π½Ρ‹ΠΉ ΠΏΠΎΡ‚ΠΎΠΊ;
  • t β€” врСмя.

Π­Π”Π‘ ΠΈΠ½Π΄ΡƒΠΊΡ†ΠΈΠΈ измСряСтся Ρ‚Π°ΠΊΠΆΠ΅ Π² Π²ΠΎΠ»ΡŒΡ‚Π°Ρ…. Π’ зависимости ΠΎΡ‚ Ρ‚ΠΎΠ³ΠΎ, ΠΊΠ°ΠΊΠΈΠΌ способом Π²Ρ‹Π·Ρ‹Π²Π°ΡŽΡ‚ΡΡ измСнСния ΠΌΠ°Π³Π½ΠΈΡ‚Π½ΠΎΠ³ΠΎ ΠΏΠΎΡ‚ΠΎΠΊΠ°, Ρ€Π°Π·Π»ΠΈΡ‡Π°ΡŽΡ‚:

  • ДинамичСски ΠΈΠ½Π΄ΡƒΡ†ΠΈΡ€ΠΎΠ²Π°Π½Π½ΡƒΡŽ. Когда Π² стационарном ΠΌΠ°Π³Π½ΠΈΡ‚Π½ΠΎΠΌ ΠΏΠΎΠ»Π΅ пСрСмСщаСтся ΠΏΡ€ΠΎΠ²ΠΎΠ΄Π½ΠΈΠΊ. Π₯Π°Ρ€Π°ΠΊΡ‚Π΅Ρ€Π΅Π½ для Π³Π΅Π½Π΅Ρ€Π°Ρ‚ΠΎΡ€ΠΎΠ².
  • БтатичСски ΠΈΠ½Π΄ΡƒΡ†ΠΈΡ€ΠΎΠ²Π°Π½Π½ΡƒΡŽ. Когда измСнСния ΠΏΠΎΡ‚ΠΎΠΊΠ° Π²ΠΎΠ·Π½ΠΈΠΊΠ°ΡŽΡ‚ ΠΈΠ·-Π·Π° ΠΈΠ·ΠΌΠ΅Π½Π΅Π½ΠΈΠΉ ΠΌΠ°Π³Π½ΠΈΡ‚Π½ΠΎΠ³ΠΎ поля Π²ΠΎΠΊΡ€ΡƒΠ³ Π½Π΅ΠΏΠΎΠ΄Π²ΠΈΠΆΠ½ΠΎΠ³ΠΎ ΠΏΡ€ΠΎΠ²ΠΎΠ΄Π½ΠΈΠΊΠ°. Π’Π°ΠΊ Ρ€Π°Π±ΠΎΡ‚Π°ΡŽΡ‚ трансформаторы.

Π‘ΡƒΡ‰Π΅ΡΡ‚Π²ΡƒΡŽΡ‚ Ρ‚Π°ΠΊΠΆΠ΅ источники э. Π΄. с, Π½Π΅ основанныС Π½Π° элСктрохимии ΠΈΠ»ΠΈ ΠΌΠ°Π³Π½ΠΈΡ‚Π½ΠΎΠΉ ΠΈΠ½Π΄ΡƒΠΊΡ†ΠΈΠΈ. К Ρ‚Π°ΠΊΠΈΠΌ устройствам ΠΌΠΎΠΆΠ½ΠΎ отнСсти ΠΏΠΎΠ»ΡƒΠΏΡ€ΠΎΠ²ΠΎΠ΄Π½ΠΈΠΊΠΎΠ²Ρ‹Π΅ фотоэлСмСнты, ΠΊΠΎΠ½Ρ‚Π°ΠΊΡ‚Π½Ρ‹Π΅ ΠΏΠΎΡ‚Π΅Π½Ρ†ΠΈΠ°Π»Ρ‹ ΠΈ ΠΏΡŒΠ΅Π·ΠΎΠΊΡ€ΠΈΡΡ‚Π°Π»Π»Ρ‹. ΠŸΠΎΠ½ΡΡ‚ΠΈΠ΅ Π­Π”Π‘ ΠΈΠΌΠ΅Π΅Ρ‚ практичСскоС ΠΏΡ€ΠΈΠΌΠ΅Π½Π΅Π½ΠΈΠ΅ ΠΏΡ€Π΅ΠΆΠ΄Π΅ всСго ΠΊΠ°ΠΊ ΠΏΠ°Ρ€Π°ΠΌΠ΅Ρ‚Ρ€ Π²Ρ‹Π±ΠΎΡ€Π° источников питания для Ρ‚Π΅Ρ… ΠΈΠ»ΠΈ ΠΈΠ½Ρ‹Ρ… Ρ†Π΅Π»Π΅ΠΉ. Π§Ρ‚ΠΎΠ±Ρ‹ ΠΏΠΎΠ»ΡƒΡ‡ΠΈΡ‚ΡŒ ΠΌΠ°ΠΊΡΠΈΠΌΠ°Π»ΡŒΠ½Ρ‹ΠΉ эффСкт ΠΎΡ‚ Ρ€Π°Π±ΠΎΡ‚Ρ‹ устройств Π² Ρ†Π΅ΠΏΠΈ, Π½ΡƒΠΆΠ½ΠΎ ΡΠΎΠ³Π»Π°ΡΠΎΠ²Ρ‹Π²Π°Ρ‚ΡŒ ΠΈΡ… возмоТности ΠΈ характСристики. ΠŸΡ€Π΅ΠΆΠ΄Π΅ всСго Π²Π½ΡƒΡ‚Ρ€Π΅Π½Π½Π΅Π΅ сопротивлСниС источника Π­Π”Π‘ силы с характСристиками ΠΏΠΎΠ΄ΠΊΠ»ΡŽΡ‡Π°Π΅ΠΌΠΎΠΉ Π½Π°Π³Ρ€ΡƒΠ·ΠΊΠΈ.

Π€ΠΈΠ·ΠΈΠΊΠ° – ЭлСктродвиТущая сила – БирмингСмский унивСрситСт

ЭлСктродвиТущая сила (Π­Π”Π‘) Ρ€Π°Π²Π½Π° разности ΠΏΠΎΡ‚Π΅Π½Ρ†ΠΈΠ°Π»ΠΎΠ² Π½Π° ΠΊΠ»Π΅ΠΌΠΌΠ°Ρ… ΠΏΡ€ΠΈ отсутствии Ρ‚ΠΎΠΊΠ°. Π­Π”Π‘ ΠΈ Ρ€Π°Π·Π½ΠΎΡΡ‚ΡŒ ΠΏΠΎΡ‚Π΅Π½Ρ†ΠΈΠ°Π»ΠΎΠ² Π½Π° ΠΊΠ»Π΅ΠΌΠΌΠ°Ρ… ( Π’, ) ΠΈΠ·ΠΌΠ΅Ρ€ΡΡŽΡ‚ΡΡ Π² Π²ΠΎΠ»ΡŒΡ‚Π°Ρ…, Π½ΠΎ это Π½Π΅ ΠΎΠ΄Π½ΠΎ ΠΈ Ρ‚ΠΎ ΠΆΠ΅. Π­Π”Π‘ ( Ο΅ ) – это количСство энСргии ( E ), обСспСчиваСмоС Π±Π°Ρ‚Π°Ρ€Π΅Π΅ΠΉ Π½Π° ΠΊΠ°ΠΆΠ΄Ρ‹ΠΉ ΠΊΡƒΠ»ΠΎΠ½ заряда ( Q ), проходящий Ρ‡Π΅Ρ€Π΅Π· Π½Π΅Π΅.

Как Ρ€Π°ΡΡΡ‡ΠΈΡ‚Π°Ρ‚ΡŒ Π­Π”Π‘?

Π­Π”Π‘ ΠΌΠΎΠΆΠ½ΠΎ Π·Π°ΠΏΠΈΡΠ°Ρ‚ΡŒ Ρ‡Π΅Ρ€Π΅Π· Π²Π½ΡƒΡ‚Ρ€Π΅Π½Π½Π΅Π΅ сопротивлСниС Π±Π°Ρ‚Π°Ρ€Π΅ΠΈ ( r ) Π³Π΄Π΅: Ο΅ = I (r + R )

Π§Ρ‚ΠΎ ΠΈΠ· Π·Π°ΠΊΠΎΠ½Π° Ома, ΠΌΡ‹ ΠΌΠΎΠΆΠ΅ΠΌ ΠΈΠ·ΠΌΠ΅Π½ΠΈΡ‚ΡŒ это Π² Ρ‚Π΅Ρ€ΠΌΠΈΠ½Π°Ρ… ΠΎΠΊΠΎΠ½Π΅Ρ‡Π½ΠΎΠ³ΠΎ сопротивлСния: Ο΅ = Π’ + Ir

Π­Π”Π‘ ячСйки ΠΌΠΎΠΆΠ΅Ρ‚ Π±Ρ‹Ρ‚ΡŒ ΠΎΠΏΡ€Π΅Π΄Π΅Π»Π΅Π½Π° ΠΏΡƒΡ‚Π΅ΠΌ измСрСния напряТСния Π½Π° ячСйкС с ΠΏΠΎΠΌΠΎΡ‰ΡŒΡŽ Π²ΠΎΠ»ΡŒΡ‚ΠΌΠ΅Ρ‚Ρ€Π° ΠΈ Ρ‚ΠΎΠΊΠ° Π² Ρ†Π΅ΠΏΠΈ с ΠΏΠΎΠΌΠΎΡ‰ΡŒΡŽ Π°ΠΌΠΏΠ΅Ρ€ΠΌΠ΅Ρ‚Ρ€Π° для Ρ€Π°Π·Π»ΠΈΡ‡Π½Ρ‹Ρ… сопротивлСний.Π—Π°Ρ‚Π΅ΠΌ ΠΌΡ‹ ΠΌΠΎΠΆΠ΅ΠΌ Π½Π°ΡΡ‚Ρ€ΠΎΠΈΡ‚ΡŒ схСму для опрСдСлСния Π­Π”Π‘, ΠΊΠ°ΠΊ ΠΏΠΎΠΊΠ°Π·Π°Π½ΠΎ Π½ΠΈΠΆΠ΅.

Π­Π”Π‘ ΠΈ Π²Π½ΡƒΡ‚Ρ€Π΅Π½Π½Π΅Π΅ сопротивлСниС элСктричСских элСмСнтов ΠΈ Π±Π°Ρ‚Π°Ρ€Π΅ΠΉ

ИсслСдованиС ЭМП

Как Π·Π°ΠΊΠΎΠ½ ЀарадСя соотносится с ЭМП?

Π—Π°ΠΊΠΎΠ½ ЀарадСя гласит, Ρ‡Ρ‚ΠΎ любоС ΠΈΠ·ΠΌΠ΅Π½Π΅Π½ΠΈΠ΅ ΠΌΠ°Π³Π½ΠΈΡ‚Π½ΠΎΠ³ΠΎ поля ΠΊΠ°Ρ‚ΡƒΡˆΠΊΠΈ Π±ΡƒΠ΄Π΅Ρ‚ ΠΈΠ½Π΄ΡƒΡ†ΠΈΡ€ΠΎΠ²Π°Ρ‚ΡŒ Π² ΠΊΠ°Ρ‚ΡƒΡˆΠΊΠ΅ Π­Π”Π‘ (Π° ΡΠ»Π΅Π΄ΠΎΠ²Π°Ρ‚Π΅Π»ΡŒΠ½ΠΎ, ΠΈ Ρ‚ΠΎΠΊ). Он ΠΏΡ€ΠΎΠΏΠΎΡ€Ρ†ΠΈΠΎΠ½Π°Π»Π΅Π½ минус скорости измСнСния ΠΌΠ°Π³Π½ΠΈΡ‚Π½ΠΎΠ³ΠΎ ΠΏΠΎΡ‚ΠΎΠΊΠ° ( Ο• ) (ΠΏΡ€ΠΈΠΌΠ΅Ρ‡Π°Π½ΠΈΠ΅ N – количСство Π²ΠΈΡ‚ΠΊΠΎΠ² Π² ΠΊΠ°Ρ‚ΡƒΡˆΠΊΠ΅).

Π˜ΡΠΏΠΎΠ»ΡŒΠ·ΡƒΡ Π·Π°ΠΊΠΎΠ½ ЀарадСя, общСство ΠΈΠ·Π²Π»Π΅ΠΊΠ»ΠΎ Π²Ρ‹Π³ΠΎΠ΄Ρƒ ΠΈΠ· Ρ‚Π°ΠΊΠΈΡ… Π²Π°ΠΆΠ½Ρ‹Ρ… Ρ‚Π΅Ρ…Π½ΠΎΠ»ΠΎΠ³ΠΈΠΉ, ΠΊΠ°ΠΊ трансформаторы, ΠΊΠΎΡ‚ΠΎΡ€Ρ‹Π΅ ΠΈΡΠΏΠΎΠ»ΡŒΠ·ΡƒΡŽΡ‚ΡΡ для ΠΏΠ΅Ρ€Π΅Π΄Π°Ρ‡ΠΈ элСктроэнСргии Π² Π½Π°Ρ†ΠΈΠΎΠ½Π°Π»ΡŒΠ½ΠΎΠΉ энСргосистСмС Π’Π΅Π»ΠΈΠΊΠΎΠ±Ρ€ΠΈΡ‚Π°Π½ΠΈΠΈ, которая Ρ‚Π΅ΠΏΠ΅Ρ€ΡŒ являСтся Π½Π΅ΠΎΠ±Ρ…ΠΎΠ΄ΠΈΠΌΠΎΡΡ‚ΡŒΡŽ Π² Π½Π°ΡˆΠΈΡ… Π΄ΠΎΠΌΠ°Ρ…. Π’Π°ΠΊΠΆΠ΅ ΠΎΠ½ ΠΈΡΠΏΠΎΠ»ΡŒΠ·ΡƒΠ΅Ρ‚ΡΡ Π² элСктричСских Π³Π΅Π½Π΅Ρ€Π°Ρ‚ΠΎΡ€Π°Ρ… ΠΈ двигатСлях, Ρ‚Π°ΠΊΠΈΡ… ΠΊΠ°ΠΊ ΠΏΠ»ΠΎΡ‚ΠΈΠ½Ρ‹ гидроэлСктростанций, ΠΊΠΎΡ‚ΠΎΡ€Ρ‹Π΅ производят элСктричСство, ΠΊΠΎΡ‚ΠΎΡ€ΠΎΠ΅ сСйчас являСтся Π½Π΅ΠΎΡ‚ΡŠΠ΅ΠΌΠ»Π΅ΠΌΠΎΠΉ Ρ‡Π°ΡΡ‚ΡŒΡŽ Π½Π°ΡˆΠΈΡ… соврСмСнных тСхнологичСских потрСбностСй. Π’Π΅ΠΊΡƒΡ‰ΠΈΠΉ ΠΈΡΡΠ»Π΅Π΄ΠΎΠ²Π°Ρ‚Π΅Π»ΡŒΡΠΊΠΈΠΉ ΠΏΡ€ΠΎΠ΅ΠΊΡ‚ MAG-DRIVE Π² Π‘ΠΈΡ€ΠΌΠΈΠ½Π³Π΅ΠΌΠ΅ Π½Π°ΠΏΡ€Π°Π²Π»Π΅Π½ Π½Π° поиск способов Ρ€Π°Π·Ρ€Π°Π±ΠΎΡ‚ΠΊΠΈ ΠΈ ΡƒΠ»ΡƒΡ‡ΡˆΠ΅Π½ΠΈΡ ΠΌΠ°Ρ‚Π΅Ρ€ΠΈΠ°Π»ΠΎΠ² с постоянными ΠΌΠ°Π³Π½ΠΈΡ‚Π°ΠΌΠΈ, ΠΊΠΎΡ‚ΠΎΡ€Ρ‹Π΅ ΠΌΠΎΠ³ΡƒΡ‚ Π±Ρ‹Ρ‚ΡŒ ΠΈΡΠΏΠΎΠ»ΡŒΠ·ΠΎΠ²Π°Π½Ρ‹ Π² элСктромобилях ΡΠ»Π΅Π΄ΡƒΡŽΡ‰Π΅Π³ΠΎ поколСния.ЭМП Ρ‚Π°ΠΊΠΆΠ΅ гСнСрируСтся солнСчными батарСями, поэтому ΠΎΠ½ΠΈ Π²Π°ΠΆΠ½Ρ‹ для исслСдований Π² области возобновляСмых источников энСргии.

Π›Π°Π±ΠΎΡ€Π°Ρ‚ΠΎΡ€Π½Ρ‹Π΅ признания

Π˜ΡΡΠ»Π΅Π΄ΠΎΠ²Π°Ρ‚Π΅Π»ΠΈ подкаста In the Laboratory Confessions Ρ€Π°ΡΡΠΊΠ°Π·Ρ‹Π²Π°ΡŽΡ‚ ΠΎ своСм Π»Π°Π±ΠΎΡ€Π°Ρ‚ΠΎΡ€Π½ΠΎΠΌ ΠΎΠΏΡ‹Ρ‚Π΅ Π² контСкстС практичСских экзамСнов A Level. Π­ΠΏΠΈΠ·ΠΎΠ΄Ρ‹, посвящСнныС ΠΏΡ€Π°Π²ΠΈΠ»ΡŒΠ½ΠΎΠΌΡƒ использованию Ρ†ΠΈΡ„Ρ€ΠΎΠ²Ρ‹Ρ… инструмСнтов (простоС гармоничСскоС Π΄Π²ΠΈΠΆΠ΅Π½ΠΈΠ΅), ΠΏΡ€Π°Π²ΠΈΠ»ΡŒΠ½ΠΎΠΌΡƒ ΠΏΠΎΡΡ‚Ρ€ΠΎΠ΅Π½ΠΈΡŽ ΠΏΡ€ΠΈΠ½Ρ†ΠΈΠΏΠΈΠ°Π»ΡŒΠ½Ρ‹Ρ… схСм (ΡƒΠ΄Π΅Π»ΡŒΠ½ΠΎΠ΅ сопротивлСниС Π² ΠΏΡ€ΠΎΠ²ΠΎΠ΄Π΅) ΠΈ использованию источников питания постоянного Ρ‚ΠΎΠΊΠ° (кондСнсаторов), ΠΈΠΌΠ΅ΡŽΡ‚ ΠΎΡ‚Π½ΠΎΡˆΠ΅Π½ΠΈΠ΅ ΠΊ экспСримСнту ΠΏΠΎ Π­Π”Π‘, Π½ΠΈΠΆΠ΅ Π²Ρ‹ ΠΌΠΎΠΆΠ΅Ρ‚Π΅ ΡƒΡΠ»Ρ‹ΡˆΠ°Ρ‚ΡŒ ΡƒΠ΄Π΅Π»ΡŒΠ½ΠΎΠ΅ сопротивлСниС. Π² ΠΏΡ€ΠΎΠ²ΠΎΠ΄Π½ΠΎΠΌ подкастС.

Как ΠΌΡ‹ ΠΈΠ½Ρ‚Π΅Ρ€ΠΏΡ€Π΅Ρ‚ΠΈΡ€ΡƒΠ΅ΠΌ наши Π΄Π°Π½Π½Ρ‹Π΅?

По ΠΌΠ΅Ρ€Π΅ увСличСния сопротивлСния ΠΏΠ΅Ρ€Π΅ΠΌΠ΅Π½Π½ΠΎΠ³ΠΎ рСзистора Π²Π΅Π»ΠΈΡ‡ΠΈΠ½Π° Ρ‚ΠΎΠΊΠ° Π±ΡƒΠ΄Π΅Ρ‚ ΡƒΠΌΠ΅Π½ΡŒΡˆΠ°Ρ‚ΡŒΡΡ. Π“Ρ€Π°Ρ„ΠΈΠΊ зависимости напряТСния ΠΎΡ‚ Ρ‚ΠΎΠΊΠ° Π΄ΠΎΠ»ΠΆΠ΅Π½ Π΄Π°Π²Π°Ρ‚ΡŒ Π»ΠΈΠ½Π΅ΠΉΠ½ΡƒΡŽ Π·Π°Π²ΠΈΡΠΈΠΌΠΎΡΡ‚ΡŒ, Π³Π΄Π΅ Π³Ρ€Π°Π΄ΠΈΠ΅Π½Ρ‚ Π»ΠΈΠ½ΠΈΠΈ Π΄Π°Π΅Ρ‚ ΠΎΡ‚Ρ€ΠΈΡ†Π°Ρ‚Π΅Π»ΡŒΠ½ΠΎΠ΅ Π²Π½ΡƒΡ‚Ρ€Π΅Π½Π½Π΅Π΅ сопротивлСниС ячСйки ( -r ), Π° Ρ‚ΠΎΡ‡ΠΊΠ° пСрСсСчСния Π΄Π°Π΅Ρ‚ Π­Π”Π‘ (напряТСниС, ΠΏΡ€ΠΈ ΠΊΠΎΡ‚ΠΎΡ€ΠΎΠΌ Ρ‚ΠΎΠΊ Ρ€Π°Π²Π΅Π½ 0).

Π’Ρ‹ΠΏΠΎΠ»Π½Π΅Π½ΠΈΠ΅ Π½Π΅ΡΠΊΠΎΠ»ΡŒΠΊΠΈΡ… ΠΈΠ·ΠΌΠ΅Ρ€Π΅Π½ΠΈΠΉ ΠΏΡ€ΠΈ Ρ€Π°Π·Π½Ρ‹Ρ… значСниях сопротивлСния даст большС Ρ‚ΠΎΡ‡Π΅ΠΊ Π½Π° Π³Ρ€Π°Ρ„ΠΈΠΊΠ΅ V-I, Ρ‡Ρ‚ΠΎ сдСлаСт ΠΏΠΎΠ΄Π±ΠΎΡ€ Π±ΠΎΠ»Π΅Π΅ Π½Π°Π΄Π΅ΠΆΠ½Ρ‹ΠΌ.Π’Π°ΠΊΠΆΠ΅ рСкомСндуСтся ΠΏΠΎΠ²Ρ‚ΠΎΡ€ΠΈΡ‚ΡŒ измСрСния, Ρ‚Π°ΠΊ ΠΊΠ°ΠΊ ячСйка Π±ΡƒΠ΄Π΅Ρ‚ постСпСнно ΡΡ‚Π΅ΠΊΠ°Ρ‚ΡŒ, Ρ‡Ρ‚ΠΎ повлияСт Π½Π° показания. Π’ΠΎ ΠΈΠ·Π±Π΅ΠΆΠ°Π½ΠΈΠ΅ разряда элСмСнта / Π±Π°Ρ‚Π°Ρ€Π΅ΠΈ Π΅Π΅ слСдуСт ΠΎΡ‚ΠΊΠ»ΡŽΡ‡Π°Ρ‚ΡŒ ΠΌΠ΅ΠΆΠ΄Ρƒ измСрСниями. Π’ качСствС Π°Π»ΡŒΡ‚Π΅Ρ€Π½Π°Ρ‚ΠΈΠ²Ρ‹ Π² схСму ΠΌΠΎΠΆΠ½ΠΎ Π²ΠΊΠ»ΡŽΡ‡ΠΈΡ‚ΡŒ Π²Ρ‹ΠΊΠ»ΡŽΡ‡Π°Ρ‚Π΅Π»ΡŒ. Π’Π°ΠΊΠΆΠ΅ Π½Π΅ рСкомСндуСтся ΠΈΡΠΏΠΎΠ»ΡŒΠ·ΠΎΠ²Π°Ρ‚ΡŒ аккумуляторныС Π±Π°Ρ‚Π°Ρ€Π΅ΠΈ, Ρ‚Π°ΠΊ ΠΊΠ°ΠΊ ΠΎΠ½ΠΈ ΠΈΠΌΠ΅ΡŽΡ‚ Π½ΠΈΠ·ΠΊΠΎΠ΅ Π²Π½ΡƒΡ‚Ρ€Π΅Π½Π½Π΅Π΅ сопротивлСниС.

НСсмотря Π½Π° Ρ‚ΠΎ, Ρ‡Ρ‚ΠΎ этот экспСримСнт довольно прост, ΠΎΠ½ ΠΏΠΎΠΌΠΎΠΆΠ΅Ρ‚ Π²Π°ΠΌ ΠΎΡ‚Π»ΠΈΡ‡ΠΈΡ‚ΡŒ ΠΊΠΎΠ½Π΅Ρ‡Π½ΡƒΡŽ Ρ€Π°Π·Π½ΠΈΡ†Ρƒ ΠΎΡ‚ Π­Π”Π‘, Ρ‡Ρ‚ΠΎ ΠΌΠΎΠΆΠ΅Ρ‚ Π±Ρ‹Ρ‚ΡŒ слоТной ΠΊΠΎΠ½Ρ†Π΅ΠΏΡ†ΠΈΠ΅ΠΉ для понимания учащимися.ΠŸΠΎΡΠΊΠΎΠ»ΡŒΠΊΡƒ люди становятся всС Π±ΠΎΠ»Π΅Π΅ зависимыми ΠΎΡ‚ элСктричСства, исслСдования, связанныС с ЭМП, Π²Π°ΠΆΠ½Ρ‹ для развития ΠΈ тСхничСского прогрСсса элСктричСства.

Π‘Π»Π΅Π΄ΡƒΡŽΡ‰ΠΈΠ΅ шаги

Π­Ρ‚ΠΈ ссылки ΠΏΡ€Π΅Π΄ΠΎΡΡ‚Π°Π²Π»ΡΡŽΡ‚ΡΡ Ρ‚ΠΎΠ»ΡŒΠΊΠΎ для удобства ΠΈ Π² ΠΈΠ½Ρ„ΠΎΡ€ΠΌΠ°Ρ†ΠΈΠΎΠ½Π½Ρ‹Ρ… цСлях; ΠΎΠ½ΠΈ Π½Π΅ ΠΎΠ·Π½Π°Ρ‡Π°ΡŽΡ‚ одобрСния ΠΈΠ»ΠΈ одобрСния БирмингСмским унивСрситСтом ΠΊΠ°ΠΊΠΎΠΉ-Π»ΠΈΠ±ΠΎ ΠΈΠ½Ρ„ΠΎΡ€ΠΌΠ°Ρ†ΠΈΠΈ, содСрТащСйся Π½Π° внСшнСм Π²Π΅Π±-сайтС. БирмингСмский унивСрситСт Π½Π΅ нСсСт отвСтствСнности Π·Π° Ρ‚ΠΎΡ‡Π½ΠΎΡΡ‚ΡŒ, Π·Π°ΠΊΠΎΠ½Π½ΠΎΡΡ‚ΡŒ ΠΈΠ»ΠΈ содСрТаниС внСшнСго сайта ΠΈΠ»ΠΈ ΠΏΠΎΡΠ»Π΅Π΄ΡƒΡŽΡ‰ΠΈΡ… ссылок.ΠŸΠΎΠΆΠ°Π»ΡƒΠΉΡΡ‚Π°, ΡΠ²ΡΠΆΠΈΡ‚Π΅ΡΡŒ с внСшним сайтом для получСния ΠΎΡ‚Π²Π΅Ρ‚ΠΎΠ² Π½Π° вопросы ΠΎΡ‚Π½ΠΎΡΠΈΡ‚Π΅Π»ΡŒΠ½ΠΎ Π΅Π³ΠΎ содСрТания.

9B: ЭлСктричСский Ρ‚ΠΎΠΊ, Π­Π”Π‘ ΠΈ Π·Π°ΠΊΠΎΠ½ Ома

Π’Π΅ΠΏΠ΅Ρ€ΡŒ ΠΌΡ‹ приступим ΠΊ ΠΈΠ·ΡƒΡ‡Π΅Π½ΠΈΡŽ элСктричСских Ρ†Π΅ΠΏΠ΅ΠΉ. ЦСпь – это Π·Π°ΠΌΠΊΠ½ΡƒΡ‚Ρ‹ΠΉ проводящий ΠΏΡƒΡ‚ΡŒ, ΠΏΠΎ ΠΊΠΎΡ‚ΠΎΡ€ΠΎΠΌΡƒ Ρ‚Π΅Ρ‡Π΅Ρ‚ заряд. Π’ схСмах заряд ΠΈΠ΄Π΅Ρ‚ пСтлями. Π‘ΠΊΠΎΡ€ΠΎΡΡ‚ΡŒ ΠΏΠΎΡ‚ΠΎΠΊΠ° заряда называСтся элСктричСским Ρ‚ΠΎΠΊΠΎΠΌ. Π‘Ρ…Π΅ΠΌΠ° состоит ΠΈΠ· элСмСнтов схСмы, соСдинСнных ΠΌΠ΅ΠΆΠ΄Ρƒ собой ΠΏΡ€ΠΎΠ²ΠΎΠ΄Π°ΠΌΠΈ. ΠšΠΎΠ½Π΄Π΅Π½ΡΠ°Ρ‚ΠΎΡ€ – это ΠΏΡ€ΠΈΠΌΠ΅Ρ€ элСмСнта схСмы, с ΠΊΠΎΡ‚ΠΎΡ€Ρ‹ΠΌ Π²Ρ‹ ΡƒΠΆΠ΅ Π·Π½Π°ΠΊΠΎΠΌΡ‹.Π’ этой Π³Π»Π°Π²Π΅ ΠΌΡ‹ прСдставим Π΅Ρ‰Π΅ нСсколько схСмных элСмСнтов. ΠŸΡ€ΠΈ Π°Π½Π°Π»ΠΈΠ·Π΅ схСм ΠΌΡ‹ рассматриваСм ΠΏΡ€ΠΎΠ²ΠΎΠ΄Π° ΠΊΠ°ΠΊ ΠΈΠ΄Π΅Π°Π»ΡŒΠ½Ρ‹Π΅ ΠΏΡ€ΠΎΠ²ΠΎΠ΄Π½ΠΈΠΊΠΈ, Π° элСмСнты схСмы ΠΊΠ°ΠΊ ΠΈΠ΄Π΅Π°Π»ΡŒΠ½Ρ‹Π΅ элСмСнты схСмы. Π‘Π»ΠΎΠΆΠ½ΠΎΡΡ‚ΡŒ схСм ΠΎΡ‡Π΅Π½ΡŒ Ρ€Π°Π·Π½ΠΎΠΎΠ±Ρ€Π°Π·Π½Π°. ΠšΠΎΠΌΠΏΡŒΡŽΡ‚Π΅Ρ€ – слоТная схСма. Π€ΠΎΠ½Π°Ρ€ΠΈΠΊ прСдставляСт собой ΠΏΡ€ΠΎΡΡ‚ΡƒΡŽ схСму.

Π­Π»Π΅ΠΌΠ΅Π½Ρ‚Ρ‹ схСмы, с ΠΊΠΎΡ‚ΠΎΡ€Ρ‹ΠΌΠΈ Π²Ρ‹ Π±ΡƒΠ΄Π΅Ρ‚Π΅ ΠΈΠΌΠ΅Ρ‚ΡŒ Π΄Π΅Π»ΠΎ Π² этом курсС, – это Π΄Π²ΡƒΡ…ΠΊΠΎΠ½Ρ‚Π°ΠΊΡ‚Π½Ρ‹Π΅ элСмСнты схСмы. БущСствуСт нСсколько Ρ€Π°Π·Π»ΠΈΡ‡Π½Ρ‹Ρ… Ρ‚ΠΈΠΏΠΎΠ² Π΄Π²ΡƒΡ…ΠΊΠΎΠ½Ρ‚Π°ΠΊΡ‚Π½Ρ‹Ρ… схСмных элСмСнтов, Π½ΠΎ всС ΠΎΠ½ΠΈ ΠΈΠΌΠ΅ΡŽΡ‚ Π½Π΅ΠΊΠΎΡ‚ΠΎΡ€Ρ‹Π΅ ΠΎΠ±Ρ‰ΠΈΠ΅ Ρ‡Π΅Ρ€Ρ‚Ρ‹.Π”Π²ΡƒΡ…ΠΊΠΎΠ½Ρ‚Π°ΠΊΡ‚Π½Ρ‹ΠΉ элСмСнт схСмы – это устройство с двумя ΠΊΠΎΠ½Ρ†Π°ΠΌΠΈ, ΠΊΠ°ΠΆΠ΄Ρ‹ΠΉ ΠΈΠ· ΠΊΠΎΡ‚ΠΎΡ€Ρ‹Ρ… являСтся ΠΏΡ€ΠΎΠ²ΠΎΠ΄Π½ΠΈΠΊΠΎΠΌ. Π”Π²Π° ΠΏΡ€ΠΎΠ²ΠΎΠ΄Π½ΠΈΠΊΠ° Π½Π°Π·Ρ‹Π²Π°ΡŽΡ‚ΡΡ ΠΊΠ»Π΅ΠΌΠΌΠ°ΠΌΠΈ. Π’Π΅Ρ€ΠΌΠΈΠ½Π°Π»Ρ‹ ΠΌΠΎΠ³ΡƒΡ‚ ΠΈΠΌΠ΅Ρ‚ΡŒ Ρ€Π°Π·Π½Ρ‹Π΅ Ρ„ΠΎΡ€ΠΌΡ‹. НСкоторыС ΠΈΠ· Π½ΠΈΡ… – ΠΏΡ€ΠΎΠ²ΠΎΠ΄Π°, Π½Π΅ΠΊΠΎΡ‚ΠΎΡ€Ρ‹Π΅ – мСталличСскиС пластины, Π½Π΅ΠΊΠΎΡ‚ΠΎΡ€Ρ‹Π΅ – мСталличСскиС ΠΊΠ½ΠΎΠΏΠΊΠΈ, Π° Π½Π΅ΠΊΠΎΡ‚ΠΎΡ€Ρ‹Π΅ – мСталличСскиС столбы. К ΠΊΠ»Π΅ΠΌΠΌΠ°ΠΌ ΠΏΠΎΠ΄ΠΊΠ»ΡŽΡ‡Π°ΡŽΡ‚ΡΡ ΠΏΡ€ΠΎΠ²ΠΎΠ΄Π°, Ρ‡Ρ‚ΠΎΠ±Ρ‹ ΡΠ΄Π΅Π»Π°Ρ‚ΡŒ элСмСнт схСмы Ρ‡Π°ΡΡ‚ΡŒΡŽ схСмы.

Π’Π°ΠΆΠ½Ρ‹ΠΌ Π΄Π²ΡƒΡ…ΠΊΠΎΠ½Ρ‚Π°ΠΊΡ‚Π½Ρ‹ΠΌ элСмСнтом схСмы являСтся мСсто располоТСния Π­Π”Π‘. Π’Ρ‹ ΠΌΠΎΠΆΠ΅Ρ‚Π΅ Π΄ΡƒΠΌΠ°Ρ‚ΡŒ ΠΎ сидСньС EMF ΠΊΠ°ΠΊ ΠΎΠ± идСальном аккумуляторС ΠΈΠ»ΠΈ ΠΊΠ°ΠΊ ΠΎΠ± идСальном источникС питания. Он ΠΏΠΎΠ΄Π΄Π΅Ρ€ΠΆΠΈΠ²Π°Π΅Ρ‚ ΠΏΠΎΡΡ‚ΠΎΡΠ½Π½ΡƒΡŽ Ρ€Π°Π·Π½ΠΎΡΡ‚ΡŒ ΠΏΠΎΡ‚Π΅Π½Ρ†ΠΈΠ°Π»ΠΎΠ² (a.ΠΊ.Π°. постоянноС напряТСниС) ΠΌΠ΅ΠΆΠ΄Ρƒ Π΅Π³ΠΎ Π²Ρ‹Π²ΠΎΠ΄Π°ΠΌΠΈ. Для прСдставлСния разности ΠΏΠΎΡ‚Π΅Π½Ρ†ΠΈΠ°Π»ΠΎΠ² ΠΈΡΠΏΠΎΠ»ΡŒΠ·ΡƒΠ΅Ρ‚ΡΡ Π»ΠΈΠ±ΠΎ имя константы \ (\ varepsilon \) (сцСнарий \ (E \)), Π»ΠΈΠ±ΠΎ имя константы \ (V \).

Π§Ρ‚ΠΎΠ±Ρ‹ Π΄ΠΎΡΡ‚ΠΈΡ‡ΡŒ разности ΠΏΠΎΡ‚Π΅Π½Ρ†ΠΈΠ°Π»ΠΎΠ² \ (E \) ΠΌΠ΅ΠΆΠ΄Ρƒ своими Π²Ρ‹Π²ΠΎΠ΄Π°ΠΌΠΈ, ΠΎΡ‡Π°Π³ Π­Π”Π‘, ΠΊΠΎΠ³Π΄Π° ΠΎΠ½ Π²ΠΏΠ΅Ρ€Π²Ρ‹Π΅ Π²ΠΎΠ·Π½ΠΈΠΊΠ°Π΅Ρ‚, Π΄ΠΎΠ»ΠΆΠ΅Π½ ΠΏΠ΅Ρ€Π΅ΠΌΠ΅Ρ‰Π°Ρ‚ΡŒ Π½Π΅ΠΊΠΎΡ‚ΠΎΡ€Ρ‹ΠΉ заряд (ΠΌΡ‹ рассматриваСм Π΄Π²ΠΈΠΆΠ΅Π½ΠΈΠ΅ заряда ΠΊΠ°ΠΊ Π΄Π²ΠΈΠΆΠ΅Π½ΠΈΠ΅ ΠΏΠΎΠ»ΠΎΠΆΠΈΡ‚Π΅Π»ΡŒΠ½ΠΎΠ³ΠΎ заряда) ΠΎΡ‚ ΠΎΠ΄Π½ΠΎΠ³ΠΎ Π²Ρ‹Π²ΠΎΠ΄Π° ΠΊ Π΄Ρ€ΡƒΠ³ΠΎΠΉ. «Один Π²Ρ‹Π²ΠΎΠ΄Β» остаСтся с чистым ΠΎΡ‚Ρ€ΠΈΡ†Π°Ρ‚Π΅Π»ΡŒΠ½Ρ‹ΠΌ зарядом, Π° Β«Π΄Ρ€ΡƒΠ³ΠΎΠΉΒ» ΠΏΡ€ΠΈΠΎΠ±Ρ€Π΅Ρ‚Π°Π΅Ρ‚ чистый ΠΏΠΎΠ»ΠΎΠΆΠΈΡ‚Π΅Π»ΡŒΠ½Ρ‹ΠΉ заряд.ΠœΠ΅ΡΡ‚ΠΎ Π­Π”Π‘ ΠΏΠ΅Ρ€Π΅ΠΌΠ΅Ρ‰Π°Π΅Ρ‚ заряд Π΄ΠΎ Ρ‚Π΅Ρ… ΠΏΠΎΡ€, ΠΏΠΎΠΊΠ° ΠΏΠΎΠ»ΠΎΠΆΠΈΡ‚Π΅Π»ΡŒΠ½Ρ‹ΠΉ Π²Ρ‹Π²ΠΎΠ΄ Π½Π΅ Π±ΡƒΠ΄Π΅Ρ‚ ΠΈΠΌΠ΅Ρ‚ΡŒ ΠΏΠΎΡ‚Π΅Π½Ρ†ΠΈΠ°Π» \ (E \) Π²Ρ‹ΡˆΠ΅, Ρ‡Π΅ΠΌ ΠΎΡ‚Ρ€ΠΈΡ†Π°Ρ‚Π΅Π»ΡŒΠ½Ρ‹ΠΉ Π²Ρ‹Π²ΠΎΠ΄. ΠžΠ±Ρ€Π°Ρ‚ΠΈΡ‚Π΅ Π²Π½ΠΈΠΌΠ°Π½ΠΈΠ΅, Ρ‡Ρ‚ΠΎ мСсто Π­Π”Π‘ Π½Π΅ ΠΏΡ€ΠΎΠΈΠ·Π²ΠΎΠ΄ΠΈΡ‚ заряда; ΠΎΠ½ просто Π²Ρ‹Ρ‚Π°Π»ΠΊΠΈΠ²Π°Π΅Ρ‚ ΡΡƒΡ‰Π΅ΡΡ‚Π²ΡƒΡŽΡ‰ΠΈΠΉ заряд. Если Π²Ρ‹ ΠΏΠΎΠ΄ΠΊΠ»ΡŽΡ‡ΠΈΡ‚Π΅ ΠΈΠ·ΠΎΠ»ΠΈΡ€ΠΎΠ²Π°Π½Π½Ρ‹ΠΉ ΠΏΡ€ΠΎΠ²ΠΎΠ΄ ΠΊ ΠΏΠΎΠ»ΠΎΠΆΠΈΡ‚Π΅Π»ΡŒΠ½ΠΎΠΌΡƒ Π²Ρ‹Π²ΠΎΠ΄Ρƒ, Ρ‚ΠΎ ΠΎΠ½ Π±ΡƒΠ΄Π΅Ρ‚ ΠΈΠΌΠ΅Ρ‚ΡŒ Ρ‚ΠΎΡ‚ ΠΆΠ΅ ΠΏΠΎΡ‚Π΅Π½Ρ†ΠΈΠ°Π», Ρ‡Ρ‚ΠΎ ΠΈ ΠΏΠΎΠ»ΠΎΠΆΠΈΡ‚Π΅Π»ΡŒΠ½Ρ‹ΠΉ Π²Ρ‹Π²ΠΎΠ΄, ΠΈ, ΠΏΠΎΡΠΊΠΎΠ»ΡŒΠΊΡƒ заряд Π½Π° ΠΏΠΎΠ»ΠΎΠΆΠΈΡ‚Π΅Π»ΡŒΠ½ΠΎΠΌ Π²Ρ‹Π²ΠΎΠ΄Π΅ Π±ΡƒΠ΄Π΅Ρ‚ Ρ€Π°ΡΠΏΡ€ΠΎΡΡ‚Ρ€Π°Π½ΡΡ‚ΡŒΡΡ ΠΏΠΎ ΠΏΡ€ΠΎΠ²ΠΎΠ΄Ρƒ, Π³Π½Π΅Π·Π΄ΠΎ Π­Π”Π‘ Π±ΡƒΠ΄Π΅Ρ‚ ΠΈΠΌΠ΅Ρ‚ΡŒ Ρ‚Π°ΠΊΠΎΠΉ ΠΆΠ΅ ΠΏΠΎΡ‚Π΅Π½Ρ†ΠΈΠ°Π», ΠΊΠ°ΠΊ ΠΈ ΠΏΠΎΠ»ΠΎΠΆΠΈΡ‚Π΅Π»ΡŒΠ½Ρ‹ΠΉ Π²Ρ‹Π²ΠΎΠ΄. ΠΏΠ΅Ρ€Π΅ΠΌΠ΅ΡΡ‚ΠΈΡ‚ΡŒ Π΅Ρ‰Π΅ Π½Π΅ΠΌΠ½ΠΎΠ³ΠΎ заряда с ΠΊΠ»Π΅ΠΌΠΌΡ‹ с Π±ΠΎΠ»Π΅Π΅ Π½ΠΈΠ·ΠΊΠΈΠΌ ΠΏΠΎΡ‚Π΅Π½Ρ†ΠΈΠ°Π»ΠΎΠΌ для поддСрТания разности ΠΏΠΎΡ‚Π΅Π½Ρ†ΠΈΠ°Π»ΠΎΠ².{-12} Π‘ \)). ΠšΡ€ΠΎΠΌΠ΅ Ρ‚ΠΎΠ³ΠΎ, Π½Π°ΠΊΠΎΠΏΠ»Π΅Π½ΠΈΠ΅ заряда происходит ΠΏΠΎΡ‡Ρ‚ΠΈ ΠΌΠ³Π½ΠΎΠ²Π΅Π½Π½ΠΎ, поэтому ΠΊ Ρ‚ΠΎΠΌΡƒ Π²Ρ€Π΅ΠΌΠ΅Π½ΠΈ, ΠΊΠΎΠ³Π΄Π° Π²Ρ‹ Π·Π°ΠΊΠ°Π½Ρ‡ΠΈΠ²Π°Π΅Ρ‚Π΅ ΠΏΠΎΠ΄ΠΊΠ»ΡŽΡ‡Π°Ρ‚ΡŒ ΠΏΡ€ΠΎΠ²ΠΎΠ΄ ΠΊ ΠΊΠ»Π΅ΠΌΠΌΠ΅, этот ΠΏΡ€ΠΎΠ²ΠΎΠ΄ ΡƒΠΆΠ΅ ΠΈΠΌΠ΅Π΅Ρ‚ заряд, ΠΎ ΠΊΠΎΡ‚ΠΎΡ€ΠΎΠΌ ΠΌΡ‹ Π³ΠΎΠ²ΠΎΡ€ΠΈΠΌ. Π’ ΠΎΠ±Ρ‰Π΅ΠΌ, ΠΌΡ‹ Π½Π΅ Π·Π½Π°Π΅ΠΌ, сколько заряда находится Π½Π° ΠΏΠΎΠ»ΠΎΠΆΠΈΡ‚Π΅Π»ΡŒΠ½ΠΎΠΉ ΠΊΠ»Π΅ΠΌΠΌΠ΅ ΠΈ ΠΊΠ°ΠΊΠΎΠΉ ΠΏΡ€ΠΎΠ²ΠΎΠ΄ ΠΊ Π½Π΅ΠΉ ΠΌΠΎΠΆΠ΅Ρ‚ Π±Ρ‹Ρ‚ΡŒ ΠΏΠΎΠ΄ΠΊΠ»ΡŽΡ‡Π΅Π½, ΠΈ Π½Π°ΠΌ всС Ρ€Π°Π²Π½ΠΎ. Π­Ρ‚ΠΎ Π½ΠΈΡ‡Ρ‚ΠΎΠΆΠ½ΠΎ ΠΌΠ°Π»ΠΎ. Но этого достаточно, Ρ‡Ρ‚ΠΎΠ±Ρ‹ Ρ€Π°Π·Π½ΠΎΡΡ‚ΡŒ ΠΏΠΎΡ‚Π΅Π½Ρ†ΠΈΠ°Π»ΠΎΠ² ΠΌΠ΅ΠΆΠ΄Ρƒ Π²Ρ‹Π²ΠΎΠ΄Π°ΠΌΠΈ Π±Ρ‹Π»Π° Π½ΠΎΠΌΠΈΠ½Π°Π»ΡŒΠ½Ρ‹ΠΌ напряТСниСм мСста возникновСния Π­Π”Π‘.

Как Π²Ρ‹ ΠΏΠΎΠΌΠ½ΠΈΡ‚Π΅, элСктричСский ΠΏΠΎΡ‚Π΅Π½Ρ†ΠΈΠ°Π» – это Ρ‚ΠΎ, Ρ‡Ρ‚ΠΎ ΠΈΡΠΏΠΎΠ»ΡŒΠ·ΡƒΠ΅Ρ‚ΡΡ для характСристики элСктричСского поля.Вызывая Ρ€Π°Π·Π½ΠΎΡΡ‚ΡŒ ΠΏΠΎΡ‚Π΅Π½Ρ†ΠΈΠ°Π»ΠΎΠ² ΠΌΠ΅ΠΆΠ΄Ρƒ Π΅Π³ΠΎ Π²Ρ‹Π²ΠΎΠ΄Π°ΠΌΠΈ ΠΈ ΠΌΠ΅ΠΆΠ΄Ρƒ любой ΠΏΠ°Ρ€ΠΎΠΉ ΠΏΡ€ΠΎΠ²ΠΎΠ΄ΠΎΠ², ΠΊΠΎΡ‚ΠΎΡ€Ρ‹Π΅ ΠΌΠΎΠ³ΡƒΡ‚ Π±Ρ‹Ρ‚ΡŒ ΠΏΠΎΠ΄ΠΊΠ»ΡŽΡ‡Π΅Π½Ρ‹ ΠΊ Π΅Π³ΠΎ Π²Ρ‹Π²ΠΎΠ΄Π°ΠΌ, мСсто дСйствия Π­Π”Π‘ создаСт элСктричСскоС ΠΏΠΎΠ»Π΅. ЭлСктричСскоС ΠΏΠΎΠ»Π΅ зависит ΠΎΡ‚ располоТСния ΠΏΡ€ΠΎΠ²ΠΎΠ΄ΠΎΠ², ΠΊΠΎΡ‚ΠΎΡ€Ρ‹Π΅ ΠΏΠΎΠ΄ΠΊΠ»ΡŽΡ‡Π°ΡŽΡ‚ΡΡ ΠΊ ΠΊΠ»Π΅ΠΌΠΌΠ°ΠΌ Π³Π½Π΅Π·Π΄Π° Π­Π”Π‘. ЭлСктричСскоС ΠΏΠΎΠ»Π΅ – Π΅Ρ‰Π΅ ΠΎΠ΄Π½Π° Π²Π΅Π»ΠΈΡ‡ΠΈΠ½Π°, ΠΊΠΎΡ‚ΠΎΡ€ΡƒΡŽ ΠΌΡ‹ Ρ€Π΅Π΄ΠΊΠΎ обсуТдаСм ΠΏΡ€ΠΈ Π°Π½Π°Π»ΠΈΠ·Π΅ Ρ†Π΅ΠΏΠ΅ΠΉ. ΠžΠ±Ρ‹Ρ‡Π½ΠΎ ΠΌΡ‹ ΠΌΠΎΠΆΠ΅ΠΌ Π²Ρ‹ΡΡΠ½ΠΈΡ‚ΡŒ, Ρ‡Ρ‚ΠΎ Π½Π°ΠΌ Π½ΡƒΠΆΠ½ΠΎ ΡƒΠ·Π½Π°Ρ‚ΡŒ, ΠΏΠΎ Π·Π½Π°Ρ‡Π΅Π½ΠΈΡŽ разности ΠΏΠΎΡ‚Π΅Π½Ρ†ΠΈΠ°Π»ΠΎΠ² E, ΠΊΠΎΡ‚ΠΎΡ€ΠΎΠ΅ мСсто Π­Π”Π‘ ΠΏΠΎΠ΄Π΄Π΅Ρ€ΠΆΠΈΠ²Π°Π΅Ρ‚ ΠΌΠ΅ΠΆΠ΄Ρƒ своими Ρ‚Π΅Ρ€ΠΌΠΈΠ½Π°Π»Π°ΠΌΠΈ.Но элСктричСскоС ΠΏΠΎΠ»Π΅ Π΄Π΅ΠΉΡΡ‚Π²ΠΈΡ‚Π΅Π»ΡŒΠ½ΠΎ сущСствуСт, ΠΈ Π² цСпях элСктричСскоС ΠΏΠΎΠ»Π΅ заряда Π½Π° ΠΏΡ€ΠΎΠ²ΠΎΠ΄Π°Ρ…, ΠΏΠΎΠ΄ΠΊΠ»ΡŽΡ‡Π΅Π½Π½Ρ‹Ρ… ΠΊ мСсту возникновСния Π­Π”Π‘, – это Ρ‚ΠΎ, Ρ‡Ρ‚ΠΎ заставляСт заряд Ρ‚Π΅Ρ‡ΡŒ Π² Ρ†Π΅ΠΏΠΈ, Π° ΠΏΠΎΡ‚ΠΎΠΊ заряда Π² Ρ†Π΅ΠΏΠΈ – огромная Ρ‡Π°ΡΡ‚ΡŒ Ρ‚ΠΎΠ³ΠΎ, Ρ‡Ρ‚ΠΎ схСма – это всС ΠΎ.

Π˜ΡΠΏΠΎΠ»ΡŒΠ·ΡƒΠ΅ΠΌ символ

для прСдставлСния мСста располоТСния Π­Π”Π‘ Π½Π° ΠΏΡ€ΠΈΠ½Ρ†ΠΈΠΏΠΈΠ°Π»ΡŒΠ½ΠΎΠΉ схСмС (Ρ‚Π°ΠΊΠΆΠ΅ извСстной ΠΊΠ°ΠΊ схСматичСская Π΄ΠΈΠ°Π³Ρ€Π°ΠΌΠΌΠ° Ρ†Π΅ΠΏΠΈ), Π³Π΄Π΅ Π΄Π²Π° ΠΊΠΎΠ»Π»ΠΈΠ½Π΅Π°Ρ€Π½Ρ‹Ρ… ΠΎΡ‚Ρ€Π΅Π·ΠΊΠ° Π»ΠΈΠ½ΠΈΠΈ ΠΏΡ€Π΅Π΄ΡΡ‚Π°Π²Π»ΡΡŽΡ‚ Π²Ρ‹Π²ΠΎΠ΄Ρ‹ мСста располоТСния Π­Π”Π‘, ΠΏΡ€ΠΈΡ‡Π΅ΠΌ Ρ‚ΠΎΡ‚, ΠΊΠΎΡ‚ΠΎΡ€Ρ‹ΠΉ ΠΏΠΎΠ΄ΠΊΠ»ΡŽΡ‡Π΅Π½ ΠΊ Π±ΠΎΠ»Π΅Π΅ ΠΊΠΎΡ€ΠΎΡ‚ΠΊΠΎΠΌΡƒ ΠΈΠ· ΠΏΠ°Ρ€Π°Π»Π»Π΅Π»ΡŒΠ½Ρ‹Ρ… ΠΎΡ‚Ρ€Π΅Π·ΠΊΠΎΠ² прямой, являСтся ΠΎΡ‚Ρ€ΠΈΡ†Π°Ρ‚Π΅Π»ΡŒΠ½Ρ‹ΠΌ, Π½ΠΈΠ·ΠΊΠΎΠ²ΠΎΠ»ΡŒΡ‚Π½Ρ‹ΠΉ, Ρ‚Π΅Ρ€ΠΌΠΈΠ½Π°Π»ΡŒΠ½Ρ‹ΠΉ; Π° Ρ‚Π°ΠΊΠΆΠ΅; Ρ‚ΠΎΡ‚, ΠΊΠΎΡ‚ΠΎΡ€Ρ‹ΠΉ ΠΏΠΎΠ΄ΠΊΠ»ΡŽΡ‡Π΅Π½ ΠΊ Π±ΠΎΠ»Π΅Π΅ Π΄Π»ΠΈΠ½Π½ΠΎΠΌΡƒ ΠΈΠ· ΠΏΠ°Ρ€Π°Π»Π»Π΅Π»ΡŒΠ½Ρ‹Ρ… сСгмСнтов Π»ΠΈΠ½ΠΈΠΈ, являСтся ΠΏΠΎΠ»ΠΎΠΆΠΈΡ‚Π΅Π»ΡŒΠ½Ρ‹ΠΌ Π²Ρ‹Π²ΠΎΠ΄ΠΎΠΌ с Π±ΠΎΠ»Π΅Π΅ высоким ΠΏΠΎΡ‚Π΅Π½Ρ†ΠΈΠ°Π»ΠΎΠΌ.

Π”Ρ€ΡƒΠ³ΠΎΠΉ элСмСнт схСмы, ΠΊΠΎΡ‚ΠΎΡ€Ρ‹ΠΉ я Ρ…ΠΎΡ‡Ρƒ ΠΏΡ€Π΅Π΄ΡΡ‚Π°Π²ΠΈΡ‚ΡŒ Π² этой Π³Π»Π°Π²Π΅, – это рСзистор. РСзистор – ΠΏΠ»ΠΎΡ…ΠΎΠΉ ΠΏΡ€ΠΎΠ²ΠΎΠ΄Π½ΠΈΠΊ. Π‘ΠΎΠΏΡ€ΠΎΡ‚ΠΈΠ²Π»Π΅Π½ΠΈΠ΅ рСзистора являСтся ΠΌΠ΅Ρ€ΠΎΠΉ Ρ‚ΠΎΠ³ΠΎ, насколько ΠΏΠ»ΠΎΡ…ΠΎΠΉ ΠΏΡ€ΠΎΠ²ΠΎΠ΄Π½ΠΈΠΊ являСтся рСзистор. Π§Π΅ΠΌ большС Π·Π½Π°Ρ‡Π΅Π½ΠΈΠ΅ сопротивлСния, Ρ‚Π΅ΠΌ Ρ…ΡƒΠΆΠ΅ элСмСнт схСмы пропускаСт заряд Ρ‡Π΅Ρ€Π΅Π· сСбя. РСзисторы Π±Ρ‹Π²Π°ΡŽΡ‚ Ρ€Π°Π·Π½Ρ‹Ρ… Ρ„ΠΎΡ€ΠΌ. ΠΠΈΡ‚ΡŒ Π½Π°ΠΊΠ°Π»Π° Π»Π°ΠΌΠΏΠΎΡ‡ΠΊΠΈ – это рСзистор. Π­Π»Π΅ΠΌΠ΅Π½Ρ‚ тостСра (Ρ‡Π°ΡΡ‚ΡŒ, которая свСтится красным, ΠΊΠΎΠ³Π΄Π° тостСр Π²ΠΊΠ»ΡŽΡ‡Π΅Π½) – это рСзистор. Π›ΡŽΠ΄ΠΈ ΠΈΠ·Π³ΠΎΡ‚Π°Π²Π»ΠΈΠ²Π°ΡŽΡ‚ нСбольшиС кСрамичСскиС Ρ†ΠΈΠ»ΠΈΠ½Π΄Ρ€Ρ‹ (с ΡƒΠ³Π»Π΅Ρ€ΠΎΠ΄Π½Ρ‹ΠΌ ΠΏΠΎΠΊΡ€Ρ‹Ρ‚ΠΈΠ΅ΠΌ ΠΈ ΠΏΡ€ΠΎΠ²ΠΎΠ»ΠΎΠΊΠΎΠΉ, Ρ‚ΠΎΡ€Ρ‡Π°Ρ‰Π΅ΠΉ Π½Π° ΠΊΠ°ΠΆΠ΄ΠΎΠΌ ΠΊΠΎΠ½Ρ†Π΅), Ρ‡Ρ‚ΠΎΠ±Ρ‹ ΠΈΠΌΠ΅Ρ‚ΡŒ ΠΎΠΏΡ€Π΅Π΄Π΅Π»Π΅Π½Π½Ρ‹Π΅ значСния сопротивлСния.Π£ ΠΊΠ°ΠΆΠ΄ΠΎΠ³ΠΎ ΠΈΠ· Π½ΠΈΡ… Π΅ΡΡ‚ΡŒ Π·Π½Π°Ρ‡Π΅Π½ΠΈΠ΅ сопротивлСния, ΡƒΠΊΠ°Π·Π°Π½Π½ΠΎΠ΅ Π½Π° самом рСзисторС. Π‘ΠΈΠΌΠ²ΠΎΠ»

ΠΈΡΠΏΠΎΠ»ΡŒΠ·ΡƒΠ΅Ρ‚ΡΡ для обозначСния рСзистора Π½Π° ΠΏΡ€ΠΈΠ½Ρ†ΠΈΠΏΠΈΠ°Π»ΡŒΠ½ΠΎΠΉ схСмС. Π‘ΠΈΠΌΠ²ΠΎΠ» R ΠΎΠ±Ρ‹Ρ‡Π½ΠΎ ΠΈΡΠΏΠΎΠ»ΡŒΠ·ΡƒΠ΅Ρ‚ΡΡ для обозначСния сопротивлСния рСзистора.

Π’Π΅ΠΏΠ΅Ρ€ΡŒ ΠΌΡ‹ Π³ΠΎΡ‚ΠΎΠ²Ρ‹ Ρ€Π°ΡΡΠΌΠΎΡ‚Ρ€Π΅Ρ‚ΡŒ ΡΠ»Π΅Π΄ΡƒΡŽΡ‰ΡƒΡŽ ΠΏΡ€ΠΎΡΡ‚ΡƒΡŽ схСму:

Π’ΠΎΡ‚ ΠΈ снова Π±Π΅Π· ΡΡ‚ΠΎΠ»ΡŒΠΊΠΈΡ… этикСток:

Π’Π΅Ρ€Ρ…Π½ΠΈΠΉ ΠΏΡ€ΠΎΠ²ΠΎΠ΄ (ΠΏΡ€ΠΎΠ²ΠΎΠ΄Π½ΠΈΠΊ) ΠΈΠΌΠ΅Π΅Ρ‚ ΠΎΠ΄Π½ΠΎ Π·Π½Π°Ρ‡Π΅Π½ΠΈΠ΅ элСктричСского ΠΏΠΎΡ‚Π΅Π½Ρ†ΠΈΠ°Π»Π° (Π½Π°Π·ΠΎΠ²ΠΈΡ‚Π΅ Π΅Π³ΠΎ \ (\ varphi_ {HI} \)), Π° Π½ΠΈΠΆΠ½ΠΈΠΉ ΠΏΡ€ΠΎΠ²ΠΎΠ΄ ΠΈΠΌΠ΅Π΅Ρ‚ Π΄Ρ€ΡƒΠ³ΠΎΠ΅ Π·Π½Π°Ρ‡Π΅Π½ΠΈΠ΅ элСктричСского ΠΏΠΎΡ‚Π΅Π½Ρ†ΠΈΠ°Π»Π° (Π½Π°Π·ΠΎΠ²ΠΈΡ‚Π΅ Π΅Π³ΠΎ \ (\ varphi_ {LOW} \)), Π½Π°ΠΏΡ€ΠΈΠΌΠ΅Ρ€ Ρ‡Ρ‚ΠΎ Ρ€Π°Π·Π½ΠΈΡ†Π° \ (\ space \ varphi_ {HI} – \ varphi_ {LOW} \ space \) Ρ€Π°Π²Π½Π° \ (\ space \ varepsilon \).

\ [\ varphi_ {HI} – \ varphi_ {LOW} = \ varepsilon \]

Π§Ρ‚ΠΎΠ±Ρ‹ ΠΏΠΎΠ΄Π΄Π΅Ρ€ΠΆΠΈΠ²Π°Ρ‚ΡŒ Ρ€Π°Π·Π½ΠΎΡΡ‚ΡŒ ΠΏΠΎΡ‚Π΅Π½Ρ†ΠΈΠ°Π»ΠΎΠ² \ (\ varepsilon \) ΠΌΠ΅ΠΆΠ΄Ρƒ двумя ΠΏΡ€ΠΎΠ²ΠΎΠ΄Π½ΠΈΠΊΠ°ΠΌΠΈ, мСсто располоТСния Π­Π”Π‘ Π²Ρ‹Π·Ρ‹Π²Π°Π΅Ρ‚ появлСниС нСбольшого количСства ΠΏΠΎΠ»ΠΎΠΆΠΈΡ‚Π΅Π»ΡŒΠ½ΠΎΠ³ΠΎ заряда Π½Π° Π²Π΅Ρ€Ρ…Π½Π΅ΠΌ ΠΏΡ€ΠΎΠ²ΠΎΠ΄Π΅ ΠΈ Ρ‚Π°ΠΊΠΎΠ΅ ΠΆΠ΅ количСство ΠΎΡ‚Ρ€ΠΈΡ†Π°Ρ‚Π΅Π»ΡŒΠ½ΠΎΠ³ΠΎ заряда Π½Π° Π½ΠΈΠΆΠ½Π΅ΠΌ ΠΏΡ€ΠΎΠ²ΠΎΠ΄Π΅. Π­Ρ‚ΠΎ Ρ€Π°Π·Π΄Π΅Π»Π΅Π½ΠΈΠ΅ зарядов Π²Ρ‹Π·Ρ‹Π²Π°Π΅Ρ‚ элСктричСскоС ΠΏΠΎΠ»Π΅ Π² рСзисторС.

(ΠœΡ‹ ΠΏΡ€ΠΎΠ²ΠΎΠ΄ΠΈΠΌ этот Π°Ρ€Π³ΡƒΠΌΠ΅Π½Ρ‚ Π² ΠΌΠΎΠ΄Π΅Π»ΠΈ ΠΏΠΎΠ»ΠΎΠΆΠΈΡ‚Π΅Π»ΡŒΠ½ΠΎΠ³ΠΎ носитСля заряда. Π₯отя это Π½Π΅ ΠΈΠΌΠ΅Π΅Ρ‚ значСния для схСмы, Π½Π° самом Π΄Π΅Π»Π΅ это ΠΎΡ‚Ρ€ΠΈΡ†Π°Ρ‚Π΅Π»ΡŒΠ½ΠΎ заряТСнныС частицы, двиТущиСся Π² ΠΏΡ€ΠΎΡ‚ΠΈΠ²ΠΎΠΏΠΎΠ»ΠΎΠΆΠ½ΠΎΠΌ Π½Π°ΠΏΡ€Π°Π²Π»Π΅Π½ΠΈΠΈ.Π­Ρ„Ρ„Π΅ΠΊΡ‚ Ρ‚ΠΎΡ‚ ΠΆΠ΅.)

Π’Π°ΠΆΠ½ΠΎ ΠΏΠΎΠ½ΠΈΠΌΠ°Ρ‚ΡŒ, Ρ‡Ρ‚ΠΎ каТдая Ρ‡Π°ΡΡ‚ΡŒ Ρ†Π΅ΠΏΠΈ ΠΏΠ΅Ρ€Π΅ΠΏΠΎΠ»Π½Π΅Π½Π° ΠΎΠ±ΠΎΠΈΠΌΠΈ Π²ΠΈΠ΄Π°ΠΌΠΈ заряда. ΠŸΡ€ΠΎΠ²ΠΎΠ΄, рСзистор, всС нСвСроятно Π·Π°Π±ΠΈΡ‚ΠΎ ΠΊΠ°ΠΊ ΠΏΠΎΠ»ΠΎΠΆΠΈΡ‚Π΅Π»ΡŒΠ½Ρ‹ΠΌ, Ρ‚Π°ΠΊ ΠΈ ΠΎΡ‚Ρ€ΠΈΡ†Π°Ρ‚Π΅Π»ΡŒΠ½Ρ‹ΠΌ зарядом. Один Π²ΠΈΠ΄ заряда ΠΌΠΎΠΆΠ΅Ρ‚ Π΄Π²ΠΈΠ³Π°Ρ‚ΡŒΡΡ Π½Π° Ρ„ΠΎΠ½Π΅ Π΄Ρ€ΡƒΠ³ΠΎΠ³ΠΎ. Π’Π΅ΠΏΠ΅Ρ€ΡŒ элСктричСскоС ΠΏΠΎΠ»Π΅ Π² рСзисторС Ρ‚ΠΎΠ»ΠΊΠ°Π΅Ρ‚ ΠΏΠΎΠ»ΠΎΠΆΠΈΡ‚Π΅Π»ΡŒΠ½Ρ‹ΠΉ заряд Π² рСзисторС Π² Π½Π°ΠΏΡ€Π°Π²Π»Π΅Π½ΠΈΠΈ ΠΎΡ‚ Π²Ρ‹Π²ΠΎΠ΄Π° с Π±ΠΎΠ»Π΅Π΅ высоким ΠΏΠΎΡ‚Π΅Π½Ρ†ΠΈΠ°Π»ΠΎΠΌ ΠΊ ​​выводу с Π±ΠΎΠ»Π΅Π΅ Π½ΠΈΠ·ΠΊΠΈΠΌ ΠΏΠΎΡ‚Π΅Π½Ρ†ΠΈΠ°Π»ΠΎΠΌ.

НаправлСниС ΠΏΠΎΠ»ΠΎΠΆΠΈΡ‚Π΅Π»ΡŒΠ½ΠΎΠ³ΠΎ заряда Π½Π° ΠΏΡ€ΠΎΠ²ΠΎΠ΄ с Π±ΠΎΠ»Π΅Π΅ Π½ΠΈΠ·ΠΊΠΈΠΌ ΠΏΠΎΡ‚Π΅Π½Ρ†ΠΈΠ°Π»ΠΎΠΌ ΠΏΡ€ΠΈΠ²Π΅Π΄Π΅Ρ‚ ΠΊ ΡƒΠ²Π΅Π»ΠΈΡ‡Π΅Π½ΠΈΡŽ ΠΏΠΎΡ‚Π΅Π½Ρ†ΠΈΠ°Π»Π° ΠΏΡ€ΠΎΠ²ΠΎΠ΄Π° с Π±ΠΎΠ»Π΅Π΅ Π½ΠΈΠ·ΠΊΠΈΠΌ ΠΏΠΎΡ‚Π΅Π½Ρ†ΠΈΠ°Π»ΠΎΠΌ ΠΈ оставит Π²Π΅Ρ€Ρ…Π½ΠΈΠΉ ΠΊΠΎΠ½Π΅Ρ† рСзистора с ΠΎΡ‚Ρ€ΠΈΡ†Π°Ρ‚Π΅Π»ΡŒΠ½Ρ‹ΠΌ зарядом.Π― Π³ΠΎΠ²ΠΎΡ€ΡŽ Β«Π±Ρ‹Π» Π±Ρ‹Β», ΠΏΠΎΡ‚ΠΎΠΌΡƒ Ρ‡Ρ‚ΠΎ любая тСндСнция ΠΊ измСнСнию ΠΎΡ‚Π½ΠΎΡΠΈΡ‚Π΅Π»ΡŒΠ½ΠΎΠ³ΠΎ ΠΏΠΎΡ‚Π΅Π½Ρ†ΠΈΠ°Π»Π° Π΄Π²ΡƒΡ… ΠΏΡ€ΠΎΠ²ΠΎΠ΄ΠΎΠ² Π½Π΅ΠΌΠ΅Π΄Π»Π΅Π½Π½ΠΎ компСнсируСтся мСстом располоТСния Π­Π”Π‘. ΠŸΠΎΠΌΠ½ΠΈΡ‚Π΅, Ρ‡Ρ‚ΠΎ это Ρ‚ΠΎ, Ρ‡Ρ‚ΠΎ Π΄Π΅Π»Π°Π΅Ρ‚ сидСньС Π­Π”Π‘, ΠΎΠ½ΠΎ ΠΏΠΎΠ΄Π΄Π΅Ρ€ΠΆΠΈΠ²Π°Π΅Ρ‚ ΠΏΠΎΡΡ‚ΠΎΡΠ½Π½ΡƒΡŽ Ρ€Π°Π·Π½ΠΎΡΡ‚ΡŒ ΠΏΠΎΡ‚Π΅Π½Ρ†ΠΈΠ°Π»ΠΎΠ² ΠΌΠ΅ΠΆΠ΄Ρƒ ΠΏΡ€ΠΎΠ²ΠΎΠ΄Π°ΠΌΠΈ. Для этого Π² рассматриваСмом случаС Π³Π½Π΅Π·Π΄ΠΎ Π­Π”Π‘ Π΄ΠΎΠ»ΠΆΠ½ΠΎ Π²Ρ‹Ρ‚ΡΠ³ΠΈΠ²Π°Ρ‚ΡŒ ΠΏΠΎΠ»ΠΎΠΆΠΈΡ‚Π΅Π»ΡŒΠ½Ρ‹Π΅ заряды ΠΈΠ· ΠΏΡ€ΠΎΠ²ΠΎΠ΄Π° с Π±ΠΎΠ»Π΅Π΅ Π½ΠΈΠ·ΠΊΠΈΠΌ ΠΏΠΎΡ‚Π΅Π½Ρ†ΠΈΠ°Π»ΠΎΠΌ ΠΈ ΠΏΠΎΠ΄Ρ‚Π°Π»ΠΊΠΈΠ²Π°Ρ‚ΡŒ ΠΈΡ… ΠΊ ΠΏΡ€ΠΎΠ²ΠΎΠ΄Ρƒ с Π±ΠΎΠ»Π΅Π΅ высоким ΠΏΠΎΡ‚Π΅Π½Ρ†ΠΈΠ°Π»ΠΎΠΌ. ΠšΡ€ΠΎΠΌΠ΅ Ρ‚ΠΎΠ³ΠΎ, любая тСндСнция Π²Π΅Ρ€Ρ…Π½Π΅Π³ΠΎ ΠΊΠΎΠ½Ρ†Π° рСзистора ΡΡ‚Π°Π½ΠΎΠ²ΠΈΡ‚ΡŒΡΡ ΠΎΡ‚Ρ€ΠΈΡ†Π°Ρ‚Π΅Π»ΡŒΠ½Ρ‹ΠΌ сразу ΠΆΠ΅ ΠΏΡ€ΠΈΠ²ΠΎΠ΄ΠΈΡ‚ ΠΊ силС притяТСния Π½Π° ΠΏΠΎΠ»ΠΎΠΆΠΈΡ‚Π΅Π»ΡŒΠ½Ρ‹ΠΉ заряд Π² ΠΏΡ€ΠΎΠ²ΠΎΠ΄Π΅ с Π±ΠΎΠ»Π΅Π΅ высоким ΠΏΠΎΡ‚Π΅Π½Ρ†ΠΈΠ°Π»ΠΎΠΌ.Π­Ρ‚ΠΎ заставляСт этот ΠΏΠΎΠ»ΠΎΠΆΠΈΡ‚Π΅Π»ΡŒΠ½Ρ‹ΠΉ заряд ΠΏΠ΅Ρ€Π΅ΠΌΠ΅Ρ‰Π°Ρ‚ΡŒΡΡ Π²Π½ΠΈΠ· Π² рСзистор вмСсто заряда, ΠΊΠΎΡ‚ΠΎΡ€Ρ‹ΠΉ Ρ‚ΠΎΠ»ΡŒΠΊΠΎ Ρ‡Ρ‚ΠΎ пСрСмСщался вдоль рСзистора ΠΊ ΠΏΡ€ΠΎΠ²ΠΎΠ΄Ρƒ с Π±ΠΎΠ»Π΅Π΅ Π½ΠΈΠ·ΠΊΠΈΠΌ ΠΏΠΎΡ‚Π΅Π½Ρ†ΠΈΠ°Π»ΠΎΠΌ. Чистый эффСкт – это Π½Π΅ΠΏΡ€Π΅Ρ€Ρ‹Π²Π½ΠΎΠ΅ Π΄Π²ΠΈΠΆΠ΅Π½ΠΈΠ΅ заряда ΠΏΠΎ часовой стрСлкС ΠΏΠΎ ΠΏΠ΅Ρ‚Π»Π΅, ΠΊΠ°ΠΊ ΠΌΡ‹ Π²ΠΈΠ΄ΠΈΠΌ Π½Π° Π΄ΠΈΠ°Π³Ρ€Π°ΠΌΠΌΠ΅, ΠΏΡ€ΠΈ этом чистоС количСство заряда Π² любом ΠΊΠΎΡ€ΠΎΡ‚ΠΊΠΎΠΌ участкС Ρ†Π΅ΠΏΠΈ Π½ΠΈΠΊΠΎΠ³Π΄Π° Π½Π΅ мСняСтся. Π’Ρ‹Π±Π΅Ρ€ΠΈΡ‚Π΅ мСсто Π² любом мСстС трассы. Π’Π°ΠΊ ΠΆΠ΅ быстро, ΠΊΠ°ΠΊ ΠΏΠΎΠ»ΠΎΠΆΠΈΡ‚Π΅Π»ΡŒΠ½Ρ‹ΠΉ заряд ΠΏΠΎΠΊΠΈΠ΄Π°Π΅Ρ‚ это мСсто, большС ΠΏΠΎΠ»ΠΎΠΆΠΈΡ‚Π΅Π»ΡŒΠ½ΠΎΠ³ΠΎ заряда ΠΈΠ· сосСднСго мСста пСрСмСщаСтся Π²Π½ΡƒΡ‚Ρ€ΡŒ. Π£ нас Π΅ΡΡ‚ΡŒ вся скопившаяся масса носитСлСй ΠΏΠΎΠ»ΠΎΠΆΠΈΡ‚Π΅Π»ΡŒΠ½ΠΎΠ³ΠΎ заряда, двиТущихся ΠΏΠΎ ΠΏΠ΅Ρ‚Π»Π΅ ΠΏΠΎ часовой стрСлкС, всС ΠΈΠ·-Π·Π° элСктричСского поля Π² рСзисторС, ΠΈ Β«Π½Π°ΡΡ‚ΠΎΠΉΡ‡ΠΈΠ²ΠΎΡΡ‚ΡŒΒ» Π­Π”Π‘ Π² ΠΏΠΎΠ΄Π΄Π΅Ρ€ΠΆΠ°Π½ΠΈΠΈ постоянной разности ΠΏΠΎΡ‚Π΅Π½Ρ†ΠΈΠ°Π»ΠΎΠ² ΠΌΠ΅ΠΆΠ΄Ρƒ ΠΏΡ€ΠΎΠ²ΠΎΠ΄Π°ΠΌΠΈ.

Π’Π΅ΠΏΠ΅Ρ€ΡŒ нарисуйтС ΠΏΡƒΠ½ΠΊΡ‚ΠΈΡ€Π½ΡƒΡŽ линию ΠΏΠΎΠΏΠ΅Ρ€Π΅ΠΊ ΠΊΠΎΠ½Ρ‚ΡƒΡ€Π° Π² любой Ρ‚ΠΎΡ‡ΠΊΠ΅ ΠΊΠΎΠ½Ρ‚ΡƒΡ€Π°, ΠΊΠ°ΠΊ ΠΏΠΎΠΊΠ°Π·Π°Π½ΠΎ Π½ΠΈΠΆΠ΅.

Π‘ΠΊΠΎΡ€ΠΎΡΡ‚ΡŒ, с ΠΊΠΎΡ‚ΠΎΡ€ΠΎΠΉ заряд пСрСсСкаСт эту линию, являСтся ΡΠΊΠΎΡ€ΠΎΡΡ‚ΡŒΡŽ ΠΏΠΎΡ‚ΠΎΠΊΠ° заряда Π² этой Ρ‚ΠΎΡ‡ΠΊΠ΅ (Ρ‚ΠΎΡ‡ΠΊΠ΅, Π² ΠΊΠΎΡ‚ΠΎΡ€ΠΎΠΉ Π²Ρ‹ нарисовали ΠΏΡƒΠ½ΠΊΡ‚ΠΈΡ€Π½ΡƒΡŽ линию) Π² Ρ†Π΅ΠΏΠΈ. Π‘ΠΊΠΎΡ€ΠΎΡΡ‚ΡŒ ΠΏΠΎΡ‚ΠΎΠΊΠ° заряда, сколько ΠΊΡƒΠ»ΠΎΠ½ΠΎΠ² заряда Π² сСкунду пСрСсСкаСт эту линию, называСтся элСктричСским Ρ‚ΠΎΠΊΠΎΠΌ Π² этой Ρ‚ΠΎΡ‡ΠΊΠ΅. Π’ Π΄Π°Π½Π½ΠΎΠΌ случаС, ΠΏΠΎΡΠΊΠΎΠ»ΡŒΠΊΡƒ вся схСма состоит ΠΈΠ· ΠΎΠ΄Π½ΠΎΠΉ ΠΏΠ΅Ρ‚Π»ΠΈ, Ρ‚ΠΎΠΊ ΠΎΠ΄ΠΈΠ½Π°ΠΊΠΎΠ² Π² ΠΊΠ°ΠΆΠ΄ΠΎΠΉ Ρ‚ΠΎΡ‡ΠΊΠ΅ схСмы – Π½Π΅ ΠΈΠΌΠ΅Π΅Ρ‚ значСния, Π³Π΄Π΅ Π²Ρ‹ «рисуСтС линию».”Бимвол, ΠΊΠΎΡ‚ΠΎΡ€Ρ‹ΠΉ ΠΎΠ±Ρ‹Ρ‡Π½ΠΎ ΠΈΡΠΏΠΎΠ»ΡŒΠ·ΡƒΠ΅Ρ‚ΡΡ для прСдставлСния значСния Ρ‚ΠΎΠΊΠ°, – это \ (I \).

ΠŸΡ€ΠΈ Π°Π½Π°Π»ΠΈΠ·Π΅ схСмы, Ссли тСкущая пСрСмСнная Π΅Ρ‰Π΅ Π½Π΅ ΠΎΠΏΡ€Π΅Π΄Π΅Π»Π΅Π½Π° для вас, Π²Ρ‹ Π΄ΠΎΠ»ΠΆΠ½Ρ‹ ΠΎΠΏΡ€Π΅Π΄Π΅Π»ΠΈΡ‚ΡŒ Π΅Π΅
, нарисовав стрСлку Π½Π° схСмС ΠΈ ΠΏΠΎΠΌΠ΅Ρ‚ΠΈΠ² Π΅Π΅ \ (I \) ΠΈΠ»ΠΈ \ (I \) Π½ΠΈΠΆΠ½ΠΈΠΌ индСксом.

Π•Π΄ΠΈΠ½ΠΈΡ†Ρ‹ измСрСния Ρ‚ΠΎΠΊΠ° – ΠΊΡƒΠ»ΠΎΠ½Ρ‹ Π² сСкунду (\ (C / s \)). Π­Ρ‚ΠΎΠΉ ΠΊΠΎΠΌΠ±ΠΈΠ½Π°Ρ†ΠΈΠΈ Π΅Π΄ΠΈΠ½ΠΈΡ† Π΄Π°Π½ΠΎ имя: Π°ΠΌΠΏΠ΅Ρ€, сокращСнно \ (A \).

\ [1A = 1 \ frac {C} {s} \]

Π’Π΅ΠΏΠ΅Ρ€ΡŒ ΠΎΠ± этом рСзисторС: Π² нашСй ΠΌΠΎΠ΄Π΅Π»ΠΈ носитСля ΠΏΠΎΠ»ΠΎΠΆΠΈΡ‚Π΅Π»ΡŒΠ½ΠΎΠ³ΠΎ заряда заряТСнныС частицы, ΠΊΠΎΡ‚ΠΎΡ€Ρ‹Π΅ ΠΌΠΎΠ³ΡƒΡ‚ свободно ΠΏΠ΅Ρ€Π΅ΠΌΠ΅Ρ‰Π°Ρ‚ΡŒΡΡ Π² рСзисторС, ΠΈΡΠΏΡ‹Ρ‚Ρ‹Π²Π°ΡŽΡ‚ силу, Π΄Π΅ΠΉΡΡ‚Π²ΡƒΡŽΡ‰ΡƒΡŽ Π½Π° Π½ΠΈΡ… элСктричСским ΠΏΠΎΠ»Π΅ΠΌ Π² Π½Π°ΠΏΡ€Π°Π²Π»Π΅Π½ΠΈΠΈ элСктричСского поля.Π’ Ρ€Π΅Π·ΡƒΠ»ΡŒΡ‚Π°Ρ‚Π΅ ΠΎΠ½ΠΈ ΠΈΡΠΏΡ‹Ρ‚Ρ‹Π²Π°ΡŽΡ‚ ускорСниС. Но Ρ„ΠΎΠ½ΠΎΠ²Ρ‹ΠΉ ΠΌΠ°Ρ‚Π΅Ρ€ΠΈΠ°Π», ΡΠΎΡΡ‚Π°Π²Π»ΡΡŽΡ‰ΠΈΠΉ вСщСство, Ρ‡Π°ΡΡ‚ΡŒΡŽ ΠΊΠΎΡ‚ΠΎΡ€ΠΎΠ³ΠΎ ΡΠ²Π»ΡΡŽΡ‚ΡΡ носитСли заряда, ΠΎΠΊΠ°Π·Ρ‹Π²Π°Π΅Ρ‚ Π½Π° носитСли заряда Π·Π°ΠΌΠ΅Π΄Π»ΡΡŽΡ‰ΡƒΡŽ силу, Π·Π°Π²ΠΈΡΡΡ‰ΡƒΡŽ ΠΎΡ‚ скорости. Π§Π΅ΠΌ быстрСС ΠΎΠ½ΠΈ двиТутся, Ρ‚Π΅ΠΌ большС тормозящая сила. ПослС Π·Π°Π²Π΅Ρ€ΡˆΠ΅Π½ΠΈΡ Ρ†Π΅ΠΏΠΈ (создания послСднСго соСдинСния ΠΏΡ€ΠΎΠ²ΠΎΠ΄-ΠΊΠ»Π΅ΠΌΠΌΠ°) носитСли заряда Π² рСзисторС ΠΏΠΎΡ‡Ρ‚ΠΈ ΠΌΠ³Π½ΠΎΠ²Π΅Π½Π½ΠΎ Π΄ΠΎΡΡ‚ΠΈΠ³Π°ΡŽΡ‚ ΠΊΠΎΠ½Π΅Ρ‡Π½ΠΎΠΉ скорости, ΠΏΡ€ΠΈ ΠΊΠΎΡ‚ΠΎΡ€ΠΎΠΉ тормозящая сила Π½Π° Π΄Π°Π½Π½ΠΎΠΌ носитСлС заряда Ρ‚Π°ΠΊ ΠΆΠ΅ Π²Π΅Π»ΠΈΠΊΠ°, ΠΊΠ°ΠΊ ΠΈ сила, Π΄Π΅ΠΉΡΡ‚Π²ΡƒΡŽΡ‰Π°Ρ Π½Π° Π½Π΅Π³ΠΎ. элСктричСскоС ΠΏΠΎΠ»Π΅ Π½Π° этом носитСлС заряда.Π—Π½Π°Ρ‡Π΅Π½ΠΈΠ΅ ΠΊΠΎΠ½Π΅Ρ‡Π½ΠΎΠΉ скорости вмСстС с количСством носитСлСй заряда Π½Π° объСм Π² рСзисторС ΠΈ ΠΏΠ»ΠΎΡ‰Π°Π΄ΡŒΡŽ ΠΏΠΎΠΏΠ΅Ρ€Π΅Ρ‡Π½ΠΎΠ³ΠΎ сСчСния ΠΏΠ»ΠΎΡ…ΠΎ проводящСго ΠΌΠ°Ρ‚Π΅Ρ€ΠΈΠ°Π»Π°, ΡΠΎΡΡ‚Π°Π²Π»ΡΡŽΡ‰Π΅Π³ΠΎ рСзистор, ΠΎΠΏΡ€Π΅Π΄Π΅Π»ΡΡŽΡ‚ ΡΠΊΠΎΡ€ΠΎΡΡ‚ΡŒ ΠΏΠΎΡ‚ΠΎΠΊΠ° заряда, Ρ‚ΠΎΠΊ , Π² рСзисторС. Π’ рассматриваСмой простой схСмС расход заряда Π² рСзисторС – это расход заряда Π²ΠΎ всСй Ρ†Π΅ΠΏΠΈ.

Π‘Π°ΠΌΠΎ Π·Π½Π°Ρ‡Π΅Π½ΠΈΠ΅ ΠΊΠΎΠ½Π΅Ρ‡Π½ΠΎΠΉ скорости зависит ΠΎΡ‚ силы элСктричСского поля ΠΈ ΠΏΡ€ΠΈΡ€ΠΎΠ΄Ρ‹ тормозящСй силы.Π₯Π°Ρ€Π°ΠΊΡ‚Π΅Ρ€ тормозящСй силы зависит ΠΎΡ‚ ΠΌΠ°Ρ‚Π΅Ρ€ΠΈΠ°Π»Π°, ΠΈΠ· ΠΊΠΎΡ‚ΠΎΡ€ΠΎΠ³ΠΎ ΠΈΠ·Π³ΠΎΡ‚ΠΎΠ²Π»Π΅Π½ рСзистор. Один Π²ΠΈΠ΄ ΠΌΠ°Ρ‚Π΅Ρ€ΠΈΠ°Π»Π° ΠΏΡ€ΠΈΠ²Π΅Π΄Π΅Ρ‚ ΠΊ большСй ΠΊΠΎΠ½Π΅Ρ‡Π½ΠΎΠΉ скорости для Ρ‚ΠΎΠ³ΠΎ ΠΆΠ΅ элСктричСского поля, Ρ‡Ρ‚ΠΎ ΠΈ Π΄Ρ€ΡƒΠ³ΠΎΠΉ Π²ΠΈΠ΄ ΠΌΠ°Ρ‚Π΅Ρ€ΠΈΠ°Π»Π°. Π”Π°ΠΆΠ΅ с ΠΎΠ΄Π½ΠΈΠΌ Ρ‚ΠΈΠΏΠΎΠΌ ΠΌΠ°Ρ‚Π΅Ρ€ΠΈΠ°Π»Π° Π²ΠΎΠ·Π½ΠΈΠΊΠ°Π΅Ρ‚ вопрос, ΠΊΠ°ΠΊ тормозящая сила зависит ΠΎΡ‚ скорости. Он ΠΏΡ€ΠΎΠΏΠΎΡ€Ρ†ΠΈΠΎΠ½Π°Π»Π΅Π½ ΠΊΠ²Π°Π΄Ρ€Π°Ρ‚Ρƒ скорости, Π»ΠΎΠ³Π°Ρ€ΠΈΡ„ΠΌΡƒ скорости ΠΈΠ»ΠΈ Ρ‡Π΅ΠΌΡƒ-Ρ‚ΠΎ Π΅Ρ‰Π΅? ЭкспСримСнт ΠΏΠΎΠΊΠ°Π·Ρ‹Π²Π°Π΅Ρ‚, Ρ‡Ρ‚ΠΎ Π² Π²Π°ΠΆΠ½ΠΎΠΌ подмноТСствС ΠΌΠ°Ρ‚Π΅Ρ€ΠΈΠ°Π»ΠΎΠ² Π² ΠΎΠΏΡ€Π΅Π΄Π΅Π»Π΅Π½Π½Ρ‹Ρ… Π΄ΠΈΠ°ΠΏΠ°Π·ΠΎΠ½Π°Ρ… ΠΊΠΎΠ½Π΅Ρ‡Π½ΠΎΠΉ скорости тормозящая сила ΠΏΡ€ΠΎΠΏΠΎΡ€Ρ†ΠΈΠΎΠ½Π°Π»ΡŒΠ½Π° самой скорости.Π’Π°ΠΊΠΈΠ΅ ΠΌΠ°Ρ‚Π΅Ρ€ΠΈΠ°Π»Ρ‹ ΠΏΠΎΠ΄Ρ‡ΠΈΠ½ΡΡŽΡ‚ΡΡ Π·Π°ΠΊΠΎΠ½Ρƒ Ома ΠΈ Π½Π°Π·Ρ‹Π²Π°ΡŽΡ‚ΡΡ омичСскими ΠΌΠ°Ρ‚Π΅Ρ€ΠΈΠ°Π»Π°ΠΌΠΈ.

Рассмотрим рСзистор Π² простой схСмС, с ΠΊΠΎΡ‚ΠΎΡ€ΠΎΠΉ ΠΌΡ‹ ΠΈΠΌΠ΅Π»ΠΈ Π΄Π΅Π»ΠΎ.

Если Π²Ρ‹ ΡƒΠ΄Π²ΠΎΠΈΡ‚Π΅ напряТСниС Π½Π° рСзисторС (ΠΈΡΠΏΠΎΠ»ΡŒΠ·ΡƒΡ Π³Π½Π΅Π·Π΄ΠΎ Π­Π”Π‘, ΠΊΠΎΡ‚ΠΎΡ€ΠΎΠ΅ ΠΏΠΎΠ΄Π΄Π΅Ρ€ΠΆΠΈΠ²Π°Π΅Ρ‚ Π²Π΄Π²ΠΎΠ΅ Π±ΠΎΠ»ΡŒΡˆΡƒΡŽ Ρ€Π°Π·Π½ΠΎΡΡ‚ΡŒ ΠΏΠΎΡ‚Π΅Π½Ρ†ΠΈΠ°Π»ΠΎΠ² ΠΌΠ΅ΠΆΠ΄Ρƒ Π΅Π³ΠΎ Π²Ρ‹Π²ΠΎΠ΄Π°ΠΌΠΈ, ΠΊΠ°ΠΊ исходноС Π³Π½Π΅Π·Π΄ΠΎ Π­Π”Π‘), Ρ‚ΠΎ Π²Ρ‹ ΡƒΠ΄Π²ΠΎΠΈΡ‚Π΅ элСктричСскоС ΠΏΠΎΠ»Π΅ Π² рСзисторС. Π­Ρ‚ΠΎ ΡƒΠ΄Π²Π°ΠΈΠ²Π°Π΅Ρ‚ силу, Π΄Π΅ΠΉΡΡ‚Π²ΡƒΡŽΡ‰ΡƒΡŽ Π½Π° ΠΊΠ°ΠΆΠ΄Ρ‹ΠΉ Π½ΠΎΡΠΈΡ‚Π΅Π»ΡŒ заряда. Π­Ρ‚ΠΎ ΠΎΠ·Π½Π°Ρ‡Π°Π΅Ρ‚, Ρ‡Ρ‚ΠΎ ΠΏΡ€ΠΈ ΠΏΡ€Π΅Π΄Π΅Π»ΡŒΠ½ΠΎΠΉ скорости любого носитСля заряда тормозящая сила Π΄ΠΎΠ»ΠΆΠ½Π° Π±Ρ‹Ρ‚ΡŒ Π²Π΄Π²ΠΎΠ΅ большС.(ΠŸΠΎΡΠΊΠΎΠ»ΡŒΠΊΡƒ послС ΠΏΠΎΠ΄ΠΊΠ»ΡŽΡ‡Π΅Π½ΠΈΡ этой послСднСй Ρ†Π΅ΠΏΠΈ ΡΠΊΠΎΡ€ΠΎΡΡ‚ΡŒ носитСлСй заряда увСличиваСтся Π΄ΠΎ Ρ‚Π΅Ρ… ΠΏΠΎΡ€, ΠΏΠΎΠΊΠ° тормозящая сила Π½Π° ΠΊΠ°ΠΆΠ΄ΠΎΠΌ носитСлС заряда Π½Π΅ станСт Ρ€Π°Π²Π½ΠΎΠΉ ΠΏΠΎ Π²Π΅Π»ΠΈΡ‡ΠΈΠ½Π΅ ΠΏΡ€ΠΈΠ»ΠΎΠΆΠ΅Π½Π½ΠΎΠΉ силС.) Π’ омичСском ΠΌΠ°Ρ‚Π΅Ρ€ΠΈΠ°Π»Π΅, Ссли тормозящая сила Π²Π΄Π²ΠΎΠ΅ большС, Ρ‚ΠΎ ΡΠΊΠΎΡ€ΠΎΡΡ‚ΡŒ Π²Π΄Π²ΠΎΠ΅ большС. Если ΡΠΊΠΎΡ€ΠΎΡΡ‚ΡŒ Π²Π΄Π²ΠΎΠ΅ большС, Ρ‚ΠΎ расход заряда, элСктричСский Ρ‚ΠΎΠΊ, Π²Π΄Π²ΠΎΠ΅ большС. Π’Π°ΠΊΠΈΠΌ ΠΎΠ±Ρ€Π°Π·ΠΎΠΌ, ΡƒΠ΄Π²ΠΎΠ΅Π½ΠΈΠ΅ напряТСния Π½Π° рСзисторС ΡƒΠ²Π΅Π»ΠΈΡ‡ΠΈΠ²Π°Π΅Ρ‚ Π²Π΄Π²ΠΎΠ΅ Ρ‚ΠΎΠΊ. Π”Π΅ΠΉΡΡ‚Π²ΠΈΡ‚Π΅Π»ΡŒΠ½ΠΎ, для рСзистора, ΠΏΠΎΠ΄Ρ‡ΠΈΠ½ΡΡŽΡ‰Π΅Π³ΠΎΡΡ Π·Π°ΠΊΠΎΠ½Ρƒ Ома, Ρ‚ΠΎΠΊ Π² рСзисторС прямо ΠΏΡ€ΠΎΠΏΠΎΡ€Ρ†ΠΈΠΎΠ½Π°Π»Π΅Π½ Π½Π°ΠΏΡ€ΡΠΆΠ΅Π½ΠΈΡŽ Π½Π° рСзисторС.

ПодвСдСниС ΠΈΡ‚ΠΎΠ³ΠΎΠ²: ΠΊΠΎΠ³Π΄Π° Π²Ρ‹ ΠΏΠΎΠ΄Π°Π΅Ρ‚Π΅ напряТСниС Π½Π° рСзистор, Π² этом рСзисторС Π΅ΡΡ‚ΡŒ Ρ‚ΠΎΠΊ. ΠžΡ‚Π½ΠΎΡˆΠ΅Π½ΠΈΠ΅ напряТСния ΠΊ Ρ‚ΠΎΠΊΡƒ называСтся сопротивлСниСм рСзистора.

\ [R = \ frac {V} {I} \]

Π­Ρ‚ΠΎ ΠΎΠΏΡ€Π΅Π΄Π΅Π»Π΅Π½ΠΈΠ΅ сопротивлСния согласуСтся с нашим ΠΏΠΎΠ½ΠΈΠΌΠ°Π½ΠΈΠ΅ΠΌ Ρ‚ΠΎΠ³ΠΎ, Ρ‡Ρ‚ΠΎ сопротивлСниС рСзистора являСтся ΠΌΠ΅Ρ€ΠΎΠΉ Ρ‚ΠΎΠ³ΠΎ, насколько ΠΎΠ½ ΠΏΠ»ΠΎΡ…ΠΎΠΉ ΠΏΡ€ΠΎΠ²ΠΎΠ΄Π½ΠΈΠΊ. ΠŸΡ€ΠΎΠ²Π΅Ρ€ΠΈΡ‚ΡŒ это. Если для Π΄Π°Π½Π½ΠΎΠ³ΠΎ напряТСния Π½Π° рСзисторС Π²Ρ‹ ΠΏΠΎΠ»ΡƒΡ‡Π°Π΅Ρ‚Π΅ ΠΊΡ€ΠΎΡˆΠ΅Ρ‡Π½Ρ‹ΠΉ нСбольшой Ρ‚ΠΎΠΊ (это ΠΎΠ·Π½Π°Ρ‡Π°Π΅Ρ‚, Ρ‡Ρ‚ΠΎ рСзистор являСтся ΠΎΡ‡Π΅Π½ΡŒ ΠΏΠ»ΠΎΡ…ΠΈΠΌ ΠΏΡ€ΠΎΠ²ΠΎΠ΄Π½ΠΈΠΊΠΎΠΌ), Π·Π½Π°Ρ‡Π΅Π½ΠΈΠ΅ сопротивлСния \ (R = \ frac {V} {I} \) с этим малСньким Π·Π½Π°Ρ‡Π΅Π½ΠΈΠ΅ΠΌ Ρ‚ΠΎΠΊ Π² Π·Π½Π°ΠΌΠ΅Π½Π°Ρ‚Π΅Π»Π΅ ΠΎΡ‡Π΅Π½ΡŒ Π²Π΅Π»ΠΈΠΊ.Если, с Π΄Ρ€ΡƒΠ³ΠΎΠΉ стороны, ΠΏΡ€ΠΈ Ρ‚ΠΎΠΌ ΠΆΠ΅ напряТСнии Π²Ρ‹ ΠΏΠΎΠ»ΡƒΡ‡Π°Π΅Ρ‚Π΅ большой Ρ‚ΠΎΠΊ (Ρ‡Ρ‚ΠΎ ΠΎΠ·Π½Π°Ρ‡Π°Π΅Ρ‚, Ρ‡Ρ‚ΠΎ рСзистор являСтся Ρ…ΠΎΡ€ΠΎΡˆΠΈΠΌ ΠΏΡ€ΠΎΠ²ΠΎΠ΄Π½ΠΈΠΊΠΎΠΌ), Ρ‚ΠΎΠ³Π΄Π° Π·Π½Π°Ρ‡Π΅Π½ΠΈΠ΅ сопротивлСния \ (R = \ frac {V} {I} \) ΠΌΠ°Π»ΠΎ.

Если ΠΌΠ°Ρ‚Π΅Ρ€ΠΈΠ°Π», ΠΈΠ· ΠΊΠΎΡ‚ΠΎΡ€ΠΎΠ³ΠΎ ΠΈΠ·Π³ΠΎΡ‚ΠΎΠ²Π»Π΅Π½ рСзистор, подчиняСтся Π·Π°ΠΊΠΎΠ½Ρƒ Ома, Ρ‚ΠΎ сопротивлСниС \ (R \) являСтся постоянным, Ρ‡Ρ‚ΠΎ ΠΎΠ·Π½Π°Ρ‡Π°Π΅Ρ‚, Ρ‡Ρ‚ΠΎ Π΅Π³ΠΎ Π·Π½Π°Ρ‡Π΅Π½ΠΈΠ΅ ΠΎΠ΄ΠΈΠ½Π°ΠΊΠΎΠ²ΠΎ для Ρ€Π°Π·Π½Ρ‹Ρ… напряТСний. ΠžΡ‚Π½ΠΎΡˆΠ΅Π½ΠΈΠ΅ \ (R = \ frac {V} {I} \) ΠΎΠ±Ρ‹Ρ‡Π½ΠΎ записываСтся Π² Ρ„ΠΎΡ€ΠΌΠ΅ \ (V = IR \).

Π—Π°ΠΊΠΎΠ½ Ома:

Π‘ΠΎΠΏΡ€ΠΎΡ‚ΠΈΠ²Π»Π΅Π½ΠΈΠ΅ \ (R \) Π² Π²Ρ‹Ρ€Π°ΠΆΠ΅Π½ΠΈΠΈ \ (V = IR \) являСтся постоянной Π²Π΅Π»ΠΈΡ‡ΠΈΠ½ΠΎΠΉ.

Π—Π°ΠΊΠΎΠ½

Ома ΠΏΠΎΠ΄Ρ…ΠΎΠ΄ΠΈΡ‚ для рСзисторов, ΠΈΠ·Π³ΠΎΡ‚ΠΎΠ²Π»Π΅Π½Π½Ρ‹Ρ… ΠΈΠ· ΠΎΠΏΡ€Π΅Π΄Π΅Π»Π΅Π½Π½Ρ‹Ρ… ΠΌΠ°Ρ‚Π΅Ρ€ΠΈΠ°Π»ΠΎΠ² (Π½Π°Π·Ρ‹Π²Π°Π΅ΠΌΡ‹Ρ… омичСскими ΠΌΠ°Ρ‚Π΅Ρ€ΠΈΠ°Π»Π°ΠΌΠΈ) Π² ΠΎΠ³Ρ€Π°Π½ΠΈΡ‡Π΅Π½Π½ΠΎΠΌ Π΄ΠΈΠ°ΠΏΠ°Π·ΠΎΠ½Π΅ напряТСний.

Π•Π΄ΠΈΠ½ΠΈΡ† сопротивлСния

Учитывая, Ρ‡Ρ‚ΠΎ сопротивлСниС рСзистора опрСдСляСтся ΠΊΠ°ΠΊ ΠΎΡ‚Π½ΠΎΡˆΠ΅Π½ΠΈΠ΅ напряТСния Π½Π° этом рСзисторС ΠΊ Ρ€Π΅Π·ΡƒΠ»ΡŒΡ‚ΠΈΡ€ΡƒΡŽΡ‰Π΅ΠΌΡƒ Ρ‚ΠΎΠΊΡƒ Π½Π° этом рСзисторС,

\ [R = \ frac {V} {I} \]

ΠΎΡ‡Π΅Π²ΠΈΠ΄Π½ΠΎ, Ρ‡Ρ‚ΠΎ Π΅Π΄ΠΈΠ½ΠΈΡ†Π΅ΠΉ сопротивлСния являСтся Π²ΠΎΠ»ΡŒΡ‚ Π½Π° Π°ΠΌΠΏΠ΅Ρ€, \ (\ frac {V} {A} \). Π­Ρ‚ΠΎΠΌΡƒ ΠΊΠΎΠΌΠ±ΠΈΠ½ΠΈΡ€ΠΎΠ²Π°Π½Π½ΠΎΠΌΡƒ Π±Π»ΠΎΠΊΡƒ присвоСно имя. ΠœΡ‹ Π½Π°Π·Ρ‹Π²Π°Π΅ΠΌ это ΠΎΠΌ, сокращСнно \ (\ Omega \), грСчСская Π±ΡƒΠΊΠ²Π° Π² Π²Π΅Ρ€Ρ…Π½Π΅ΠΌ рСгистрС ΠΎΠΌΠ΅Π³Π°.

\ [1 \ Omega = 1 \ frac {\ mbox {volt}} {\ mbox {ampere}} \]

Авторы ΠΈ авторство

10.2: ЭлСктродвиТущая сила – Physics LibreTexts

Π¦Π΅Π»ΠΈ обучСния

К ΠΊΠΎΠ½Ρ†Ρƒ Ρ€Π°Π·Π΄Π΅Π»Π° Π²Ρ‹ смоТСтС:

  • ΠžΠΏΠΈΡˆΠΈΡ‚Π΅ ΡΠ»Π΅ΠΊΡ‚Ρ€ΠΎΠ΄Π²ΠΈΠΆΡƒΡ‰ΡƒΡŽ силу (Π­Π”Π‘) ΠΈ Π²Π½ΡƒΡ‚Ρ€Π΅Π½Π½Π΅Π΅ сопротивлСниС Π±Π°Ρ‚Π°Ρ€Π΅ΠΈ
  • ΠžΠ±ΡŠΡΡΠ½ΠΈΡ‚Π΅ ΠΎΡΠ½ΠΎΠ²Π½ΡƒΡŽ Ρ€Π°Π±ΠΎΡ‚Ρƒ аккумулятора

Если Π²Ρ‹ Π·Π°Π±Ρ‹Π»ΠΈ Π²Ρ‹ΠΊΠ»ΡŽΡ‡ΠΈΡ‚ΡŒ Π°Π²Ρ‚ΠΎΠΌΠΎΠ±ΠΈΠ»ΡŒΠ½Ρ‹Π΅ Ρ„Π°Ρ€Ρ‹, ΠΎΠ½ΠΈ постСпСнно Ρ‚ΡƒΡΠΊΠ½Π΅ΡŽΡ‚ ΠΏΠΎ ΠΌΠ΅Ρ€Π΅ разрядки аккумулятора.ΠŸΠΎΡ‡Π΅ΠΌΡƒ ΠΎΠ½ΠΈ Π½Π΅ ΠΌΠΈΠ³Π°ΡŽΡ‚ Π²Π½Π΅Π·Π°ΠΏΠ½ΠΎ, ΠΊΠΎΠ³Π΄Π° разрядился аккумулятор? Π˜Ρ… постСпСнноС Π·Π°Ρ‚Π΅ΠΌΠ½Π΅Π½ΠΈΠ΅ ΠΎΠ·Π½Π°Ρ‡Π°Π΅Ρ‚, Ρ‡Ρ‚ΠΎ Π²Ρ‹Ρ…ΠΎΠ΄Π½ΠΎΠ΅ напряТСниС Π±Π°Ρ‚Π°Ρ€Π΅ΠΈ ΡƒΠΌΠ΅Π½ΡŒΡˆΠ°Π΅Ρ‚ΡΡ ΠΏΠΎ ΠΌΠ΅Ρ€Π΅ разряда Π±Π°Ρ‚Π°Ρ€Π΅ΠΈ. ΠŸΡ€ΠΈΡ‡ΠΈΠ½Π° сниТСния Π²Ρ‹Ρ…ΠΎΠ΄Π½ΠΎΠ³ΠΎ напряТСния для разряТСнных Π±Π°Ρ‚Π°Ρ€Π΅ΠΉ Π·Π°ΠΊΠ»ΡŽΡ‡Π°Π΅Ρ‚ΡΡ Π² Ρ‚ΠΎΠΌ, Ρ‡Ρ‚ΠΎ всС источники напряТСния состоят ΠΈΠ· Π΄Π²ΡƒΡ… основных частСй – источника элСктричСской энСргии ΠΈ Π²Π½ΡƒΡ‚Ρ€Π΅Π½Π½Π΅Π³ΠΎ сопротивлСния. Π’ этом Ρ€Π°Π·Π΄Π΅Π»Π΅ ΠΌΡ‹ исслСдуСм источник энСргии ΠΈ Π²Π½ΡƒΡ‚Ρ€Π΅Π½Π½Π΅Π΅ сопротивлСниС.

Π’Π²Π΅Π΄Π΅Π½ΠΈΠ΅ Π² ΡΠ»Π΅ΠΊΡ‚Ρ€ΠΎΠ΄Π²ΠΈΠΆΡƒΡ‰ΡƒΡŽ силу

Voltage ΠΈΠΌΠ΅Π΅Ρ‚ мноТСство источников, Π½Π΅ΠΊΠΎΡ‚ΠΎΡ€Ρ‹Π΅ ΠΈΠ· ΠΊΠΎΡ‚ΠΎΡ€Ρ‹Ρ… ΠΏΠΎΠΊΠ°Π·Π°Π½Ρ‹ Π½Π° рисункС \ (\ PageIndex {2} \).ВсС Ρ‚Π°ΠΊΠΈΠ΅ устройства ΡΠΎΠ·Π΄Π°ΡŽΡ‚ Ρ€Π°Π·Π½ΠΎΡΡ‚ΡŒ ΠΏΠΎΡ‚Π΅Π½Ρ†ΠΈΠ°Π»ΠΎΠ² ΠΈ ΠΌΠΎΠ³ΡƒΡ‚ ΠΏΠΎΠ΄Π°Π²Π°Ρ‚ΡŒ Ρ‚ΠΎΠΊ, Ссли ΠΏΠΎΠ΄ΠΊΠ»ΡŽΡ‡Π΅Π½Ρ‹ ΠΊ Ρ†Π΅ΠΏΠΈ. ΠžΡΠΎΠ±Ρ‹ΠΉ Ρ‚ΠΈΠΏ разности ΠΏΠΎΡ‚Π΅Π½Ρ†ΠΈΠ°Π»ΠΎΠ² извСстСн ΠΊΠ°ΠΊ элСктродвиТущая сила (Π­Π”Π‘) . Π­Π”Π‘ – это вовсС Π½Π΅ сила, Π½ΠΎ Ρ‚Π΅Ρ€ΠΌΠΈΠ½ «элСктродвиТущая сила» ΠΈΡΠΏΠΎΠ»ΡŒΠ·ΡƒΠ΅Ρ‚ΡΡ ΠΏΠΎ историчСским ΠΏΡ€ΠΈΡ‡ΠΈΠ½Π°ΠΌ. Он Π±Ρ‹Π» ΠΈΠ·ΠΎΠ±Ρ€Π΅Ρ‚Π΅Π½ АлСссандро Π’ΠΎΠ»ΡŒΡ‚Π° Π² 1800-Ρ… Π³ΠΎΠ΄Π°Ρ…, ΠΊΠΎΠ³Π΄Π° ΠΎΠ½ ΠΈΠ·ΠΎΠ±Ρ€Π΅Π» ΠΏΠ΅Ρ€Π²ΡƒΡŽ Π±Π°Ρ‚Π°Ρ€Π΅ΡŽ, Ρ‚Π°ΠΊΠΆΠ΅ ΠΈΠ·Π²Π΅ΡΡ‚Π½ΡƒΡŽ ΠΊΠ°ΠΊ Π²ΠΎΠ»ΡŒΡ‚ΠΎΠ²ΡƒΡŽ Π±Π°Ρ‚Π°Ρ€Π΅ΡŽ . ΠŸΠΎΡΠΊΠΎΠ»ΡŒΠΊΡƒ элСктродвиТущая сила Π½Π΅ являСтся силой, принято Π½Π°Π·Ρ‹Π²Π°Ρ‚ΡŒ эти источники просто источниками Π­Π”Π‘ (произносимыми Π±ΡƒΠΊΠ²Π°ΠΌΠΈ Β«ee-em-effΒ»), Π° Π½Π΅ источниками элСктродвиТущСй силы.

Рисунок \ (\ PageIndex {1} \): Ρ€Π°Π·Π»ΠΈΡ‡Π½Ρ‹Π΅ источники напряТСния. Π°) вСтряная элСктростанция Бразос Π² Π€Π»ΡƒΠ²Π°Π½Π½Π°, ΡˆΡ‚Π°Ρ‚ ВСхас; (Π±) ΠšΡ€Π°ΡΠ½ΠΎΡΡ€ΡΠΊΠ°Ρ ΠΏΠ»ΠΎΡ‚ΠΈΠ½Π° Π² России; (c) солнСчная Ρ„Π΅Ρ€ΠΌΠ°; (d) Π³Ρ€ΡƒΠΏΠΏΠ° никСль-ΠΌΠ΅Ρ‚Π°Π»Π»ΠΎΠ³ΠΈΠ΄Ρ€ΠΈΠ΄Π½Ρ‹Ρ… Π±Π°Ρ‚Π°Ρ€Π΅ΠΉ. Π’Ρ‹Ρ…ΠΎΠ΄Π½ΠΎΠ΅ напряТСниС ΠΊΠ°ΠΆΠ΄ΠΎΠ³ΠΎ устройства зависит ΠΎΡ‚ Π΅Π³ΠΎ конструкции ΠΈ Π½Π°Π³Ρ€ΡƒΠ·ΠΊΠΈ. Π’Ρ‹Ρ…ΠΎΠ΄Π½ΠΎΠ΅ напряТСниС Ρ€Π°Π²Π½ΠΎ Π­Π”Π‘ Ρ‚ΠΎΠ»ΡŒΠΊΠΎ ΠΏΡ€ΠΈ отсутствии Π½Π°Π³Ρ€ΡƒΠ·ΠΊΠΈ. (ΠΊΡ€Π΅Π΄ΠΈΡ‚ a: модификация Ρ€Π°Π±ΠΎΡ‚Ρ‹ Β«LeafletΒ» / Wikimedia Commons; ΠΊΡ€Π΅Π΄ΠΈΡ‚ b: модификация Ρ€Π°Π±ΠΎΡ‚Ρ‹ АлСкса ПолСТаСва; ΠΊΡ€Π΅Π΄ΠΈΡ‚ c: модификация Ρ€Π°Π±ΠΎΡ‚Ρ‹ ΠœΠΈΠ½ΠΈΡΡ‚Π΅Ρ€ΡΡ‚Π²Π° энСргСтики БША; ΠΊΡ€Π΅Π΄ΠΈΡ‚ d: модификация Ρ€Π°Π±ΠΎΡ‚Ρ‹ Π’ΠΈΠ°Π° ΠœΠΎΠ½Ρ‚ΠΎ)

Если ЭлСктродвиТущая сила – это Π²ΠΎΠΎΠ±Ρ‰Π΅ Π½Π΅ сила, Ρ‚ΠΎΠ³Π΄Π° Ρ‡Ρ‚ΠΎ Ρ‚Π°ΠΊΠΎΠ΅ Π­Π”Π‘ ΠΈ Ρ‡Ρ‚ΠΎ являСтся источником Π­Π”Π‘? Π§Ρ‚ΠΎΠ±Ρ‹ ΠΎΡ‚Π²Π΅Ρ‚ΠΈΡ‚ΡŒ Π½Π° эти вопросы, рассмотрим ΠΏΡ€ΠΎΡΡ‚ΡƒΡŽ схСму Π»Π°ΠΌΠΏΡ‹ 12 Π’, ΠΏΠΎΠ΄ΠΊΠ»ΡŽΡ‡Π΅Π½Π½ΠΎΠΉ ΠΊ Π±Π°Ρ‚Π°Ρ€Π΅Π΅ 12 Π’, ΠΊΠ°ΠΊ ΠΏΠΎΠΊΠ°Π·Π°Π½ΠΎ Π½Π° рисункС \ (\ PageIndex {2} \).БатарСя , ΠΌΠΎΠΆΠ΅Ρ‚ Π±Ρ‹Ρ‚ΡŒ смодСлирована ΠΊΠ°ΠΊ устройство с двумя Π²Ρ‹Π²ΠΎΠ΄Π°ΠΌΠΈ, ΠΊΠΎΡ‚ΠΎΡ€ΠΎΠ΅ ΠΏΠΎΠ΄Π΄Π΅Ρ€ΠΆΠΈΠ²Π°Π΅Ρ‚ ΠΎΠ΄ΠΈΠ½ Π²Ρ‹Π²ΠΎΠ΄ с Π±ΠΎΠ»Π΅Π΅ высоким элСктричСским ΠΏΠΎΡ‚Π΅Π½Ρ†ΠΈΠ°Π»ΠΎΠΌ, Ρ‡Π΅ΠΌ Π²Ρ‚ΠΎΡ€ΠΎΠΉ Π²Ρ‹Π²ΠΎΠ΄. Π‘ΠΎΠ»Π΅Π΅ высокий элСктричСский ΠΏΠΎΡ‚Π΅Π½Ρ†ΠΈΠ°Π» ΠΈΠ½ΠΎΠ³Π΄Π° Π½Π°Π·Ρ‹Π²Π°ΡŽΡ‚ ΠΏΠΎΠ»ΠΎΠΆΠΈΡ‚Π΅Π»ΡŒΠ½ΠΎΠΉ ΠΊΠ»Π΅ΠΌΠΌΠΎΠΉ ΠΈ ΠΎΠ±ΠΎΠ·Π½Π°Ρ‡Π°ΡŽΡ‚ Π·Π½Π°ΠΊΠΎΠΌ плюс. ΠšΠ»Π΅ΠΌΠΌΡƒ с Π±ΠΎΠ»Π΅Π΅ Π½ΠΈΠ·ΠΊΠΈΠΌ ΠΏΠΎΡ‚Π΅Π½Ρ†ΠΈΠ°Π»ΠΎΠΌ ΠΈΠ½ΠΎΠ³Π΄Π° Π½Π°Π·Ρ‹Π²Π°ΡŽΡ‚ ΠΎΡ‚Ρ€ΠΈΡ†Π°Ρ‚Π΅Π»ΡŒΠ½ΠΎΠΉ ΠΊΠ»Π΅ΠΌΠΌΠΎΠΉ ΠΈ ΠΎΠ±ΠΎΠ·Π½Π°Ρ‡Π°ΡŽΡ‚ Π·Π½Π°ΠΊΠΎΠΌ минус. Π­Ρ‚ΠΎ источник Π­Π”Π‘.

Рисунок \ (\ PageIndex {2} \): Π˜ΡΡ‚ΠΎΡ‡Π½ΠΈΠΊ Π­Π”Π‘ ΠΏΠΎΠ΄Π΄Π΅Ρ€ΠΆΠΈΠ²Π°Π΅Ρ‚ Π½Π° ΠΎΠ΄Π½ΠΎΠΌ Π²Ρ‹Π²ΠΎΠ΄Π΅ Π±ΠΎΠ»Π΅Π΅ высокий элСктричСский ΠΏΠΎΡ‚Π΅Π½Ρ†ΠΈΠ°Π», Ρ‡Π΅ΠΌ Π½Π° Π΄Ρ€ΡƒΠ³ΠΎΠΌ Π²Ρ‹Π²ΠΎΠ΄Π΅, дСйствуя ΠΊΠ°ΠΊ источник Ρ‚ΠΎΠΊΠ° Π² Ρ†Π΅ΠΏΠΈ.

Когда источник Π­Π”Π‘ Π½Π΅ ΠΏΠΎΠ΄ΠΊΠ»ΡŽΡ‡Π΅Π½ ΠΊ Π»Π°ΠΌΠΏΠ΅, Π½Π΅Ρ‚ чистого ΠΏΠΎΡ‚ΠΎΠΊΠ° заряда Π²Π½ΡƒΡ‚Ρ€ΠΈ источника Π­Π”Π‘. Как Ρ‚ΠΎΠ»ΡŒΠΊΠΎ батарСя ΠΏΠΎΠ΄ΠΊΠ»ΡŽΡ‡Π΅Π½Π° ΠΊ Π»Π°ΠΌΠΏΠ΅, заряды Ρ‚Π΅ΠΊΡƒΡ‚ ΠΎΡ‚ ΠΎΠ΄Π½ΠΎΠΉ ΠΊΠ»Π΅ΠΌΠΌΡ‹ Π±Π°Ρ‚Π°Ρ€Π΅ΠΈ Ρ‡Π΅Ρ€Π΅Π· Π»Π°ΠΌΠΏΡƒ (Π² Ρ€Π΅Π·ΡƒΠ»ΡŒΡ‚Π°Ρ‚Π΅ Ρ‡Π΅Π³ΠΎ Π»Π°ΠΌΠΏΠ° загораСтся) ΠΈ ΠΎΠ±Ρ€Π°Ρ‚Π½ΠΎ ΠΊ Π΄Ρ€ΡƒΠ³ΠΎΠΉ ΠΊΠ»Π΅ΠΌΠΌΠ΅ Π±Π°Ρ‚Π°Ρ€Π΅ΠΈ. Если ΠΌΡ‹ рассмотрим ΠΏΡ€ΠΎΡ‚Π΅ΠΊΠ°Π½ΠΈΠ΅ ΠΏΠΎΠ»ΠΎΠΆΠΈΡ‚Π΅Π»ΡŒΠ½ΠΎΠ³ΠΎ (ΠΎΠ±Ρ‹Ρ‡Π½ΠΎΠ³ΠΎ) Ρ‚ΠΎΠΊΠ°, ΠΏΠΎΠ»ΠΎΠΆΠΈΡ‚Π΅Π»ΡŒΠ½Ρ‹Π΅ заряды ΠΏΠΎΠΊΠΈΠ΄Π°ΡŽΡ‚ ΠΏΠΎΠ»ΠΎΠΆΠΈΡ‚Π΅Π»ΡŒΠ½Ρ‹ΠΉ Π²Ρ‹Π²ΠΎΠ΄, проходят Ρ‡Π΅Ρ€Π΅Π· Π»Π°ΠΌΠΏΡƒ ΠΈ ΠΏΠΎΠΏΠ°Π΄Π°ΡŽΡ‚ Π² ΠΎΡ‚Ρ€ΠΈΡ†Π°Ρ‚Π΅Π»ΡŒΠ½Ρ‹ΠΉ Π²Ρ‹Π²ΠΎΠ΄.

ΠŸΠΎΠ»ΠΎΠΆΠΈΡ‚Π΅Π»ΡŒΠ½Ρ‹ΠΉ Ρ‚ΠΎΠΊ ΠΈΡΠΏΠΎΠ»ΡŒΠ·ΡƒΠ΅Ρ‚ΡΡ для большСй части Π°Π½Π°Π»ΠΈΠ·Π° схСм Π² этой Π³Π»Π°Π²Π΅, Π½ΠΎ Π² мСталличСских ΠΏΡ€ΠΎΠ²ΠΎΠ΄Π°Ρ… ΠΈ рСзисторах наибольший Π²ΠΊΠ»Π°Π΄ Π² Ρ‚ΠΎΠΊ вносят элСктроны, ΠΏΡ€ΠΎΡ‚Π΅ΠΊΠ°ΡŽΡ‰ΠΈΠ΅ Π² Π½Π°ΠΏΡ€Π°Π²Π»Π΅Π½ΠΈΠΈ, ΠΏΡ€ΠΎΡ‚ΠΈΠ²ΠΎΠΏΠΎΠ»ΠΎΠΆΠ½ΠΎΠΌ ΠΏΠΎΠ»ΠΎΠΆΠΈΡ‚Π΅Π»ΡŒΠ½ΠΎΠΌΡƒ ΠΏΠΎΡ‚ΠΎΠΊΡƒ Ρ‚ΠΎΠΊΠ°.ΠŸΠΎΡΡ‚ΠΎΠΌΡƒ Π±ΠΎΠ»Π΅Π΅ рСалистично Ρ€Π°ΡΡΠΌΠ°Ρ‚Ρ€ΠΈΠ²Π°Ρ‚ΡŒ Π΄Π²ΠΈΠΆΠ΅Π½ΠΈΠ΅ элСктронов для Π°Π½Π°Π»ΠΈΠ·Π° схСмы Π½Π° рисункС \ (\ PageIndex {2} \). Π­Π»Π΅ΠΊΡ‚Ρ€ΠΎΠ½Ρ‹ ΠΏΠΎΠΊΠΈΠ΄Π°ΡŽΡ‚ ΠΎΡ‚Ρ€ΠΈΡ†Π°Ρ‚Π΅Π»ΡŒΠ½ΡƒΡŽ ΠΊΠ»Π΅ΠΌΠΌΡƒ, проходят Ρ‡Π΅Ρ€Π΅Π· Π»Π°ΠΌΠΏΡƒ ΠΈ Π²ΠΎΠ·Π²Ρ€Π°Ρ‰Π°ΡŽΡ‚ΡΡ ΠΊ ΠΏΠΎΠ»ΠΎΠΆΠΈΡ‚Π΅Π»ΡŒΠ½ΠΎΠΉ ΠΊΠ»Π΅ΠΌΠΌΠ΅. Π§Ρ‚ΠΎΠ±Ρ‹ источник Π­Π”Π‘ ΠΏΠΎΠ΄Π΄Π΅Ρ€ΠΆΠΈΠ²Π°Π» Ρ€Π°Π·Π½ΠΎΡΡ‚ΡŒ ΠΏΠΎΡ‚Π΅Π½Ρ†ΠΈΠ°Π»ΠΎΠ² ΠΌΠ΅ΠΆΠ΄Ρƒ двумя Π²Ρ‹Π²ΠΎΠ΄Π°ΠΌΠΈ, ΠΎΡ‚Ρ€ΠΈΡ†Π°Ρ‚Π΅Π»ΡŒΠ½Ρ‹Π΅ заряды (элСктроны) Π΄ΠΎΠ»ΠΆΠ½Ρ‹ ΠΏΠ΅Ρ€Π΅ΠΌΠ΅Ρ‰Π°Ρ‚ΡŒΡΡ с ΠΏΠΎΠ»ΠΎΠΆΠΈΡ‚Π΅Π»ΡŒΠ½ΠΎΠ³ΠΎ Π²Ρ‹Π²ΠΎΠ΄Π° Π½Π° ΠΎΡ‚Ρ€ΠΈΡ†Π°Ρ‚Π΅Π»ΡŒΠ½Ρ‹ΠΉ. Π˜ΡΡ‚ΠΎΡ‡Π½ΠΈΠΊ Π­Π”Π‘ дСйствуСт ΠΊΠ°ΠΊ Π½Π°ΠΊΠ°Ρ‡ΠΊΠ° заряда, пСрСмСщая ΠΎΡ‚Ρ€ΠΈΡ†Π°Ρ‚Π΅Π»ΡŒΠ½Ρ‹Π΅ заряды ΠΎΡ‚ ΠΏΠΎΠ»ΠΎΠΆΠΈΡ‚Π΅Π»ΡŒΠ½ΠΎΠ³ΠΎ Π²Ρ‹Π²ΠΎΠ΄Π° ΠΊ ΠΎΡ‚Ρ€ΠΈΡ†Π°Ρ‚Π΅Π»ΡŒΠ½ΠΎΠΌΡƒ для поддСрТания разности ΠΏΠΎΡ‚Π΅Π½Ρ†ΠΈΠ°Π»ΠΎΠ².Π­Ρ‚ΠΎ ΡƒΠ²Π΅Π»ΠΈΡ‡ΠΈΠ²Π°Π΅Ρ‚ ΠΏΠΎΡ‚Π΅Π½Ρ†ΠΈΠ°Π»ΡŒΠ½ΡƒΡŽ ΡΠ½Π΅Ρ€Π³ΠΈΡŽ зарядов ΠΈ, ΡΠ»Π΅Π΄ΠΎΠ²Π°Ρ‚Π΅Π»ΡŒΠ½ΠΎ, элСктричСский ΠΏΠΎΡ‚Π΅Π½Ρ†ΠΈΠ°Π» зарядов.

Π‘ΠΈΠ»Π°, Π΄Π΅ΠΉΡΡ‚Π²ΡƒΡŽΡ‰Π°Ρ Π½Π° ΠΎΡ‚Ρ€ΠΈΡ†Π°Ρ‚Π΅Π»ΡŒΠ½Ρ‹ΠΉ заряд элСктричСского поля, дСйствуСт Π² Π½Π°ΠΏΡ€Π°Π²Π»Π΅Π½ΠΈΠΈ, ΠΏΡ€ΠΎΡ‚ΠΈΠ²ΠΎΠΏΠΎΠ»ΠΎΠΆΠ½ΠΎΠΌ элСктричСскому полю, ΠΊΠ°ΠΊ ΠΏΠΎΠΊΠ°Π·Π°Π½ΠΎ Π½Π° рисункС \ (\ PageIndex {2} \). Π§Ρ‚ΠΎΠ±Ρ‹ ΠΎΡ‚Ρ€ΠΈΡ†Π°Ρ‚Π΅Π»ΡŒΠ½Ρ‹Π΅ заряды ΠΏΠ΅Ρ€Π΅ΠΌΠ΅ΡΡ‚ΠΈΠ»ΠΈΡΡŒ Π½Π° ΠΎΡ‚Ρ€ΠΈΡ†Π°Ρ‚Π΅Π»ΡŒΠ½Ρ‹ΠΉ Π²Ρ‹Π²ΠΎΠ΄, Π½Π΅ΠΎΠ±Ρ…ΠΎΠ΄ΠΈΠΌΠΎ провСсти Ρ€Π°Π±ΠΎΡ‚Ρƒ с ΠΎΡ‚Ρ€ΠΈΡ†Π°Ρ‚Π΅Π»ΡŒΠ½Ρ‹ΠΌΠΈ зарядами. Для этого трСбуСтся энСргия, которая Π²ΠΎΠ·Π½ΠΈΠΊΠ°Π΅Ρ‚ Π² Ρ€Π΅Π·ΡƒΠ»ΡŒΡ‚Π°Ρ‚Π΅ химичСских Ρ€Π΅Π°ΠΊΡ†ΠΈΠΉ Π² Π±Π°Ρ‚Π°Ρ€Π΅Π΅. ΠŸΠΎΡ‚Π΅Π½Ρ†ΠΈΠ°Π» поддСрТиваСтся высоким Π½Π° ΠΏΠΎΠ»ΠΎΠΆΠΈΡ‚Π΅Π»ΡŒΠ½ΠΎΠΉ ΠΊΠ»Π΅ΠΌΠΌΠ΅ ΠΈ Π½ΠΈΠ·ΠΊΠΈΠΌ Π½Π° ΠΎΡ‚Ρ€ΠΈΡ†Π°Ρ‚Π΅Π»ΡŒΠ½ΠΎΠΉ ΠΊΠ»Π΅ΠΌΠΌΠ΅, Ρ‡Ρ‚ΠΎΠ±Ρ‹ ΠΏΠΎΠ΄Π΄Π΅Ρ€ΠΆΠΈΠ²Π°Ρ‚ΡŒ Ρ€Π°Π·Π½ΠΎΡΡ‚ΡŒ ΠΏΠΎΡ‚Π΅Π½Ρ†ΠΈΠ°Π»ΠΎΠ² ΠΌΠ΅ΠΆΠ΄Ρƒ двумя ΠΊΠ»Π΅ΠΌΠΌΠ°ΠΌΠΈ.Π­Π”Π‘ Ρ€Π°Π²Π½Π° Ρ€Π°Π±ΠΎΡ‚Π΅, выполняСмой Π½Π°Π΄ зарядом Π½Π° Π΅Π΄ΠΈΠ½ΠΈΡ†Ρƒ заряда \ (\ left (\ epsilon = \ frac {dW} {dq} \ right) \) ΠΏΡ€ΠΈ отсутствии Ρ‚ΠΎΠΊΠ°. ΠŸΠΎΡΠΊΠΎΠ»ΡŒΠΊΡƒ Π΅Π΄ΠΈΠ½ΠΈΡ†Π΅ΠΉ Ρ€Π°Π±ΠΎΡ‚Ρ‹ являСтся Π΄ΠΆΠΎΡƒΠ»ΡŒ, Π° Π΅Π΄ΠΈΠ½ΠΈΡ†Π΅ΠΉ заряда – ΠΊΡƒΠ»ΠΎΠ½, Π΅Π΄ΠΈΠ½ΠΈΡ†Π΅ΠΉ измСрСния Π­Π”Π‘ являСтся Π²ΠΎΠ»ΡŒΡ‚ \ ((1 \, V = 1 \, J / C) \).

НапряТСниС Π½Π° ΠΊΠ»Π΅ΠΌΠΌΠ°Ρ… \ (V_ {ΠΊΠ»Π΅ΠΌΠΌΠ°} \) Π±Π°Ρ‚Π°Ρ€Π΅ΠΈ – это напряТСниС, ΠΈΠ·ΠΌΠ΅Ρ€Π΅Π½Π½ΠΎΠ΅ Π½Π° ΠΊΠ»Π΅ΠΌΠΌΠ°Ρ… Π±Π°Ρ‚Π°Ρ€Π΅ΠΈ, ΠΊΠΎΠ³Π΄Π° ΠΊ ΠΊΠ»Π΅ΠΌΠΌΠ΅ Π½Π΅ ΠΏΠΎΠ΄ΠΊΠ»ΡŽΡ‡Π΅Π½Π° Π½Π°Π³Ρ€ΡƒΠ·ΠΊΠ°. ИдСальная батарСя – это источник Π­Π”Π‘, ΠΊΠΎΡ‚ΠΎΡ€Ρ‹ΠΉ ΠΏΠΎΠ΄Π΄Π΅Ρ€ΠΆΠΈΠ²Π°Π΅Ρ‚ постоянноС напряТСниС Π½Π° ΠΊΠ»Π΅ΠΌΠΌΠ°Ρ…, нСзависимо ΠΎΡ‚ Ρ‚ΠΎΠΊΠ° ΠΌΠ΅ΠΆΠ΄Ρƒ двумя ΠΊΠ»Π΅ΠΌΠΌΠ°ΠΌΠΈ.ИдСальная батарСя Π½Π΅ ΠΈΠΌΠ΅Π΅Ρ‚ Π²Π½ΡƒΡ‚Ρ€Π΅Π½Π½Π΅Π³ΠΎ сопротивлСния, Π° напряТСниС Π½Π° ΠΊΠ»Π΅ΠΌΠΌΠ°Ρ… Ρ€Π°Π²Π½ΠΎ Π­Π”Π‘ Π±Π°Ρ‚Π°Ρ€Π΅ΠΈ. Π’ ΡΠ»Π΅Π΄ΡƒΡŽΡ‰Π΅ΠΌ Ρ€Π°Π·Π΄Π΅Π»Π΅ ΠΌΡ‹ ΠΏΠΎΠΊΠ°ΠΆΠ΅ΠΌ, Ρ‡Ρ‚ΠΎ Ρƒ Ρ€Π΅Π°Π»ΡŒΠ½ΠΎΠΉ Π±Π°Ρ‚Π°Ρ€Π΅ΠΈ Π΅ΡΡ‚ΡŒ Π²Π½ΡƒΡ‚Ρ€Π΅Π½Π½Π΅Π΅ сопротивлСниС, Π° напряТСниС Π½Π° ΠΊΠ»Π΅ΠΌΠΌΠ°Ρ… всСгда мСньшС, Ρ‡Π΅ΠΌ Π­Π”Π‘ Π±Π°Ρ‚Π°Ρ€Π΅ΠΈ.

Π˜ΡΡ‚ΠΎΡ‡Π½ΠΈΠΊ ΠΏΠΎΡ‚Π΅Π½Ρ†ΠΈΠ°Π»Π° Π±Π°Ρ‚Π°Ρ€Π΅ΠΈ

Π­Π”Π‘ Π±Π°Ρ‚Π°Ρ€Π΅ΠΈ опрСдСляСтся сочСтаниСм химичСских вСщСств ΠΈ составом Π²Ρ‹Π²ΠΎΠ΄ΠΎΠ² Π±Π°Ρ‚Π°Ρ€Π΅ΠΈ. Π‘Π²ΠΈΠ½Ρ†ΠΎΠ²ΠΎ-кислотный аккумулятор , ΠΈΡΠΏΠΎΠ»ΡŒΠ·ΡƒΠ΅ΠΌΡ‹ΠΉ Π² автомобилях ΠΈ Π΄Ρ€ΡƒΠ³ΠΈΡ… транспортных срСдствах, являСтся ΠΎΠ΄Π½ΠΈΠΌ ΠΈΠ· Π½Π°ΠΈΠ±ΠΎΠ»Π΅Π΅ распространСнных сочСтаний химичСских вСщСств.На рисункС \ (\ PageIndex {3} \) ΠΏΠΎΠΊΠ°Π·Π°Π½Π° ΠΎΠ΄Π½Π° ячСйка (ΠΎΠ΄Π½Π° ΠΈΠ· ΡˆΠ΅ΡΡ‚ΠΈ) этой Π±Π°Ρ‚Π°Ρ€Π΅ΠΈ. ΠšΠ°Ρ‚ΠΎΠ΄Π½Π°Ρ (ΠΏΠΎΠ»ΠΎΠΆΠΈΡ‚Π΅Π»ΡŒΠ½Π°Ρ) ΠΊΠ»Π΅ΠΌΠΌΠ° ячСйки соСдинСна с пластиной ΠΈΠ· оксида свинца, Π° анодная (ΠΎΡ‚Ρ€ΠΈΡ†Π°Ρ‚Π΅Π»ΡŒΠ½Π°Ρ) ΠΊΠ»Π΅ΠΌΠΌΠ° ΠΏΠΎΠ΄ΠΊΠ»ΡŽΡ‡Π΅Π½Π° ΠΊ свинцовой пластинС. ОбС пластины ΠΏΠΎΠ³Ρ€ΡƒΠΆΠ΅Π½Ρ‹ Π² ΡΠ΅Ρ€Π½ΡƒΡŽ кислоту, элСктролит для систСмы.

Рисунок \ (\ PageIndex {3} \): Π₯имичСскиС Ρ€Π΅Π°ΠΊΡ†ΠΈΠΈ Π² свинцово-кислотном элСмСнтС Ρ€Π°Π·Π΄Π΅Π»ΡΡŽΡ‚ заряд, отправляя ΠΎΡ‚Ρ€ΠΈΡ†Π°Ρ‚Π΅Π»ΡŒΠ½Ρ‹ΠΉ заряд Π½Π° Π°Π½ΠΎΠ΄, ΠΊΠΎΡ‚ΠΎΡ€Ρ‹ΠΉ соСдинСн со свинцовыми пластинами. ΠŸΠ»Π°ΡΡ‚ΠΈΠ½Ρ‹ ΠΈΠ· оксида свинца ΠΏΠΎΠ΄ΠΊΠ»ΡŽΡ‡Π°ΡŽΡ‚ΡΡ ΠΊ ΠΏΠΎΠ»ΠΎΠΆΠΈΡ‚Π΅Π»ΡŒΠ½ΠΎΠΌΡƒ ΠΈΠ»ΠΈ ΠΊΠ°Ρ‚ΠΎΠ΄Π½ΠΎΠΌΡƒ Π²Ρ‹Π²ΠΎΠ΄Ρƒ ячСйки.БСрная кислота ΠΏΡ€ΠΎΠ²ΠΎΠ΄ΠΈΡ‚ заряд, Π° Ρ‚Π°ΠΊΠΆΠ΅ участвуСт Π² химичСской Ρ€Π΅Π°ΠΊΡ†ΠΈΠΈ.

НСбольшоС Π·Π½Π°Π½ΠΈΠ΅ Ρ‚ΠΎΠ³ΠΎ, ΠΊΠ°ΠΊ Π²Π·Π°ΠΈΠΌΠΎΠ΄Π΅ΠΉΡΡ‚Π²ΡƒΡŽΡ‚ химичСскиС вСщСства Π² свинцово-кислотной Π±Π°Ρ‚Π°Ρ€Π΅Π΅, ΠΏΠΎΠΌΠΎΠ³Π°Π΅Ρ‚ ΠΏΠΎΠ½ΡΡ‚ΡŒ ΠΏΠΎΡ‚Π΅Π½Ρ†ΠΈΠ°Π», создаваСмый Π±Π°Ρ‚Π°Ρ€Π΅Π΅ΠΉ. На рисункС \ (\ PageIndex {4} \) ΠΏΠΎΠΊΠ°Π·Π°Π½ Ρ€Π΅Π·ΡƒΠ»ΡŒΡ‚Π°Ρ‚ ΠΎΠ΄Π½ΠΎΠΉ химичСской Ρ€Π΅Π°ΠΊΡ†ΠΈΠΈ. Π”Π²Π° элСктрона ΠΏΠΎΠΌΠ΅Ρ‰Π°ΡŽΡ‚ΡΡ Π½Π° Π°Π½ΠΎΠ΄ , Ρ‡Ρ‚ΠΎ Π΄Π΅Π»Π°Π΅Ρ‚ Π΅Π³ΠΎ ΠΎΡ‚Ρ€ΠΈΡ†Π°Ρ‚Π΅Π»ΡŒΠ½Ρ‹ΠΌ, ΠΏΡ€ΠΈ условии, Ρ‡Ρ‚ΠΎ ΠΊΠ°Ρ‚ΠΎΠ΄ ΠΏΠΎΠ΄Π°Π΅Ρ‚ Π΄Π²Π° элСктрона. Π­Ρ‚ΠΎ оставляСт ΠΊΠ°Ρ‚ΠΎΠ΄ ΠΏΠΎΠ»ΠΎΠΆΠΈΡ‚Π΅Π»ΡŒΠ½ΠΎ заряТСнным, ΠΏΠΎΡ‚ΠΎΠΌΡƒ Ρ‡Ρ‚ΠΎ ΠΎΠ½ потСрял Π΄Π²Π° элСктрона.ΠšΠΎΡ€ΠΎΡ‡Π΅ говоря, Ρ€Π°Π·Π΄Π΅Π»Π΅Π½ΠΈΠ΅ заряда Π±Ρ‹Π»ΠΎ Π²Ρ‹Π·Π²Π°Π½ΠΎ химичСской Ρ€Π΅Π°ΠΊΡ†ΠΈΠ΅ΠΉ.

ΠžΠ±Ρ€Π°Ρ‚ΠΈΡ‚Π΅ Π²Π½ΠΈΠΌΠ°Π½ΠΈΠ΅, Ρ‡Ρ‚ΠΎ рСакция Π½Π΅ происходит, Ссли Π½Π΅Ρ‚ Π·Π°ΠΌΠΊΠ½ΡƒΡ‚ΠΎΠΉ Ρ†Π΅ΠΏΠΈ, ΠΏΠΎΠ·Π²ΠΎΠ»ΡΡŽΡ‰Π΅ΠΉ ΠΏΠΎΠ΄Π°Π²Π°Ρ‚ΡŒ Π΄Π²Π° элСктрона Π½Π° ΠΊΠ°Ρ‚ΠΎΠ΄. Π’ΠΎ ΠΌΠ½ΠΎΠ³ΠΈΡ… случаях эти элСктроны выходят ΠΈΠ· Π°Π½ΠΎΠ΄Π°, проходят Ρ‡Π΅Ρ€Π΅Π· сопротивлСниС ΠΈ Π²ΠΎΠ·Π²Ρ€Π°Ρ‰Π°ΡŽΡ‚ΡΡ Π½Π° ΠΊΠ°Ρ‚ΠΎΠ΄. ΠžΡ‚ΠΌΠ΅Ρ‚ΠΈΠΌ Ρ‚Π°ΠΊΠΆΠ΅, Ρ‡Ρ‚ΠΎ, ΠΏΠΎΡΠΊΠΎΠ»ΡŒΠΊΡƒ Π² химичСских рСакциях ΡƒΡ‡Π°ΡΡ‚Π²ΡƒΡŽΡ‚ вСщСства, ΠΎΠ±Π»Π°Π΄Π°ΡŽΡ‰ΠΈΠ΅ сопротивлСниСм, Π½Π΅Π²ΠΎΠ·ΠΌΠΎΠΆΠ½ΠΎ ΡΠΎΠ·Π΄Π°Ρ‚ΡŒ Π­Π”Π‘ Π±Π΅Π· Π²Π½ΡƒΡ‚Ρ€Π΅Π½Π½Π΅Π³ΠΎ сопротивлСния.

Рисунок \ (\ PageIndex {4} \): Π’ свинцово-кислотной Π±Π°Ρ‚Π°Ρ€Π΅Π΅ Π΄Π²Π° элСктрона ΠΏΡ€ΠΈΠΆΠΈΠΌΠ°ΡŽΡ‚ΡΡ ΠΊ Π°Π½ΠΎΠ΄Ρƒ элСмСнта, Π° Π΄Π²Π° элСктрона ΡƒΠ΄Π°Π»ΡΡŽΡ‚ΡΡ с ΠΊΠ°Ρ‚ΠΎΠ΄Π° элСмСнта.Π’ Ρ€Π΅Π·ΡƒΠ»ΡŒΡ‚Π°Ρ‚Π΅ химичСской Ρ€Π΅Π°ΠΊΡ†ΠΈΠΈ Π² свинцово-кислотной Π±Π°Ρ‚Π°Ρ€Π΅Π΅ Π΄Π²Π° элСктрона ΠΏΠΎΠΌΠ΅Ρ‰Π°ΡŽΡ‚ΡΡ Π½Π° Π°Π½ΠΎΠ΄ ΠΈ Π΄Π²Π° элСктрона ΡƒΠ΄Π°Π»ΡΡŽΡ‚ΡΡ с ΠΊΠ°Ρ‚ΠΎΠ΄Π°. Для Ρ€Π°Π±ΠΎΡ‚Ρ‹ трСбуСтся замкнутая Ρ†Π΅ΠΏΡŒ, Ρ‚Π°ΠΊ ΠΊΠ°ΠΊ Π΄Π²Π° элСктрона Π΄ΠΎΠ»ΠΆΠ½Ρ‹ Π±Ρ‹Ρ‚ΡŒ доставлСны Π½Π° ΠΊΠ°Ρ‚ΠΎΠ΄.

Π’Π½ΡƒΡ‚Ρ€Π΅Π½Π½Π΅Π΅ сопротивлСниС ΠΈ напряТСниС Π½Π° ΠΊΠ»Π΅ΠΌΠΌΠ°Ρ…

Π’Π΅Π»ΠΈΡ‡ΠΈΠ½Π° сопротивлСния Ρ‚ΠΎΠΊΡƒ Π²Π½ΡƒΡ‚Ρ€ΠΈ источника напряТСния называСтся Π²Π½ΡƒΡ‚Ρ€Π΅Π½Π½ΠΈΠΌ сопротивлСниСм . Π’Π½ΡƒΡ‚Ρ€Π΅Π½Π½Π΅Π΅ сопротивлСниС Π±Π°Ρ‚Π°Ρ€Π΅ΠΈ r ΠΌΠΎΠΆΠ΅Ρ‚ вСсти сСбя слоТным ΠΎΠ±Ρ€Π°Π·ΠΎΠΌ. ΠžΠ±Ρ‹Ρ‡Π½ΠΎ ΠΎΠ½Π° увСличиваСтся ΠΏΠΎ ΠΌΠ΅Ρ€Π΅ разряда Π±Π°Ρ‚Π°Ρ€Π΅ΠΈ ΠΈΠ·-Π·Π° окислСния пластин ΠΈΠ»ΠΈ сниТСния кислотности элСктролита.Однако Π²Π½ΡƒΡ‚Ρ€Π΅Π½Π½Π΅Π΅ сопротивлСниС Ρ‚Π°ΠΊΠΆΠ΅ ΠΌΠΎΠΆΠ΅Ρ‚ Π·Π°Π²ΠΈΡΠ΅Ρ‚ΡŒ ΠΎΡ‚ Π²Π΅Π»ΠΈΡ‡ΠΈΠ½Ρ‹ ΠΈ направлСния Ρ‚ΠΎΠΊΠ° Ρ‡Π΅Ρ€Π΅Π· источник напряТСния, Π΅Π³ΠΎ Ρ‚Π΅ΠΌΠΏΠ΅Ρ€Π°Ρ‚ΡƒΡ€Ρ‹ ΠΈ Π΄Π°ΠΆΠ΅ Π΅Π³ΠΎ прСдыстории. НапримСр, Π²Π½ΡƒΡ‚Ρ€Π΅Π½Π½Π΅Π΅ сопротивлСниС пСрСзаряТаСмых никСль-ΠΊΠ°Π΄ΠΌΠΈΠ΅Π²Ρ‹Ρ… элСмСнтов зависит ΠΎΡ‚ Ρ‚ΠΎΠ³ΠΎ, сколько Ρ€Π°Π· ΠΈ насколько Π³Π»ΡƒΠ±ΠΎΠΊΠΎ ΠΎΠ½ΠΈ Π±Ρ‹Π»ΠΈ разряТСны. ΠŸΡ€ΠΎΡΡ‚Π°Ρ модСль Π±Π°Ρ‚Π°Ρ€Π΅ΠΈ состоит ΠΈΠ· ΠΈΠ΄Π΅Π°Π»ΠΈΠ·ΠΈΡ€ΠΎΠ²Π°Π½Π½ΠΎΠ³ΠΎ источника Π­Π”Π‘ \ (\ epsilon \) ΠΈ Π²Π½ΡƒΡ‚Ρ€Π΅Π½Π½Π΅Π³ΠΎ сопротивлСния r (рисунок \ (\ PageIndex {5} \)).

Рисунок \ (\ PageIndex {5} \): Π‘Π°Ρ‚Π°Ρ€Π΅ΡŽ ΠΌΠΎΠΆΠ½ΠΎ ΡΠΌΠΎΠ΄Π΅Π»ΠΈΡ€ΠΎΠ²Π°Ρ‚ΡŒ ΠΊΠ°ΠΊ ΠΈΠ΄Π΅Π°Π»ΠΈΠ·ΠΈΡ€ΠΎΠ²Π°Π½Π½ΡƒΡŽ Π­Π”Π‘ \ ((\ epsilon) \) с Π²Π½ΡƒΡ‚Ρ€Π΅Π½Π½ΠΈΠΌ сопротивлСниСм ( r ).НапряТСниС Π½Π° ΠΊΠ»Π΅ΠΌΠΌΠ°Ρ… аккумулятора Ρ€Π°Π²Π½ΠΎ \ (V_ {terminal} = \ epsilon – Ir \).

ΠŸΡ€Π΅Π΄ΠΏΠΎΠ»ΠΎΠΆΠΈΠΌ, Ρ‡Ρ‚ΠΎ внСшний рСзистор, извСстный ΠΊΠ°ΠΊ сопротивлСниС Π½Π°Π³Ρ€ΡƒΠ·ΠΊΠΈ R , ΠΏΠΎΠ΄ΠΊΠ»ΡŽΡ‡Π΅Π½ ΠΊ источнику напряТСния, Π½Π°ΠΏΡ€ΠΈΠΌΠ΅Ρ€ Π±Π°Ρ‚Π°Ρ€Π΅Π΅, ΠΊΠ°ΠΊ ΠΏΠΎΠΊΠ°Π·Π°Π½ΠΎ Π½Π° рисункС \ (\ PageIndex {6} \). На рисункС ΠΏΠΎΠΊΠ°Π·Π°Π½Π° модСль Π±Π°Ρ‚Π°Ρ€Π΅ΠΈ с Π­Π”Π‘ Ξ΅, Π²Π½ΡƒΡ‚Ρ€Π΅Π½Π½ΠΈΠΌ сопротивлСниСм R ΠΈ Π½Π°Π³Ρ€ΡƒΠ·ΠΎΡ‡Π½Ρ‹ΠΌ рСзистором R , ΠΏΠΎΠ΄ΠΊΠ»ΡŽΡ‡Π΅Π½Π½Ρ‹ΠΌ ΠΊ Π΅Π΅ ΠΊΠ»Π΅ΠΌΠΌΠ°ΠΌ. ΠŸΡ€ΠΈ ΠΎΠ±Ρ‹Ρ‡Π½ΠΎΠΌ ΠΏΡ€ΠΎΡ‚Π΅ΠΊΠ°Π½ΠΈΠΈ Ρ‚ΠΎΠΊΠ° ΠΏΠΎΠ»ΠΎΠΆΠΈΡ‚Π΅Π»ΡŒΠ½Ρ‹Π΅ заряды ΠΏΠΎΠΊΠΈΠ΄Π°ΡŽΡ‚ ΠΏΠΎΠ»ΠΎΠΆΠΈΡ‚Π΅Π»ΡŒΠ½ΡƒΡŽ ΠΊΠ»Π΅ΠΌΠΌΡƒ Π±Π°Ρ‚Π°Ρ€Π΅ΠΈ, проходят Ρ‡Π΅Ρ€Π΅Π· рСзистор ΠΈ Π²ΠΎΠ·Π²Ρ€Π°Ρ‰Π°ΡŽΡ‚ΡΡ ΠΊ ΠΎΡ‚Ρ€ΠΈΡ†Π°Ρ‚Π΅Π»ΡŒΠ½ΠΎΠΉ ΠΊΠ»Π΅ΠΌΠΌΠ΅ Π±Π°Ρ‚Π°Ρ€Π΅ΠΈ.НапряТСниС Π½Π° ΠΊΠ»Π΅ΠΌΠΌΠ°Ρ… аккумулятора зависит ΠΎΡ‚ Π­Π”Π‘, Π²Π½ΡƒΡ‚Ρ€Π΅Π½Π½Π΅Π³ΠΎ сопротивлСния ΠΈ силы Ρ‚ΠΎΠΊΠ° ΠΈ Ρ€Π°Π²Π½ΠΎ

.

ΠŸΡ€ΠΈΠΌΠ΅Ρ‡Π°Π½ΠΈΠ΅

\ [V_ {Ρ‚Π΅Ρ€ΠΌΠΈΠ½Π°Π»} = \ epsilon – Ir \]

ΠŸΡ€ΠΈ Π·Π°Π΄Π°Π½Π½ΠΎΠΉ Π­Π”Π‘ ΠΈ Π²Π½ΡƒΡ‚Ρ€Π΅Π½Π½Π΅ΠΌ сопротивлСнии напряТСниС Π½Π° ΠΊΠ»Π΅ΠΌΠΌΠ°Ρ… ΡƒΠΌΠ΅Π½ΡŒΡˆΠ°Π΅Ρ‚ΡΡ ΠΏΠΎ ΠΌΠ΅Ρ€Π΅ увСличСния Ρ‚ΠΎΠΊΠ° ΠΈΠ·-Π·Π° падСния ΠΏΠΎΡ‚Π΅Π½Ρ†ΠΈΠ°Π»Π° Ir Π²Π½ΡƒΡ‚Ρ€Π΅Π½Π½Π΅Π³ΠΎ сопротивлСния.

Рисунок \ (\ PageIndex {6} \): Π‘Ρ…Π΅ΠΌΠ° источника напряТСния ΠΈ Π΅Π³ΠΎ Π½Π°Π³Ρ€ΡƒΠ·ΠΎΡ‡Π½ΠΎΠ³ΠΎ рСзистора R . ΠŸΠΎΡΠΊΠΎΠ»ΡŒΠΊΡƒ Π²Π½ΡƒΡ‚Ρ€Π΅Π½Π½Π΅Π΅ сопротивлСниС r ΠΏΠΎΡΠ»Π΅Π΄ΠΎΠ²Π°Ρ‚Π΅Π»ΡŒΠ½ΠΎ с Π½Π°Π³Ρ€ΡƒΠ·ΠΊΠΎΠΉ, ΠΎΠ½ΠΎ ΠΌΠΎΠΆΠ΅Ρ‚ сущСствСнно ΠΏΠΎΠ²Π»ΠΈΡΡ‚ΡŒ Π½Π° напряТСниС Π½Π° ΠΊΠ»Π΅ΠΌΠΌΠ°Ρ… ΠΈ Ρ‚ΠΎΠΊ, ΠΏΠΎΠ΄Π°Π²Π°Π΅ΠΌΡ‹ΠΉ Π½Π° Π½Π°Π³Ρ€ΡƒΠ·ΠΊΡƒ.

Π“Ρ€Π°Ρ„ΠΈΠΊ разности ΠΏΠΎΡ‚Π΅Π½Ρ†ΠΈΠ°Π»ΠΎΠ² Π½Π° ΠΊΠ°ΠΆΠ΄ΠΎΠΌ элСмСнтС Ρ†Π΅ΠΏΠΈ ΠΏΠΎΠΊΠ°Π·Π°Π½ Π½Π° рисункС \ (\ PageIndex {7} \). По Ρ†Π΅ΠΏΠΈ ΠΏΡ€ΠΎΡ…ΠΎΠ΄ΠΈΡ‚ Ρ‚ΠΎΠΊ I , Π° ΠΏΠ°Π΄Π΅Π½ΠΈΠ΅ ΠΏΠΎΡ‚Π΅Π½Ρ†ΠΈΠ°Π»Π° Π½Π° Π²Π½ΡƒΡ‚Ρ€Π΅Π½Π½Π΅ΠΌ рСзисторС Ρ€Π°Π²Π½ΠΎ Ir . НапряТСниС Π½Π° ΠΊΠ»Π΅ΠΌΠΌΠ°Ρ… Ρ€Π°Π²Π½ΠΎ \ (\ epsilon – Ir \), Ρ‡Ρ‚ΠΎ Ρ€Π°Π²Π½ΠΎ падСнию ΠΏΠΎΡ‚Π΅Π½Ρ†ΠΈΠ°Π»Π° Π½Π° Π½Π°Π³Ρ€ΡƒΠ·ΠΎΡ‡Π½ΠΎΠΌ рСзисторС \ (IR = \ epsilon – Ir \). Как ΠΈ Π² случаС с ΠΏΠΎΡ‚Π΅Π½Ρ†ΠΈΠ°Π»ΡŒΠ½ΠΎΠΉ энСргиСй, Π²Π°ΠΆΠ½ΠΎ ΠΈΠ·ΠΌΠ΅Π½Π΅Π½ΠΈΠ΅ напряТСния. Когда ΠΈΡΠΏΠΎΠ»ΡŒΠ·ΡƒΠ΅Ρ‚ΡΡ Ρ‚Π΅Ρ€ΠΌΠΈΠ½ «напряТСниС», ΠΌΡ‹ ΠΏΡ€Π΅Π΄ΠΏΠΎΠ»Π°Π³Π°Π΅ΠΌ, Ρ‡Ρ‚ΠΎ это Π½Π° самом Π΄Π΅Π»Π΅ ΠΈΠ·ΠΌΠ΅Π½Π΅Π½ΠΈΠ΅ ΠΏΠΎΡ‚Π΅Π½Ρ†ΠΈΠ°Π»Π°, ΠΈΠ»ΠΈ \ (\ Delta V \).Однако \ (\ Delta \) часто для удобства опускаСтся.

Рисунок \ (\ PageIndex {7} \): Π³Ρ€Π°Ρ„ΠΈΠΊ напряТСния Π² Ρ†Π΅ΠΏΠΈ Π±Π°Ρ‚Π°Ρ€Π΅ΠΈ ΠΈ сопротивлСния Π½Π°Π³Ρ€ΡƒΠ·ΠΊΠΈ. ЭлСктричСский ΠΏΠΎΡ‚Π΅Π½Ρ†ΠΈΠ°Π» ΡƒΠ²Π΅Π»ΠΈΡ‡ΠΈΠ²Π°Π΅Ρ‚ Π­Π”Π‘ Π±Π°Ρ‚Π°Ρ€Π΅ΠΈ ΠΈΠ·-Π·Π° химичСских Ρ€Π΅Π°ΠΊΡ†ΠΈΠΉ, Π²Ρ‹ΠΏΠΎΠ»Π½ΡΡŽΡ‰ΠΈΡ… Ρ€Π°Π±ΠΎΡ‚Ρƒ с зарядами. Π’ аккумуляторС происходит сниТСниС элСктричСского ΠΏΠΎΡ‚Π΅Π½Ρ†ΠΈΠ°Π»Π° ΠΈΠ·-Π·Π° Π²Π½ΡƒΡ‚Ρ€Π΅Π½Π½Π΅Π³ΠΎ сопротивлСния. ΠŸΠΎΡ‚Π΅Π½Ρ†ΠΈΠ°Π» ΡƒΠΌΠ΅Π½ΡŒΡˆΠ°Π΅Ρ‚ΡΡ ΠΈΠ·-Π·Π° Π²Π½ΡƒΡ‚Ρ€Π΅Π½Π½Π΅Π³ΠΎ сопротивлСния \ (- Ir \), Π² Ρ€Π΅Π·ΡƒΠ»ΡŒΡ‚Π°Ρ‚Π΅ Ρ‡Π΅Π³ΠΎ напряТСниС Π½Π° ΠΊΠ»Π΅ΠΌΠΌΠ°Ρ… Π±Π°Ρ‚Π°Ρ€Π΅ΠΈ Ρ€Π°Π²Π½ΠΎ \ ((\ epsilon – Ir) \).Π—Π°Ρ‚Π΅ΠΌ напряТСниС ΡƒΠΌΠ΅Π½ΡŒΡˆΠ°Π΅Ρ‚ΡΡ Π½Π° ( IR ). Π’ΠΎΠΊ Ρ€Π°Π²Π΅Π½ \ (I = \ frac {\ epsilon} {r + R} \).

Π’ΠΎΠΊ Ρ‡Π΅Ρ€Π΅Π· Π½Π°Π³Ρ€ΡƒΠ·ΠΎΡ‡Π½Ρ‹ΠΉ рСзистор Ρ€Π°Π²Π΅Π½ \ (I = \ frac {\ epsilon} {r + R} \). Из этого выраТСния Π²ΠΈΠ΄Π½ΠΎ, Ρ‡Ρ‚ΠΎ Ρ‡Π΅ΠΌ мСньшС Π²Π½ΡƒΡ‚Ρ€Π΅Π½Π½Π΅Π΅ сопротивлСниС r , Ρ‚Π΅ΠΌ больший Ρ‚ΠΎΠΊ источник напряТСния ΠΏΠΎΠ΄Π°Π΅Ρ‚ Π½Π° свою Π½Π°Π³Ρ€ΡƒΠ·ΠΊΡƒ R . По ΠΌΠ΅Ρ€Π΅ разряда Π±Π°Ρ‚Π°Ρ€Π΅ΠΉ r увСличиваСтся. Если r становится Π·Π½Π°Ρ‡ΠΈΡ‚Π΅Π»ΡŒΠ½ΠΎΠΉ Ρ‡Π°ΡΡ‚ΡŒΡŽ сопротивлСния Π½Π°Π³Ρ€ΡƒΠ·ΠΊΠΈ, Ρ‚ΠΎ Ρ‚ΠΎΠΊ Π·Π½Π°Ρ‡ΠΈΡ‚Π΅Π»ΡŒΠ½ΠΎ сниТаСтся, ΠΊΠ°ΠΊ ΠΏΠΎΠΊΠ°Π·Π°Π½ΠΎ Π² ΡΠ»Π΅Π΄ΡƒΡŽΡ‰Π΅ΠΌ ΠΏΡ€ΠΈΠΌΠ΅Ρ€Π΅.

ΠŸΡ€ΠΈΠΌΠ΅Ρ€ \ (\ PageIndex {1} \): Π°Π½Π°Π»ΠΈΠ· Ρ†Π΅ΠΏΠΈ с Π±Π°Ρ‚Π°Ρ€Π΅Π΅ΠΉ ΠΈ Π½Π°Π³Ρ€ΡƒΠ·ΠΊΠΎΠΉ

Данная батарСя ΠΈΠΌΠ΅Π΅Ρ‚ Π­Π”Π‘ 12,00 Π’ ΠΈ Π²Π½ΡƒΡ‚Ρ€Π΅Π½Π½Π΅Π΅ сопротивлСниС \ (0,100 \, \ ОмСга \). (a) РассчитайтС Π΅Π³ΠΎ напряТСниС Π½Π° ΠΊΠ»Π΅ΠΌΠΌΠ°Ρ… ΠΏΡ€ΠΈ ΠΏΠΎΠ΄ΠΊΠ»ΡŽΡ‡Π΅Π½ΠΈΠΈ ΠΊ Π½Π°Π³Ρ€ΡƒΠ·ΠΊΠ΅ с \ (10.00 \, \ Omega \). (b) КакоС напряТСниС Π½Π° ΠΊΠ»Π΅ΠΌΠΌΠ°Ρ… ΠΏΡ€ΠΈ ΠΏΠΎΠ΄ΠΊΠ»ΡŽΡ‡Π΅Π½ΠΈΠΈ ΠΊ Π½Π°Π³Ρ€ΡƒΠ·ΠΊΠ΅ \ (0.500 \, \ Omega \)? (c) Какая ΠΌΠΎΡ‰Π½ΠΎΡΡ‚ΡŒ рассСиваСтся ΠΏΡ€ΠΈ Π½Π°Π³Ρ€ΡƒΠ·ΠΊΠ΅ \ (0.500 \, \ Omega \)? (d) Если Π²Π½ΡƒΡ‚Ρ€Π΅Π½Π½Π΅Π΅ сопротивлСниС увСличиваСтся Π΄ΠΎ \ (0.500 \, \ Omega \), Π½Π°ΠΉΠ΄ΠΈΡ‚Π΅ Ρ‚ΠΎΠΊ, напряТСниС Π½Π° ΠΊΠ»Π΅ΠΌΠΌΠ°Ρ… ΠΈ ΠΌΠΎΡ‰Π½ΠΎΡΡ‚ΡŒ, Ρ€Π°ΡΡΠ΅ΠΈΠ²Π°Π΅ΠΌΡƒΡŽ элСмСнтом \ (0.500 \, \ Omega \) Π·Π°Π³Ρ€ΡƒΠ·ΠΊΠ°.

БтратСгия

ΠŸΡ€ΠΈΠ²Π΅Π΄Π΅Π½Π½Ρ‹ΠΉ Π²Ρ‹ΡˆΠ΅ Π°Π½Π°Π»ΠΈΠ· Π΄Π°Π» Π²Ρ‹Ρ€Π°ΠΆΠ΅Π½ΠΈΠ΅ для Ρ‚ΠΎΠΊΠ° с ΡƒΡ‡Π΅Ρ‚ΠΎΠΌ Π²Π½ΡƒΡ‚Ρ€Π΅Π½Π½Π΅Π³ΠΎ сопротивлСния. Как Ρ‚ΠΎΠ»ΡŒΠΊΠΎ Ρ‚ΠΎΠΊ Π±ΡƒΠ΄Π΅Ρ‚ Π½Π°ΠΉΠ΄Π΅Π½, напряТСниС Π½Π° ΠΊΠ»Π΅ΠΌΠΌΠ°Ρ… ΠΌΠΎΠΆΠ½ΠΎ Ρ€Π°ΡΡΡ‡ΠΈΡ‚Π°Ρ‚ΡŒ с ΠΏΠΎΠΌΠΎΡ‰ΡŒΡŽ уравнСния \ (V_ {terminal} = \ epsilon – Ir \). Как Ρ‚ΠΎΠ»ΡŒΠΊΠΎ Ρ‚ΠΎΠΊ Π±ΡƒΠ΄Π΅Ρ‚ Π½Π°ΠΉΠ΄Π΅Π½, ΠΌΡ‹ Ρ‚Π°ΠΊΠΆΠ΅ смоТСм Π½Π°ΠΉΡ‚ΠΈ ΠΌΠΎΡ‰Π½ΠΎΡΡ‚ΡŒ, Ρ€Π°ΡΡΠ΅ΠΈΠ²Π°Π΅ΠΌΡƒΡŽ рСзистором.

РСшСниС

  1. Π’Π²ΠΎΠ΄ Π·Π°Π΄Π°Π½Π½Ρ‹Ρ… Π·Π½Π°Ρ‡Π΅Π½ΠΈΠΉ Π­Π”Π‘, сопротивлСния Π½Π°Π³Ρ€ΡƒΠ·ΠΊΠΈ ΠΈ Π²Π½ΡƒΡ‚Ρ€Π΅Π½Π½Π΅Π³ΠΎ сопротивлСния Π² Π²Ρ‹Ρ€Π°ΠΆΠ΅Π½ΠΈΠ΅ Π²Ρ‹ΡˆΠ΅ Π΄Π°Π΅Ρ‚ \ [I = \ frac {\ epsilon} {R + r} = \ frac {12.00 \, V} {10.10 \, \ Omega} = 1.188 \, A. \] Π’Π²Π΅Π΄ΠΈΡ‚Π΅ извСстныС значСния Π² ΡƒΡ€Π°Π²Π½Π΅Π½ΠΈΠ΅ \ (V_ {terminal} = \ epsilon – Ir \), Ρ‡Ρ‚ΠΎΠ±Ρ‹ ΠΏΠΎΠ»ΡƒΡ‡ΠΈΡ‚ΡŒ напряТСниС Π½Π° ΠΊΠ»Π΅ΠΌΠΌΠ°Ρ…: \ [V_ { ΠΊΠ»Π΅ΠΌΠΌΠ°} = \ epsilon – Ir = 12.00 \, V – (1.188 \, A) (0.100 \, \ Omega) = 11.90 \, V. \] НапряТСниС Π½Π° ΠΊΠ»Π΅ΠΌΠΌΠ°Ρ… здСсь лишь Π½Π΅ΠΌΠ½ΠΎΠ³ΠΎ Π½ΠΈΠΆΠ΅, Ρ‡Π΅ΠΌ Π­Π”Π‘, Ρ‡Ρ‚ΠΎ ΠΎΠ·Π½Π°Ρ‡Π°Π΅Ρ‚, Ρ‡Ρ‚ΠΎ Ρ‚ΠΎΠΊ втягиваСтся этой Π»Π΅Π³ΠΊΠΎΠΉ Π½Π°Π³Ρ€ΡƒΠ·ΠΊΠΎΠΉ Π½Π΅Π·Π½Π°Ρ‡ΠΈΡ‚Π΅Π»ΡŒΠ½ΠΎ.
  2. Аналогично, с \ (R_ {load} = 0.500 \, \ Omega \), Ρ‚ΠΎΠΊ Ρ€Π°Π²Π΅Π½ \ [I = \ frac {\ epsilon} {R + r} = \ frac {12.00 \, V} {0.2} {R} \) ΠΈΠ»ΠΈ \ (IV \), Π³Π΄Π΅ Π’, – напряТСниС Π½Π° ΠΊΠ»Π΅ΠΌΠΌΠ°Ρ… (Π² Π΄Π°Π½Π½ΠΎΠΌ случаС 10,0 Π’).
  3. Π—Π΄Π΅ΡΡŒ Π²Π½ΡƒΡ‚Ρ€Π΅Π½Π½Π΅Π΅ сопротивлСниС ΡƒΠ²Π΅Π»ΠΈΡ‡ΠΈΠ»ΠΎΡΡŒ, Π²ΠΎΠ·ΠΌΠΎΠΆΠ½ΠΎ, ΠΈΠ·-Π·Π° разряда Π±Π°Ρ‚Π°Ρ€Π΅ΠΈ, Π΄ΠΎ Ρ‚ΠΎΡ‡ΠΊΠΈ, Π² ΠΊΠΎΡ‚ΠΎΡ€ΠΎΠΉ ΠΎΠ½ΠΎ Ρ€Π°Π²Π½ΠΎ ΡΠΎΠΏΡ€ΠΎΡ‚ΠΈΠ²Π»Π΅Π½ΠΈΡŽ Π½Π°Π³Ρ€ΡƒΠ·ΠΊΠΈ. Как ΠΈ Ρ€Π°Π½ΡŒΡˆΠ΅, ΠΌΡ‹ сначала Π½Π°Ρ…ΠΎΠ΄ΠΈΠΌ Ρ‚ΠΎΠΊ, вводя извСстныС значСния Π² Π²Ρ‹Ρ€Π°ΠΆΠ΅Π½ΠΈΠ΅, получая \ [I = \ frac {\ epsilon} {R + r} = \ frac {12.00 \, V} {1.00 \, \ Omega} = 12.00 \, A. \] Π’Π΅ΠΏΠ΅Ρ€ΡŒ напряТСниС Π½Π° ΠΊΠ»Π΅ΠΌΠΌΠ°Ρ… Ρ€Π°Π²Π½ΠΎ \ [V_ {terminal} = \ epsilon – Ir = 12.00 \, V – (12.2 (0.500 \, \ Omega) = 72.00 \, W. \] ΠœΡ‹ Π²ΠΈΠ΄ΠΈΠΌ, Ρ‡Ρ‚ΠΎ ΡƒΠ²Π΅Π»ΠΈΡ‡Π΅Π½Π½ΠΎΠ΅ Π²Π½ΡƒΡ‚Ρ€Π΅Π½Π½Π΅Π΅ сопротивлСниС Π·Π½Π°Ρ‡ΠΈΡ‚Π΅Π»ΡŒΠ½ΠΎ снизило напряТСниС Π½Π° ΠΊΠ»Π΅ΠΌΠΌΠ°Ρ…, Ρ‚ΠΎΠΊ ΠΈ ΠΌΠΎΡ‰Π½ΠΎΡΡ‚ΡŒ, ΠΏΠΎΠ΄Π°Π²Π°Π΅ΠΌΡƒΡŽ Π½Π° Π½Π°Π³Ρ€ΡƒΠ·ΠΊΡƒ.

Π—Π½Π°Ρ‡Π΅Π½ΠΈΠ΅

Π’Π½ΡƒΡ‚Ρ€Π΅Π½Π½Π΅Π΅ сопротивлСниС Π±Π°Ρ‚Π°Ρ€Π΅ΠΈ ΠΌΠΎΠΆΠ΅Ρ‚ ΡƒΠ²Π΅Π»ΠΈΡ‡ΠΈΠ²Π°Ρ‚ΡŒΡΡ ΠΏΠΎ ΠΌΠ½ΠΎΠ³ΠΈΠΌ ΠΏΡ€ΠΈΡ‡ΠΈΠ½Π°ΠΌ. НапримСр, Π²Π½ΡƒΡ‚Ρ€Π΅Π½Π½Π΅Π΅ сопротивлСниС пСрСзаряТаСмой Π±Π°Ρ‚Π°Ρ€Π΅ΠΈ увСличиваСтся с ΡƒΠ²Π΅Π»ΠΈΡ‡Π΅Π½ΠΈΠ΅ΠΌ количСства Ρ€Π°Π·, ΠΊΠΎΠ³Π΄Π° батарСя пСрСзаряТаСтся. ΠŸΠΎΠ²Ρ‹ΡˆΠ΅Π½Π½ΠΎΠ΅ Π²Π½ΡƒΡ‚Ρ€Π΅Π½Π½Π΅Π΅ сопротивлСниС ΠΌΠΎΠΆΠ΅Ρ‚ ΠΈΠΌΠ΅Ρ‚ΡŒ двоякоС влияниС Π½Π° аккумулятор.Π‘Π½Π°Ρ‡Π°Π»Π° снизится напряТСниС Π½Π° ΠΊΠ»Π΅ΠΌΠΌΠ°Ρ…. Π’ΠΎ-Π²Ρ‚ΠΎΡ€Ρ‹Ρ…, аккумулятор ΠΌΠΎΠΆΠ΅Ρ‚ ΠΏΠ΅Ρ€Π΅Π³Ρ€Π΅Ρ‚ΡŒΡΡ ΠΈΠ·-Π·Π° ΠΏΠΎΠ²Ρ‹ΡˆΠ΅Π½Π½ΠΎΠΉ мощности, рассСиваСмой Π²Π½ΡƒΡ‚Ρ€Π΅Π½Π½ΠΈΠΌ сопротивлСниСм.

Π£ΠΏΡ€Π°ΠΆΠ½Π΅Π½ΠΈΠ΅ \ (\ PageIndex {1} \)

Если Π²Ρ‹ помСститС ΠΏΡ€ΠΎΠ²ΠΎΠ΄ нСпосрСдствСнно ΠΌΠ΅ΠΆΠ΄Ρƒ двумя Π²Ρ‹Π²ΠΎΠ΄Π°ΠΌΠΈ Π±Π°Ρ‚Π°Ρ€Π΅ΠΈ, эффСктивно Π·Π°ΠΊΠΎΡ€ΠΎΡ‚ΠΈΠ² ΠΊΠ»Π΅ΠΌΠΌΡ‹, батарСя Π½Π°Ρ‡Π½Π΅Ρ‚ Π½Π°Π³Ρ€Π΅Π²Π°Ρ‚ΡŒΡΡ. Как Π²Ρ‹ Π΄ΡƒΠΌΠ°Π΅Ρ‚Π΅, ΠΏΠΎΡ‡Π΅ΠΌΡƒ это происходит?

РСшСниС

Если ΠΊ ΠΊΠ»Π΅ΠΌΠΌΠ°ΠΌ подсоСдинСн ΠΏΡ€ΠΎΠ²ΠΎΠ΄, сопротивлСниС Π½Π°Π³Ρ€ΡƒΠ·ΠΊΠΈ Π±Π»ΠΈΠ·ΠΊΠΎ ΠΊ Π½ΡƒΠ»ΡŽ ΠΈΠ»ΠΈ, ΠΏΠΎ ΠΊΡ€Π°ΠΉΠ½Π΅ΠΉ ΠΌΠ΅Ρ€Π΅, Π·Π½Π°Ρ‡ΠΈΡ‚Π΅Π»ΡŒΠ½ΠΎ мСньшС Π²Π½ΡƒΡ‚Ρ€Π΅Π½Π½Π΅Π³ΠΎ сопротивлСния Π±Π°Ρ‚Π°Ρ€Π΅ΠΈ.2Ρ€) \). ΠœΠΎΡ‰Π½ΠΎΡΡ‚ΡŒ рассСиваСтся Π² Π²ΠΈΠ΄Π΅ Ρ‚Π΅ΠΏΠ»Π°.

ВСстСры Π±Π°Ρ‚Π°Ρ€Π΅ΠΉ

ВСстСры Π±Π°Ρ‚Π°Ρ€Π΅ΠΉ, Ρ‚Π°ΠΊΠΈΠ΅ ΠΊΠ°ΠΊ Ρ‚Π΅, Ρ‡Ρ‚ΠΎ ΠΏΠΎΠΊΠ°Π·Π°Π½Ρ‹ Π½Π° рисункС \ (\ PageIndex {8} \), ΠΈΡΠΏΠΎΠ»ΡŒΠ·ΡƒΡŽΡ‚ ΠΌΠ°Π»Ρ‹Π΅ Π½Π°Π³Ρ€ΡƒΠ·ΠΎΡ‡Π½Ρ‹Π΅ рСзисторы, Ρ‡Ρ‚ΠΎΠ±Ρ‹ Π½Π°ΠΌΠ΅Ρ€Π΅Π½Π½ΠΎ ΠΏΠΎΡ‚Ρ€Π΅Π±Π»ΡΡ‚ΡŒ Ρ‚ΠΎΠΊ, Ρ‡Ρ‚ΠΎΠ±Ρ‹ ΠΎΠΏΡ€Π΅Π΄Π΅Π»ΠΈΡ‚ΡŒ, ΠΏΠ°Π΄Π°Π΅Ρ‚ Π»ΠΈ ΠΏΠΎΡ‚Π΅Π½Ρ†ΠΈΠ°Π» ΠΊΠ»Π΅ΠΌΠΌ Π½ΠΈΠΆΠ΅ допустимого уровня. Π₯отя ΠΈΠ·ΠΌΠ΅Ρ€ΠΈΡ‚ΡŒ Π²Π½ΡƒΡ‚Ρ€Π΅Π½Π½Π΅Π΅ сопротивлСниС Π±Π°Ρ‚Π°Ρ€Π΅ΠΈ слоТно, тСстСры Π±Π°Ρ‚Π°Ρ€Π΅ΠΈ ΠΌΠΎΠ³ΡƒΡ‚ ΠΎΠ±Π΅ΡΠΏΠ΅Ρ‡ΠΈΡ‚ΡŒ ΠΈΠ·ΠΌΠ΅Ρ€Π΅Π½ΠΈΠ΅ Π²Π½ΡƒΡ‚Ρ€Π΅Π½Π½Π΅Π³ΠΎ сопротивлСния Π±Π°Ρ‚Π°Ρ€Π΅ΠΈ. Если Π²Π½ΡƒΡ‚Ρ€Π΅Π½Π½Π΅Π΅ сопротивлСниС высокоС, батарСя разряТСна, ΠΎ Ρ‡Π΅ΠΌ ΡΠ²ΠΈΠ΄Π΅Ρ‚Π΅Π»ΡŒΡΡ‚Π²ΡƒΠ΅Ρ‚ Π½ΠΈΠ·ΠΊΠΎΠ΅ напряТСниС Π½Π° ΠΊΠ»Π΅ΠΌΠΌΠ°Ρ….

Рисунок \ (\ PageIndex {8} \): ВСстСры Π±Π°Ρ‚Π°Ρ€Π΅ΠΉ ΠΈΠ·ΠΌΠ΅Ρ€ΡΡŽΡ‚ напряТСниС Π½Π° ΠΊΠ»Π΅ΠΌΠΌΠ°Ρ… ΠΏΠΎΠ΄ Π½Π°Π³Ρ€ΡƒΠ·ΠΊΠΎΠΉ, Ρ‡Ρ‚ΠΎΠ±Ρ‹ ΠΎΠΏΡ€Π΅Π΄Π΅Π»ΠΈΡ‚ΡŒ состояниС Π±Π°Ρ‚Π°Ρ€Π΅ΠΈ. (a) Π’Π΅Ρ…Π½ΠΈΠΊ-элСктронщик Π’ΠœΠ‘ БША ΠΈΡΠΏΠΎΠ»ΡŒΠ·ΡƒΠ΅Ρ‚ тСстСр аккумуляторов для ΠΏΡ€ΠΎΠ²Π΅Ρ€ΠΊΠΈ Π±ΠΎΠ»ΡŒΡˆΠΈΡ… аккумуляторов Π½Π° Π±ΠΎΡ€Ρ‚Ρƒ авианосца USS Nimitz . ВСстСр Π±Π°Ρ‚Π°Ρ€Π΅ΠΉ, ΠΊΠΎΡ‚ΠΎΡ€Ρ‹ΠΉ ΠΎΠ½Π° ΠΈΡΠΏΠΎΠ»ΡŒΠ·ΡƒΠ΅Ρ‚, ΠΈΠΌΠ΅Π΅Ρ‚ нСбольшоС сопротивлСниС, ΠΊΠΎΡ‚ΠΎΡ€ΠΎΠ΅ ΠΌΠΎΠΆΠ΅Ρ‚ Ρ€Π°ΡΡΠ΅ΠΈΠ²Π°Ρ‚ΡŒ большоС количСство энСргии. (b) ПоказанноС нСбольшоС устройство ΠΈΡΠΏΠΎΠ»ΡŒΠ·ΡƒΠ΅Ρ‚ΡΡ Π½Π° Π½Π΅Π±ΠΎΠ»ΡŒΡˆΠΈΡ… батарСях ΠΈ ΠΈΠΌΠ΅Π΅Ρ‚ Ρ†ΠΈΡ„Ρ€ΠΎΠ²ΠΎΠΉ дисплСй для ΠΈΠ½Π΄ΠΈΠΊΠ°Ρ†ΠΈΠΈ допустимого напряТСния Π½Π° ΠΊΠ»Π΅ΠΌΠΌΠ°Ρ…. (ΠΊΡ€Π΅Π΄ΠΈΡ‚ А: модификация Ρ€Π°Π±ΠΎΡ‚Ρ‹ ДТСйсона А.ДТонстон; ΠΊΡ€Π΅Π΄ΠΈΡ‚ b: модификация Ρ€Π°Π±ΠΎΡ‚Ρ‹ ΠšΠ΅ΠΉΡ‚Π° Уильямсона)

НСкоторыС Π±Π°Ρ‚Π°Ρ€Π΅ΠΈ ΠΌΠΎΠΆΠ½ΠΎ ΠΏΠ΅Ρ€Π΅Π·Π°Ρ€ΡΠ΄ΠΈΡ‚ΡŒ, пропустив Ρ‡Π΅Ρ€Π΅Π· Π½ΠΈΡ… Ρ‚ΠΎΠΊ Π² Π½Π°ΠΏΡ€Π°Π²Π»Π΅Π½ΠΈΠΈ, ΠΏΡ€ΠΎΡ‚ΠΈΠ²ΠΎΠΏΠΎΠ»ΠΎΠΆΠ½ΠΎΠΌ Ρ‚ΠΎΠΊΡƒ, ΠΊΠΎΡ‚ΠΎΡ€Ρ‹ΠΉ ΠΎΠ½ΠΈ ΠΏΠΎΠ΄Π°ΡŽΡ‚ Π² ΠΏΡ€ΠΈΠ±ΠΎΡ€. Π­Ρ‚ΠΎ ΠΎΠ±Ρ‹Ρ‡Π½ΠΎ дСлаСтся Π² автомобилях ΠΈ батарСях для Π½Π΅Π±ΠΎΠ»ΡŒΡˆΠΈΡ… элСктроприборов ΠΈ элСктронных устройств (Рисунок \ (\ PageIndex {9} \)). Π’Ρ‹Ρ…ΠΎΠ΄Π½ΠΎΠ΅ напряТСниС зарядного устройства Π΄ΠΎΠ»ΠΆΠ½ΠΎ Π±Ρ‹Ρ‚ΡŒ большС, Ρ‡Π΅ΠΌ Π­Π”Π‘ аккумулятора, Ρ‡Ρ‚ΠΎΠ±Ρ‹ Ρ‚ΠΎΠΊ Ρ‡Π΅Ρ€Π΅Π· Π½Π΅Π³ΠΎ рСвСрсировал. Π­Ρ‚ΠΎ ΠΏΡ€ΠΈΠ²ΠΎΠ΄ΠΈΡ‚ ΠΊ Ρ‚ΠΎΠΌΡƒ, Ρ‡Ρ‚ΠΎ напряТСниС Π½Π° ΠΊΠ»Π΅ΠΌΠΌΠ°Ρ… аккумулятора ΠΏΡ€Π΅Π²Ρ‹ΡˆΠ°Π΅Ρ‚ Π­Π”Π‘, ΠΏΠΎΡΠΊΠΎΠ»ΡŒΠΊΡƒ \ (V = \ epsilon – Ir \) ΠΈ I Ρ‚Π΅ΠΏΠ΅Ρ€ΡŒ ΠΎΡ‚Ρ€ΠΈΡ†Π°Ρ‚Π΅Π»ΡŒΠ½Ρ‹.

Рисунок \ (\ PageIndex {9} \): Π°Π²Ρ‚ΠΎΠΌΠΎΠ±ΠΈΠ»ΡŒΠ½ΠΎΠ΅ зарядноС устройство мСняСт Π½ΠΎΡ€ΠΌΠ°Π»ΡŒΠ½ΠΎΠ΅ Π½Π°ΠΏΡ€Π°Π²Π»Π΅Π½ΠΈΠ΅ Ρ‚ΠΎΠΊΠ° Ρ‡Π΅Ρ€Π΅Π· аккумулятор, обращая Π²ΡΠΏΡΡ‚ΡŒ Π΅Π³ΠΎ Ρ…ΠΈΠΌΠΈΡ‡Π΅ΡΠΊΡƒΡŽ Ρ€Π΅Π°ΠΊΡ†ΠΈΡŽ ΠΈ пополняя Π΅Π΅ химичСский ΠΏΠΎΡ‚Π΅Π½Ρ†ΠΈΠ°Π».

Π’Π°ΠΆΠ½ΠΎ ΠΏΠΎΠ½ΠΈΠΌΠ°Ρ‚ΡŒ послСдствия Π²Π½ΡƒΡ‚Ρ€Π΅Π½Π½Π΅Π³ΠΎ сопротивлСния источников Π­Π”Π‘, Ρ‚Π°ΠΊΠΈΡ… ΠΊΠ°ΠΊ Π±Π°Ρ‚Π°Ρ€Π΅ΠΈ ΠΈ солнСчныС элСмСнты, Π½ΠΎ часто Π°Π½Π°Π»ΠΈΠ· Ρ†Π΅ΠΏΠ΅ΠΉ выполняСтся с ΠΏΠΎΠΌΠΎΡ‰ΡŒΡŽ напряТСния Π½Π° ΠΊΠ»Π΅ΠΌΠΌΠ°Ρ… Π±Π°Ρ‚Π°Ρ€Π΅ΠΈ, ΠΊΠ°ΠΊ ΠΌΡ‹ Π΄Π΅Π»Π°Π»ΠΈ Π² ΠΏΡ€Π΅Π΄Ρ‹Π΄ΡƒΡ‰ΠΈΡ… Ρ€Π°Π·Π΄Π΅Π»Π°Ρ…. НапряТСниС Π½Π° ΠΊΠ»Π΅ΠΌΠΌΠ°Ρ… обозначаСтся просто ΠΊΠ°ΠΊ Π’ , Π±Π΅Π· индСкса Β«ΠΊΠ»Π΅ΠΌΠΌΠ°Β».Π­Ρ‚ΠΎ связано с Ρ‚Π΅ΠΌ, Ρ‡Ρ‚ΠΎ Π²Π½ΡƒΡ‚Ρ€Π΅Π½Π½Π΅Π΅ сопротивлСниС Π±Π°Ρ‚Π°Ρ€Π΅ΠΈ Ρ‚Ρ€ΡƒΠ΄Π½ΠΎ ΠΈΠ·ΠΌΠ΅Ρ€ΠΈΡ‚ΡŒ Π½Π°ΠΏΡ€ΡΠΌΡƒΡŽ, ΠΈ ΠΎΠ½ΠΎ ΠΌΠΎΠΆΠ΅Ρ‚ со Π²Ρ€Π΅ΠΌΠ΅Π½Π΅ΠΌ ΠΈΠ·ΠΌΠ΅Π½ΠΈΡ‚ΡŒΡΡ.

Авторы ΠΈ авторство

Бэмюэл Π”ΠΆ. Π›ΠΈΠ½Π³ (ГосударствСнный унивСрситСт Врумэна), Π”ΠΆΠ΅Ρ„Ρ„ Π‘Π°Π½Π½ΠΈ (УнивСрситСт Π›ΠΎΠΉΠΎΠ»Π° ΠœΡΡ€ΠΈΠΌΠ°ΡƒΠ½Ρ‚) ΠΈ Π‘ΠΈΠ»Π» Мобс со ΠΌΠ½ΠΎΠ³ΠΈΠΌΠΈ Π°Π²Ρ‚ΠΎΡ€Π°ΠΌΠΈ. Π­Ρ‚Π° Ρ€Π°Π±ΠΎΡ‚Π° Π»ΠΈΡ†Π΅Π½Π·ΠΈΡ€ΠΎΠ²Π°Π½Π° OpenStax University Physics Π² соотвСтствии с Π»ΠΈΡ†Π΅Π½Π·ΠΈΠ΅ΠΉ Creative Commons Attribution License (4.0).

Π­Π”Π‘ ΠΈ Π²Π½ΡƒΡ‚Ρ€Π΅Π½Π½Π΅Π΅ сопротивлСниС

Π­Π”Π‘ ΠΈ Π²Π½ΡƒΡ‚Ρ€Π΅Π½Π½Π΅Π΅ сопротивлСниС
ΡΠ»Π΅Π΄ΡƒΡŽΡ‰ΠΈΠΉ: рСзисторы ΠΏΠΎΡΠ»Π΅Π΄ΠΎΠ²Π°Ρ‚Π΅Π»ΡŒΠ½ΠΎ ΠΈ Π²Π²Π΅Ρ€Ρ…: элСктричСский Ρ‚ΠΎΠΊ ΠŸΡ€Π΅Π΄Ρ‹Π΄ΡƒΡ‰ΠΈΠΉ: Π‘ΠΎΠΏΡ€ΠΎΡ‚ΠΈΠ²Π»Π΅Π½ΠΈΠ΅ ΠΈ ΡƒΠ΄Π΅Π»ΡŒΠ½ΠΎΠ΅ сопротивлСниС Π’Π΅ΠΏΠ΅Ρ€ΡŒ настоящиС Π±Π°Ρ‚Π°Ρ€Π΅ΠΈ ΠΈΠ·Π³ΠΎΡ‚Π°Π²Π»ΠΈΠ²Π°ΡŽΡ‚ΡΡ ΠΈΠ· ΠΌΠ°Ρ‚Π΅Ρ€ΠΈΠ°Π»ΠΎΠ² с Π½Π΅Π½ΡƒΠ»Π΅Π²Ρ‹ΠΌ ΡƒΠ΄Π΅Π»ΡŒΠ½Ρ‹ΠΌ сопротивлСниСм.ΠžΡ‚ΡΡŽΠ΄Π° слСдуСт, Ρ‡Ρ‚ΠΎ настоящиС Π±Π°Ρ‚Π°Ρ€Π΅ΠΈ – это Π½Π΅ просто источники чистого напряТСния. Они Ρ‚Π°ΠΊΠΆΠ΅ ΠΎΠ±Π»Π°Π΄Π°ΡŽΡ‚ Π²Π½ΡƒΡ‚Ρ€Π΅Π½Π½ΠΈΡ… сопротивлСний . ΠœΠ΅ΠΆΠ΄Ρƒ ΠΏΡ€ΠΎΡ‡ΠΈΠΌ, чистоС напряТСниС Π˜ΡΡ‚ΠΎΡ‡Π½ΠΈΠΊ ΠΎΠ±Ρ‹Ρ‡Π½ΠΎ обозначаСтся ΠΊΠ°ΠΊ Π­Π”Π‘ (Ρ‡Ρ‚ΠΎ ΠΎΠ·Π½Π°Ρ‡Π°Π΅Ρ‚ элСктродвиТущая сила ). ΠšΠΎΠ½Π΅Ρ‡Π½ΠΎ, Π­Π”Π‘ измСряСтся Π² Π²ΠΎΠ»ΡŒΡ‚Π°Ρ…. Аккумулятор ΠΌΠΎΠΆΠ½ΠΎ ΡΠΌΠΎΠ΄Π΅Π»ΠΈΡ€ΠΎΠ²Π°Ρ‚ΡŒ ΠΊΠ°ΠΊ Π­Π”Π‘, Π²ΠΊΠ»ΡŽΡ‡Π΅Π½Π½ΡƒΡŽ ΠΏΠΎΡΠ»Π΅Π΄ΠΎΠ²Π°Ρ‚Π΅Π»ΡŒΠ½ΠΎ с рСзистором. , ΠΊΠΎΡ‚ΠΎΡ€Ρ‹ΠΉ прСдставляСт собой Π΅Π³ΠΎ Π²Π½ΡƒΡ‚Ρ€Π΅Π½Π½Π΅Π΅ сопротивлСниС. ΠŸΡ€Π΅Π΄ΠΏΠΎΠ»ΠΎΠΆΠΈΠΌ, Ρ‡Ρ‚ΠΎ Ρ‚Π°ΠΊΠΈΠ΅ батарСя ΠΈΡΠΏΠΎΠ»ΡŒΠ·ΡƒΠ΅Ρ‚ΡΡ для управлСния Ρ‚ΠΎΠΊΠΎΠΌ Ρ‡Π΅Ρ€Π΅Π· внСшний Π½Π°Π³Ρ€ΡƒΠ·ΠΎΡ‡Π½Ρ‹ΠΉ рСзистор, Ρ‚Π°ΠΊ ΠΊΠ°ΠΊ ΠΈΠ·ΠΎΠ±Ρ€Π°ΠΆΠ΅Π½Π½Ρ‹ΠΉ Π½Π° рис.17. ΠžΠ±Ρ€Π°Ρ‚ΠΈΡ‚Π΅ Π²Π½ΠΈΠΌΠ°Π½ΠΈΠ΅, Ρ‡Ρ‚ΠΎ Π½Π° ΠΏΡ€ΠΈΠ½Ρ†ΠΈΠΏΠΈΠ°Π»ΡŒΠ½Ρ‹Ρ… схСмах Π­Π”Π‘ прСдставлСна ​​в Π²ΠΈΠ΄Π΅ Π΄Π²ΡƒΡ… Π±Π»ΠΈΠ·ΠΊΠΎ располоТСнных ΠΏΠ°Ρ€Π°Π»Π»Π΅Π»ΡŒΠ½Ρ‹Ρ… Π»ΠΈΠ½ΠΈΠΈ Π½Π΅Ρ€Π°Π²Π½ΠΎΠΉ Π΄Π»ΠΈΠ½Ρ‹. ЭлСктричСский ΠΏΠΎΡ‚Π΅Π½Ρ†ΠΈΠ°Π» Π±ΠΎΠ»Π΅Π΅ Π΄Π»ΠΈΠ½Π½ΠΎΠΉ Π»ΠΈΠ½ΠΈΠΈ большС, Ρ‡Π΅ΠΌ Ρ‚ΠΎΡ‚ ΠΈΠ· Π±ΠΎΠ»Π΅Π΅ ΠΊΠΎΡ€ΠΎΡ‚ΠΊΠΈΡ… ΠΏΠΎ Π²ΠΎΠ»ΡŒΡ‚Π°ΠΌ. РСзистор прСдставлСн ΠΊΠ°ΠΊ зигзагообразная линия.
Рисунок 17: БатарСя Π­Π”Π‘ ΠΈ Π²Π½ΡƒΡ‚Ρ€Π΅Π½Π½Π΅Π³ΠΎ сопротивлСния ΠΏΠΎΠ΄ΠΊΠ»ΡŽΡ‡Π΅Π½Π° ΠΊ Π½Π°Π³Ρ€ΡƒΠ·ΠΎΡ‡Π½ΠΎΠΌΡƒ рСзистору сопротивлСния.

Рассмотрим аккумулятор Π½Π° рисункС.НапряТСниС аккумулятора Ρ€Π°Π²Π½ΠΎ опрСдСляСтся ΠΊΠ°ΠΊ Ρ€Π°Π·Π½ΠΈΡ†Π° Π² элСктричСском ΠΏΠΎΡ‚Π΅Π½Ρ†ΠΈΠ°Π»Π΅ ΠΌΠ΅ΠΆΠ΄Ρƒ Π΅Π³ΠΎ ΠΏΠΎΠ»ΠΎΠΆΠΈΡ‚Π΅Π»ΡŒΠ½Ρ‹ΠΌ ΠΈ ΠΎΡ‚Ρ€ΠΈΡ†Π°Ρ‚Π΅Π»ΡŒΠ½Ρ‹Π΅ ΠΊΠ»Π΅ΠΌΠΌΡ‹: Ρ‚.Π΅., Ρ‚ΠΎΡ‡ΠΊΠΈ ΠΈ соотвСтствСнно. Когда ΠΌΡ‹ ΠΏΠ΅Ρ€Π΅Ρ…ΠΎΠ΄ΠΈΠΌ ΠΎΡ‚ ΠΊ , элСктричСский ΠΏΠΎΡ‚Π΅Π½Ρ†ΠΈΠ°Π» увСличиваСтся Π½Π° Π²ΠΎΠ»ΡŒΡ‚, ΠΊΠΎΠ³Π΄Π° ΠΌΡ‹ пСрСсСкаСм Π­Π”Π‘, Π½ΠΎ Π·Π°Ρ‚Π΅ΠΌ ΡƒΠΌΠ΅Π½ΡŒΡˆΠ°Π΅Ρ‚ΡΡ Π½Π° Π²ΠΎΠ»ΡŒΡ‚, ΠΊΠΎΠ³Π΄Π° ΠΌΡ‹ пСрСсСкаСм Π²Π½ΡƒΡ‚Ρ€Π΅Π½Π½ΠΈΠΉ рСзистор. ПадСниС напряТСния Π½Π° рСзисторС слСдуСт ΠΈΠ· Π·Π°ΠΊΠΎΠ½Π° Ома, ΠΈΠ· ΠΊΠΎΡ‚ΠΎΡ€ΠΎΠ³ΠΎ слСдуСт, Ρ‡Ρ‚ΠΎ ΠΏΠ°Π΄Π΅Π½ΠΈΠ΅ напряТСния Π½Π° рСзисторС, нСсущСм Ρ‚ΠΎΠΊ , находится Π² Ρ‚ΠΎΠΌ Π½Π°ΠΏΡ€Π°Π²Π»Π΅Π½ΠΈΠΈ, Π² ΠΊΠΎΡ‚ΠΎΡ€ΠΎΠΌ Ρ‚Π΅ΠΊΡƒΡ‰ΠΈΠ΅ ΠΏΠΎΡ‚ΠΎΠΊΠΈ.Π’Π°ΠΊΠΈΠΌ ΠΎΠ±Ρ€Π°Π·ΠΎΠΌ, напряТСниС аккумулятора связано с Π΅Π³ΠΎ Π­Π”Π‘. ΠΈ Π²Π½ΡƒΡ‚Ρ€Π΅Π½Π½Π΅Π΅ сопротивлСниС Ρ‡Π΅Ρ€Π΅Π·

(133)

ΠžΠ±Ρ‹Ρ‡Π½ΠΎ ΠΌΡ‹ Π΄ΡƒΠΌΠ°Π΅ΠΌ, Ρ‡Ρ‚ΠΎ Π­Π”Π‘ Π±Π°Ρ‚Π°Ρ€Π΅ΠΈ ΠΏΠΎ сущСству постоянная (ΠΏΠΎΡΠΊΠΎΠ»ΡŒΠΊΡƒ ΠΎΠ½Π° зависит Ρ‚ΠΎΠ»ΡŒΠΊΠΎ ΠΎΡ‚ химичСской Ρ€Π΅Π°ΠΊΡ†ΠΈΠΈ, происходящСй Π²Π½ΡƒΡ‚Ρ€ΠΈ Π±Π°Ρ‚Π°Ρ€Π΅ΠΈ, которая ΠΏΡ€Π΅ΠΎΠ±Ρ€Π°Π·ΡƒΠ΅Ρ‚ химичСская энСргия Π² ΡΠ»Π΅ΠΊΡ‚Ρ€ΠΈΡ‡Π΅ΡΠΊΡƒΡŽ), поэтому ΠΌΡ‹ Π΄ΠΎΠ»ΠΆΠ½Ρ‹ Π·Π°ΠΊΠ»ΡŽΡ‡ΠΈΡ‚ΡŒ, Ρ‡Ρ‚ΠΎ напряТСниС батарСя Π½Π° самом Π΄Π΅Π»Π΅ ΡƒΠΌΠ΅Π½ΡŒΡˆΠ°Π΅Ρ‚ΡΡ ΠΏΠΎ ΠΌΠ΅Ρ€Π΅ увСличСния Ρ‚ΠΎΠΊΠ°, потрСбляСмого ΠΎΡ‚ Π½Π΅Π΅.ЀактичСски, напряТСниС Ρ€Π°Π²Π½ΠΎ Ρ‚ΠΎΠ»ΡŒΠΊΠΎ Π­Π”Π‘ ΠΏΡ€ΠΈ ΠΏΡ€Π΅Π½Π΅Π±Ρ€Π΅ΠΆΠΈΠΌΠΎ ΠΌΠ°Π»ΠΎΠΌ Ρ‚ΠΎΠΊΠ΅. Π’Π΅ΠΊΡƒΡ‰ΠΈΠΉ Ρ€ΠΎΠ·Ρ‹Π³Ρ€Ρ‹Ρˆ ΠΎΡ‚ аккумулятора ΠΎΠ±Ρ‹Ρ‡Π½ΠΎ Π½Π΅ ΠΌΠΎΠΆΠ΅Ρ‚ ΠΏΡ€Π΅Π²Ρ‹ΡˆΠ°Ρ‚ΡŒ критичСскоС Π·Π½Π°Ρ‡Π΅Π½ΠΈΠ΅
(134)

ΠΏΠΎΡΠΊΠΎΠ»ΡŒΠΊΡƒ напряТСниС становится ΠΎΡ‚Ρ€ΠΈΡ†Π°Ρ‚Π΅Π»ΡŒΠ½Ρ‹ΠΌ (Ρ‡Ρ‚ΠΎ ΠΌΠΎΠΆΠ΅Ρ‚ ΠΏΡ€ΠΎΠΈΠ·ΠΎΠΉΡ‚ΠΈ Ρ‚ΠΎΠ»ΡŒΠΊΠΎ Ссли рСзистор Π½Π°Π³Ρ€ΡƒΠ·ΠΊΠΈ Ρ‚Π°ΠΊΠΆΠ΅ ΠΎΡ‚Ρ€ΠΈΡ†Π°Ρ‚Π΅Π»ΡŒΠ½Ρ‹ΠΉ: это практичСски Π½Π΅Π²ΠΎΠ·ΠΌΠΎΠΆΠ½ΠΎ). ΠžΡ‚ΡΡŽΠ΄Π° слСдуСт, Ρ‡Ρ‚ΠΎ Ссли ΠΌΡ‹ Π·Π°ΠΊΠΎΡ€ΠΎΡ‚ΠΈΠΌ аккумулятор, ΠΏΠΎΠ΄ΠΊΠ»ΡŽΡ‡ΠΈΠ² Π΅Π³ΠΎ ΠΏΠΎΠ»ΠΎΠΆΠΈΡ‚Π΅Π»ΡŒΠ½Ρ‹Π΅ ΠΈ ΠΎΡ‚Ρ€ΠΈΡ†Π°Ρ‚Π΅Π»ΡŒΠ½Ρ‹Π΅ ΠΊΠ»Π΅ΠΌΠΌΡ‹ вмСстС с использованиСм проводящСго ΠΏΡ€ΠΎΠ²ΠΎΠ΄Π° с Π½Π΅Π·Π½Π°Ρ‡ΠΈΡ‚Π΅Π»ΡŒΠ½Ρ‹ΠΌ сопротивлСниСм, Ρ‚ΠΎΠΊ, потрСбляСмый Π±Π°Ρ‚Π°Ρ€Π΅Π΅ΠΉ, ΠΎΠ³Ρ€Π°Π½ΠΈΡ‡Π΅Π½ Π΅Π΅ Π²Π½ΡƒΡ‚Ρ€Π΅Π½Π½ΠΈΠΌ сопротивлСниСм.ЀактичСски Π² этом случаС сила Ρ‚ΠΎΠΊΠ° Ρ€Π°Π²Π½Π° максимально Π²ΠΎΠ·ΠΌΠΎΠΆΠ½ΠΎΠΉ. Π’Π΅ΠΊΡƒΡ‰ΠΈΠΉ .

Настоящая батарСя ΠΎΠ±Ρ‹Ρ‡Π½ΠΎ характСризуСтся Π΅Π³ΠΎ Π­Π”Π‘ (, Ρ‚.Π΅. , Π΅Π³ΠΎ напряТСниС ΠΏΡ€ΠΈ Π½ΡƒΠ»Π΅Π²ΠΎΠΌ Ρ‚ΠΎΠΊΠ΅) ΠΈ ΠΌΠ°ΠΊΡΠΈΠΌΠ°Π»ΡŒΠ½Ρ‹ΠΉ Ρ‚ΠΎΠΊ, ΠΊΠΎΡ‚ΠΎΡ€Ρ‹ΠΉ ΠΎΠ½ ΠΌΠΎΠΆΠ΅Ρ‚ ΠΏΠΎΠ΄Π°Π²Π°Ρ‚ΡŒ. НапримСр, стандартный сухой элСмСнт (, Ρ‚. Π•. , своСго Ρ€ΠΎΠ΄Π° аккумулятор, ΠΈΡΠΏΠΎΠ»ΡŒΠ·ΡƒΠ΅ΠΌΡ‹ΠΉ для питания ΠΊΠ°Π»ΡŒΠΊΡƒΠ»ΡΡ‚ΠΎΡ€ΠΎΠ² ΠΈ Ρ„ΠΎΠ½Π°Ρ€Π΅ΠΉ) ΠΎΠ±Ρ‹Ρ‡Π½ΠΎ рассчитан Π½Π° ΠΈ скаТи) . Π’Π°ΠΊΠΈΠΌ ΠΎΠ±Ρ€Π°Π·ΠΎΠΌ, Π½ΠΈΡ‡Π΅Π³ΠΎ Π΄Π΅ΠΉΡΡ‚Π²ΠΈΡ‚Π΅Π»ΡŒΠ½ΠΎ катастрофичСского Π½Π΅ ΠΏΡ€ΠΎΠΈΠ·ΠΎΠΉΠ΄Π΅Ρ‚. ΠΏΡ€ΠΎΠΈΠ·ΠΎΠΉΠ΄Π΅Ρ‚, Ссли ΠΌΡ‹ Π·Π°ΠΊΠΎΡ€ΠΎΡ‚ΠΈΠΌ сухой элСмСнт.ΠœΡ‹ разрядим Π±Π°Ρ‚Π°Ρ€Π΅ΡŽ Ρ‡Π΅Ρ€Π΅Π· ΡΡ€Π°Π²Π½ΠΈΡ‚Π΅Π»ΡŒΠ½ΠΎ ΠΊΠΎΡ€ΠΎΡ‚ΠΊΠΈΠΉ ΠΏΡ€ΠΎΠΌΠ΅ΠΆΡƒΡ‚ΠΎΠΊ Π²Ρ€Π΅ΠΌΠ΅Π½ΠΈ, Π½ΠΎ опасно большой Ρ‚ΠΎΠΊ Π½Π΅ Π±ΡƒΠ΄Π΅Ρ‚ ΠΏΠΎΡ‚ΠΎΠΊ. Π‘ Π΄Ρ€ΡƒΠ³ΠΎΠΉ стороны, Π°Π²Ρ‚ΠΎΠΌΠΎΠ±ΠΈΠ»ΡŒΠ½Ρ‹ΠΉ аккумулятор ΠΎΠ±Ρ‹Ρ‡Π½ΠΎ рассчитываСтся Π½Π° ΠΈ Ρ‡Ρ‚ΠΎ-Ρ‚ΠΎ Π²Ρ€ΠΎΠ΄Π΅ (Ρ‚Π°ΠΊΠΎΠΉ Ρ‚ΠΎΠΊ Π½ΡƒΠΆΠ΅Π½ для Π·Π°ΠΏΡƒΡΡ‚ΠΈΡ‚ΡŒ стартСр). ΠŸΠΎΠ½ΡΡ‚Π½ΠΎ, Ρ‡Ρ‚ΠΎ Π°Π²Ρ‚ΠΎΠΌΠΎΠ±ΠΈΠ»ΡŒΠ½Ρ‹ΠΉ аккумулятор Π΄ΠΎΠ»ΠΆΠ΅Π½ ΠΈΠΌΠ΅Ρ‚ΡŒ ΠΌΠ½ΠΎΠ³ΠΎ мСньшСС Π²Π½ΡƒΡ‚Ρ€Π΅Π½Π½Π΅Π΅ сопротивлСниС, Ρ‡Π΅ΠΌ Ρƒ сухого элСмСнта. ΠžΡ‚ΡΡŽΠ΄Π° слСдуСт, Ρ‡Ρ‚ΠΎ Ссли ΠΌΡ‹ Π±Ρ‹Π»ΠΈ достаточно Π³Π»ΡƒΠΏΡ‹, Ρ‡Ρ‚ΠΎΠ±Ρ‹ Π·Π°ΠΌΠΊΠ½ΡƒΡ‚ΡŒ Π°Π²Ρ‚ΠΎΠΌΠΎΠ±ΠΈΠ»ΡŒΠ½Ρ‹ΠΉ аккумулятор, Π² Ρ€Π΅Π·ΡƒΠ»ΡŒΡ‚Π°Ρ‚Π΅ довольно катастрофичСски (ΠΏΡ€Π΅Π΄ΡΡ‚Π°Π²ΡŒΡ‚Π΅ сСбС всю ΡΠ½Π΅Ρ€Π³ΠΈΡŽ, Π½Π΅ΠΎΠ±Ρ…ΠΎΠ΄ΠΈΠΌΡƒΡŽ для запуска двигатСля Π°Π²Ρ‚ΠΎΠΌΠΎΠ±ΠΈΠ»ΡŒ собираСтся Ρ‚ΠΎΠ½ΠΊΠΈΠΌ ΠΏΡ€ΠΎΠ²ΠΎΠ΄ΠΎΠΌ, ΡΠΎΠ΅Π΄ΠΈΠ½ΡΡŽΡ‰ΠΈΠΌ ΠΊΠ»Π΅ΠΌΠΌΡ‹ аккумулятора вмСстС).



ΡΠ»Π΅Π΄ΡƒΡŽΡ‰ΠΈΠΉ: рСзисторы ΠΏΠΎΡΠ»Π΅Π΄ΠΎΠ²Π°Ρ‚Π΅Π»ΡŒΠ½ΠΎ ΠΈ Π²Π²Π΅Ρ€Ρ…: элСктричСский Ρ‚ΠΎΠΊ ΠŸΡ€Π΅Π΄Ρ‹Π΄ΡƒΡ‰ΠΈΠΉ: Π‘ΠΎΠΏΡ€ΠΎΡ‚ΠΈΠ²Π»Π΅Π½ΠΈΠ΅ ΠΈ ΡƒΠ΄Π΅Π»ΡŒΠ½ΠΎΠ΅ сопротивлСниС
Π ΠΈΡ‡Π°Ρ€Π΄ Π€ΠΈΡ†ΠΏΠ°Ρ‚Ρ€ΠΈΠΊ 2007-07-14

Π Π°Π·Π½ΠΈΡ†Π° ΠΌΠ΅ΠΆΠ΄Ρƒ Π­Π”Π‘ ΠΈ напряТСниСм (со ΡΡ€Π°Π²Π½ΠΈΡ‚Π΅Π»ΡŒΠ½ΠΎΠΉ Ρ‚Π°Π±Π»ΠΈΡ†Π΅ΠΉ)

Одно ΠΈΠ· основных Ρ€Π°Π·Π»ΠΈΡ‡ΠΈΠΉ ΠΌΠ΅ΠΆΠ΄Ρƒ Π­Π”Π‘ ΠΈ напряТСниСм состоит Π² Ρ‚ΠΎΠΌ, Ρ‡Ρ‚ΠΎ Π­Π”Π‘ – это энСргия, подводимая ΠΊ заряду, Ρ‚ΠΎΠ³Π΄Π° ΠΊΠ°ΠΊ напряТСниС – это энСргия, нСобходимая для пСрСмСщСния Π΅Π΄ΠΈΠ½ΠΈΡ‡Π½ΠΎΠ³ΠΎ заряда ΠΈΠ· ΠΎΠ΄Π½ΠΎΠΉ Ρ‚ΠΎΡ‡ΠΊΠΈ Π² Π΄Ρ€ΡƒΠ³ΡƒΡŽ.Π”Ρ€ΡƒΠ³ΠΈΠ΅ различия ΠΌΠ΅ΠΆΠ΄Ρƒ Π½ΠΈΠΌΠΈ ΠΎΠ±ΡŠΡΡΠ½ΡΡŽΡ‚ΡΡ Π½ΠΈΠΆΠ΅ Π² ΡΡ€Π°Π²Π½ΠΈΡ‚Π΅Π»ΡŒΠ½ΠΎΠΉ Ρ‚Π°Π±Π»ΠΈΡ†Π΅.

Π‘ΠΎΠ΄Π΅Ρ€ΠΆΠ°Π½ΠΈΠ΅: Π­Π”Π‘ ΠΏΡ€ΠΎΡ‚ΠΈΠ² напряТСния

  1. Π‘Ρ€Π°Π²Π½ΠΈΡ‚Π΅Π»ΡŒΠ½Π°Ρ Ρ‚Π°Π±Π»ΠΈΡ†Π°
  2. ΠžΠΏΡ€Π΅Π΄Π΅Π»Π΅Π½ΠΈΠ΅
  3. ΠšΠ»ΡŽΡ‡Π΅Π²Ρ‹Π΅ отличия

Π‘Ρ€Π°Π²Π½ΠΈΡ‚Π΅Π»ΡŒΠ½Π°Ρ Ρ‚Π°Π±Π»ΠΈΡ†Π°

Основа для сравнСния Π­Π”Π‘ НапряТСниС
ΠžΠΏΡ€Π΅Π΄Π΅Π»Π΅Π½ΠΈΠ΅ ΠšΠΎΠ»ΠΈΡ‡Π΅ΡΡ‚Π²ΠΎ энСргии, ΠΏΠΎΠ΄Π°Π²Π°Π΅ΠΌΠΎΠΉ источником Π½Π° ΠΊΠ°ΠΆΠ΄Ρ‹ΠΉ ΠΊΡƒΠ»ΠΎΠ½ заряда. ЭнСргия, ΠΈΡΠΏΠΎΠ»ΡŒΠ·ΡƒΠ΅ΠΌΠ°Ρ зарядом Π΅Π΄ΠΈΠ½ΠΈΡ†Ρ‹ для пСрСмСщСния ΠΈΠ· ΠΎΠ΄Π½ΠΎΠΉ Ρ‚ΠΎΡ‡ΠΊΠΈ Π² Π΄Ρ€ΡƒΠ³ΡƒΡŽ
Π€ΠΎΡ€ΠΌΡƒΠ»Π°
Π‘ΠΈΠΌΠ²ΠΎΠ» Ξ΅ V
Π˜Π·ΠΌΠ΅Ρ€Π΅Π½ΠΈΠ΅ Π˜Π·ΠΌΠ΅Ρ€Π΅Π½ΠΈΠ΅ ΠΌΠ΅ΠΆΠ΄Ρƒ ΠΊΠΎΠ½Π΅Ρ‡Π½ΠΎΠΉ Ρ‚ΠΎΡ‡ΠΊΠΎΠΉ источника, ΠΊΠΎΠ³Π΄Π° Ρ‡Π΅Ρ€Π΅Π· Π½Π΅Π³ΠΎ Π½Π΅ Ρ‚Π΅Ρ‡Π΅Ρ‚ Ρ‚ΠΎΠΊ. Π˜Π·ΠΌΠ΅Ρ€ΡŒΡ‚Π΅ расстояниС ΠΌΠ΅ΠΆΠ΄Ρƒ Π»ΡŽΠ±Ρ‹ΠΌΠΈ двумя Ρ‚ΠΎΡ‡ΠΊΠ°ΠΌΠΈ.
Π˜ΡΡ‚ΠΎΡ‡Π½ΠΈΠΊ Π”ΠΈΠ½Π°ΠΌΠΎ, элСктрохимичСский элСмСнт, трансформатор, солнСчный элСмСнт, Ρ„ΠΎΡ‚ΠΎΠ΄ΠΈΠΎΠ΄Ρ‹ ΠΈ Ρ‚. Π”. ЭлСктричСскоС ΠΈ ΠΌΠ°Π³Π½ΠΈΡ‚Π½ΠΎΠ΅ ΠΏΠΎΠ»Π΅

ΠžΠΏΡ€Π΅Π΄Π΅Π»Π΅Π½ΠΈΠ΅ напряТСния

НапряТСниС опрСдСляСтся ΠΊΠ°ΠΊ энСргия, нСобходимая для пСрСмСщСния Π΅Π΄ΠΈΠ½ΠΈΡ‡Π½ΠΎΠ³ΠΎ заряда ΠΈΠ· ΠΎΠ΄Π½ΠΎΠΉ Ρ‚ΠΎΡ‡ΠΊΠΈ Π² Π΄Ρ€ΡƒΠ³ΡƒΡŽ. Оно измСряСтся Π² Π²ΠΎΠ»ΡŒΡ‚Π°Ρ… ΠΈ ​​обозначаСтся символом V. НапряТСниС вызываСтся элСктричСским ΠΈ ΠΌΠ°Π³Π½ΠΈΡ‚Π½Ρ‹ΠΌ полями.

НапряТСниС Π²ΠΎΠ·Π½ΠΈΠΊΠ°Π΅Ρ‚ ΠΌΠ΅ΠΆΠ΄Ρƒ ΠΊΠΎΠ½Ρ†Π°ΠΌΠΈ (Ρ‚.Π΅.Π΅. ΠΊΠ°Ρ‚ΠΎΠ΄ ΠΈ Π°Π½ΠΎΠ΄) источника. ΠŸΠΎΡ‚Π΅Π½Ρ†ΠΈΠ°Π» ΠΏΠΎΠ»ΠΎΠΆΠΈΡ‚Π΅Π»ΡŒΠ½ΠΎΠΉ ΠΊΠΎΠ½Π΅Ρ‡Π½ΠΎΠΉ Ρ‚ΠΎΡ‡ΠΊΠΈ источника Π²Ρ‹ΡˆΠ΅, Ρ‡Π΅ΠΌ ΠΎΡ‚Ρ€ΠΈΡ†Π°Ρ‚Π΅Π»ΡŒΠ½ΠΎΠΉ Ρ‚ΠΎΡ‡ΠΊΠΈ. Когда Π½Π° пассивном элСмСнтС Π²ΠΎΠ·Π½ΠΈΠΊΠ°Π΅Ρ‚ напряТСниС, это называСтся ΠΏΠ°Π΄Π΅Π½ΠΈΠ΅ΠΌ напряТСния. Π‘ΡƒΠΌΠΌΠ° ΠΏΠ°Π΄Π΅Π½ΠΈΠΉ напряТСния Π² Ρ†Π΅ΠΏΠΈ Ρ€Π°Π²Π½Π° Π­Π”Π‘ согласно Π·Π°ΠΊΠΎΠ½Ρƒ ΠšΠΈΡ€Ρ…Π³ΠΎΡ„Π°.

ΠžΠΏΡ€Π΅Π΄Π΅Π»Π΅Π½ΠΈΠ΅ ЭМП

ΠŸΠΎΠ΄Π°Ρ‡Π° энСргии ΠΎΡ‚ источника ΠΊ ΠΊΠ°ΠΆΠ΄ΠΎΠΌΡƒ ΠΊΡƒΠ»ΠΎΠ½Ρƒ заряда называСтся Π­Π”Π‘. Π”Ρ€ΡƒΠ³ΠΈΠΌΠΈ словами, это ΠΏΠΎΠ΄Π°Ρ‡Π° энСргии Π½Π΅ΠΊΠΎΡ‚ΠΎΡ€Ρ‹ΠΌ Π°ΠΊΡ‚ΠΈΠ²Π½Ρ‹ΠΌ источником, Ρ‚Π°ΠΊΠΈΠΌ ΠΊΠ°ΠΊ аккумулятор, для Π΅Π΄ΠΈΠ½ΠΈΡ‡Π½ΠΎΠ³ΠΎ кулоновского заряда.Π­Π”Π‘ ΠΎΠ·Π½Π°Ρ‡Π°Π΅Ρ‚ ΡΠ»Π΅ΠΊΡ‚Ρ€ΠΎΠ΄Π²ΠΈΠΆΡƒΡ‰ΡƒΡŽ силу. Он измСряСтся Π² Π²ΠΎΠ»ΡŒΡ‚Π°Ρ… ΠΈ ​​обозначаСтся символом Ξ΅.

ЭлСктродвиТущая сила Π²Ρ‹ΡˆΠ΅ΡƒΠΊΠ°Π·Π°Π½Π½ΠΎΠΉ Ρ†Π΅ΠΏΠΈ прСдставлСна ​​формулой Π³Π΄Π΅, r – Π²Π½ΡƒΡ‚Ρ€Π΅Π½Π½Π΅Π΅ сопротивлСниС Ρ†Π΅ΠΏΠΈ.
R – Π’Π½Π΅ΡˆΠ½ΠΈΠΉ рСзистан Ρ†Π΅ΠΏΠΈ.
E – элСктродвиТущая сила.
I – Ρ‚ΠΎΠΊ

ΠšΠ»ΡŽΡ‡Π΅Π²Ρ‹Π΅ различия ΠΌΠ΅ΠΆΠ΄Ρƒ Π­Π”Π‘ ΠΈ напряТСниСм

  1. Π­Π”Π‘ – это ΠΌΠ΅Ρ€Π° ΠΏΠΎΠ΄Π°Ρ‡ΠΈ энСргии Π½Π° ΠΊΠ°ΠΆΠ΄Ρ‹ΠΉ ΠΊΡƒΠ»ΠΎΠ½ заряда, Ρ‚ΠΎΠ³Π΄Π° ΠΊΠ°ΠΊ напряТСниС – это энСргия, ΠΈΡΠΏΠΎΠ»ΡŒΠ·ΡƒΠ΅ΠΌΠ°Ρ ΠΎΠ΄Π½ΠΈΠΌ ΠΊΡƒΠ»ΠΎΠ½ΠΎΠΌ заряда для пСрСмСщСния ΠΈΠ· ΠΎΠ΄Π½ΠΎΠΉ Ρ‚ΠΎΡ‡ΠΊΠΈ Π² Π΄Ρ€ΡƒΠ³ΡƒΡŽ.
  2. Π­Π”Π‘ прСдставлСна ​​буквой Ξ΅, Ρ‚ΠΎΠ³Π΄Π° ΠΊΠ°ΠΊ символичСскоС прСдставлСниС напряТСния – V.
  3. Π­Π”Π‘ измСряСтся ΠΌΠ΅ΠΆΠ΄Ρƒ ΠΊΠΎΠ½Π΅Ρ‡Π½ΠΎΠΉ Ρ‚ΠΎΡ‡ΠΊΠΎΠΉ источника, ΠΊΠΎΠ³Π΄Π° Ρ‡Π΅Ρ€Π΅Π· Π½Π΅Π³ΠΎ Π½Π΅ Ρ‚Π΅Ρ‡Π΅Ρ‚ Ρ‚ΠΎΠΊ, Π° напряТСниС измСряСтся ΠΌΠ΅ΠΆΠ΄Ρƒ Π»ΡŽΠ±Ρ‹ΠΌΠΈ двумя Ρ‚ΠΎΡ‡ΠΊΠ°ΠΌΠΈ Π·Π°ΠΌΠΊΠ½ΡƒΡ‚ΠΎΠΉ Ρ†Π΅ΠΏΠΈ.
  4. Π­Π”Π‘ создаСтся элСктрохимичСским элСмСнтом, Π΄ΠΈΠ½Π°ΠΌΠΎ-машиной, Ρ„ΠΎΡ‚ΠΎΠ΄ΠΈΠΎΠ΄Π°ΠΌΠΈ ΠΈ Ρ‚. Π”., А напряТСниС создаСтся элСктричСским ΠΈ ΠΌΠ°Π³Π½ΠΈΡ‚Π½Ρ‹ΠΌ полями.

Π’ΠΎΠ»ΡŒΡ‚ – это Π΅Π΄ΠΈΠ½ΠΈΡ†Π° БИ ΠΊΠ°ΠΊ для Π­Π”Π‘, Ρ‚Π°ΠΊ ΠΈ для напряТСния.

Π˜Π½Π΄ΡƒΡ†ΠΈΡ€ΠΎΠ²Π°Π½Π½Π°Ρ Π­Π”Π‘ ΠΈ ΠΌΠ°Π³Π½ΠΈΡ‚Π½Ρ‹ΠΉ ΠΏΠΎΡ‚ΠΎΠΊ – College Physics

Π¦Π΅Π»ΠΈ обучСния

  • РассчитайтС ΠΏΠΎΡ‚ΠΎΠΊ ΠΎΠ΄Π½ΠΎΡ€ΠΎΠ΄Π½ΠΎΠ³ΠΎ ΠΌΠ°Π³Π½ΠΈΡ‚Π½ΠΎΠ³ΠΎ поля Ρ‡Π΅Ρ€Π΅Π· ΠΏΠ΅Ρ‚Π»ΡŽ ΠΏΡ€ΠΎΠΈΠ·Π²ΠΎΠ»ΡŒΠ½ΠΎΠΉ ΠΎΡ€ΠΈΠ΅Π½Ρ‚Π°Ρ†ΠΈΠΈ.
  • ΠžΠΏΠΈΡˆΠΈΡ‚Π΅ ΠΌΠ΅Ρ‚ΠΎΠ΄Ρ‹ создания элСктродвиТущСй силы (Π­Π”Π‘) с ΠΏΠΎΠΌΠΎΡ‰ΡŒΡŽ ΠΌΠ°Π³Π½ΠΈΡ‚Π½ΠΎΠ³ΠΎ поля ΠΈΠ»ΠΈ ΠΌΠ°Π³Π½ΠΈΡ‚Π° ΠΈ ΠΏΡ€ΠΎΠ²ΠΎΠ»ΠΎΡ‡Π½ΠΎΠΉ ΠΏΠ΅Ρ‚Π»ΠΈ.

Аппарат, ΠΈΡΠΏΠΎΠ»ΡŒΠ·ΠΎΠ²Π°Π½Π½Ρ‹ΠΉ Π€Π°Ρ€Π°Π΄Π΅Π΅ΠΌ для дСмонстрации Ρ‚ΠΎΠ³ΠΎ, Ρ‡Ρ‚ΠΎ ΠΌΠ°Π³Π½ΠΈΡ‚Π½Ρ‹Π΅ поля ΠΌΠΎΠ³ΡƒΡ‚ ΡΠΎΠ·Π΄Π°Π²Π°Ρ‚ΡŒ Ρ‚ΠΎΠΊΠΈ, ΠΏΠΎΠΊΠ°Π·Π°Π½ Π½Π° (Рисунок). Когда ΠΏΠ΅Ρ€Π΅ΠΊΠ»ΡŽΡ‡Π°Ρ‚Π΅Π»ΡŒ Π·Π°ΠΌΠΊΠ½ΡƒΡ‚, Π² ΠΊΠ°Ρ‚ΡƒΡˆΠΊΠ΅ Π² Π²Π΅Ρ€Ρ…Π½Π΅ΠΉ части ΠΆΠ΅Π»Π΅Π·Π½ΠΎΠ³ΠΎ ΠΊΠΎΠ»ΡŒΡ†Π° создаСтся ΠΌΠ°Π³Π½ΠΈΡ‚Π½ΠΎΠ΅ ΠΏΠΎΠ»Π΅, ΠΊΠΎΡ‚ΠΎΡ€ΠΎΠ΅ пСрСдаСтся ΠΊΠ°Ρ‚ΡƒΡˆΠΊΠ΅ Π² Π½ΠΈΠΆΠ½Π΅ΠΉ части ΠΊΠΎΠ»ΡŒΡ†Π°.Π“Π°Π»ΡŒΠ²Π°Π½ΠΎΠΌΠ΅Ρ‚Ρ€ ΠΈΡΠΏΠΎΠ»ΡŒΠ·ΡƒΠ΅Ρ‚ΡΡ для обнаруТСния любого Ρ‚ΠΎΠΊΠ°, Π½Π°Π²Π΅Π΄Π΅Π½Π½ΠΎΠ³ΠΎ Π² ΠΊΠ°Ρ‚ΡƒΡˆΠΊΠ΅ Π²Π½ΠΈΠ·Ρƒ. Π‘Ρ‹Π»ΠΎ ΠΎΠ±Π½Π°Ρ€ΡƒΠΆΠ΅Π½ΠΎ, Ρ‡Ρ‚ΠΎ ΠΊΠ°ΠΆΠ΄Ρ‹ΠΉ Ρ€Π°Π·, ΠΊΠΎΠ³Π΄Π° ΠΏΠ΅Ρ€Π΅ΠΊΠ»ΡŽΡ‡Π°Ρ‚Π΅Π»ΡŒ замыкаСтся, Π³Π°Π»ΡŒΠ²Π°Π½ΠΎΠΌΠ΅Ρ‚Ρ€ ΠΎΠ±Π½Π°Ρ€ΡƒΠΆΠΈΠ²Π°Π΅Ρ‚ Ρ‚ΠΎΠΊ Π² ΠΎΠ΄Π½ΠΎΠΌ Π½Π°ΠΏΡ€Π°Π²Π»Π΅Π½ΠΈΠΈ Π² ΠΊΠ°Ρ‚ΡƒΡˆΠΊΠ΅ Π²Π½ΠΈΠ·Ρƒ. (Π’Ρ‹ Ρ‚Π°ΠΊΠΆΠ΅ ΠΌΠΎΠΆΠ΅Ρ‚Π΅ Π½Π°Π±Π»ΡŽΠ΄Π°Ρ‚ΡŒ это Π² физичСской Π»Π°Π±ΠΎΡ€Π°Ρ‚ΠΎΡ€ΠΈΠΈ.) ΠšΠ°ΠΆΠ΄Ρ‹ΠΉ Ρ€Π°Π·, ΠΊΠΎΠ³Π΄Π° ΠΏΠ΅Ρ€Π΅ΠΊΠ»ΡŽΡ‡Π°Ρ‚Π΅Π»ΡŒ открываСтся, Π³Π°Π»ΡŒΠ²Π°Π½ΠΎΠΌΠ΅Ρ‚Ρ€ ΠΎΠ±Π½Π°Ρ€ΡƒΠΆΠΈΠ²Π°Π΅Ρ‚ Ρ‚ΠΎΠΊ Π² ΠΏΡ€ΠΎΡ‚ΠΈΠ²ΠΎΠΏΠΎΠ»ΠΎΠΆΠ½ΠΎΠΌ Π½Π°ΠΏΡ€Π°Π²Π»Π΅Π½ΠΈΠΈ. Π˜Π½Ρ‚Π΅Ρ€Π΅ΡΠ½ΠΎ, Ρ‡Ρ‚ΠΎ Ссли ΠΏΠ΅Ρ€Π΅ΠΊΠ»ΡŽΡ‡Π°Ρ‚Π΅Π»ΡŒ остаСтся Π·Π°ΠΌΠΊΠ½ΡƒΡ‚Ρ‹ΠΌ ΠΈΠ»ΠΈ Ρ€Π°Π·ΠΎΠΌΠΊΠ½ΡƒΡ‚Ρ‹ΠΌ Π² Ρ‚Π΅Ρ‡Π΅Π½ΠΈΠ΅ Π½Π΅ΠΊΠΎΡ‚ΠΎΡ€ΠΎΠ³ΠΎ Π²Ρ€Π΅ΠΌΠ΅Π½ΠΈ, Ρ‡Π΅Ρ€Π΅Π· Π³Π°Π»ΡŒΠ²Π°Π½ΠΎΠΌΠ΅Ρ‚Ρ€ Π½Π΅Ρ‚ Ρ‚ΠΎΠΊΠ°. Π—Π°ΠΌΡ‹ΠΊΠ°Π½ΠΈΠ΅ ΠΈ Ρ€Π°Π·ΠΌΡ‹ΠΊΠ°Π½ΠΈΠ΅ ΠΏΠ΅Ρ€Π΅ΠΊΠ»ΡŽΡ‡Π°Ρ‚Π΅Π»Ρ ΠΈΠ½Π΄ΡƒΡ†ΠΈΡ€ΡƒΠ΅Ρ‚ Ρ‚ΠΎΠΊ.Π­Ρ‚ΠΎ ΠΈΠ·ΠΌΠ΅Π½Π΅Π½ΠΈΠ΅ Π² ΠΌΠ°Π³Π½ΠΈΡ‚Π½ΠΎΠΌ ΠΏΠΎΠ»Π΅, ΠΊΠΎΡ‚ΠΎΡ€ΠΎΠ΅ создаСт Ρ‚ΠΎΠΊ. Π‘ΠΎΠ»Π΅Π΅ основным, Ρ‡Π΅ΠΌ Ρ‚Π΅ΠΊΡƒΡ‰ΠΈΠΉ Ρ‚ΠΎΠΊ, являСтся Π­Π”Π‘ , которая Π΅Π³ΠΎ Π²Ρ‹Π·Ρ‹Π²Π°Π΅Ρ‚. Π’ΠΎΠΊ являСтся Ρ€Π΅Π·ΡƒΠ»ΡŒΡ‚Π°Ρ‚ΠΎΠΌ Π­Π”Π‘ , ΠΈΠ½Π΄ΡƒΡ†ΠΈΡ€ΠΎΠ²Π°Π½Π½ΠΎΠΉ ΠΈΠ·ΠΌΠ΅Π½ΡΡŽΡ‰ΠΈΠΌΡΡ ΠΌΠ°Π³Π½ΠΈΡ‚Π½Ρ‹ΠΌ ΠΏΠΎΠ»Π΅ΠΌ , нСзависимо ΠΎΡ‚ Ρ‚ΠΎΠ³ΠΎ, Π΅ΡΡ‚ΡŒ Π»ΠΈ ΠΏΡƒΡ‚ΡŒ для протСкания Ρ‚ΠΎΠΊΠ°.

Аппарат ЀарадСя для дСмонстрации Ρ‚ΠΎΠ³ΠΎ, Ρ‡Ρ‚ΠΎ ΠΌΠ°Π³Π½ΠΈΡ‚Π½ΠΎΠ΅ ΠΏΠΎΠ»Π΅ ΠΌΠΎΠΆΠ΅Ρ‚ ΠΏΡ€ΠΎΠΈΠ·Π²ΠΎΠ΄ΠΈΡ‚ΡŒ Ρ‚ΠΎΠΊ. ИзмСнСниС поля, создаваСмого Π²Π΅Ρ€Ρ…Π½Π΅ΠΉ ΠΊΠ°Ρ‚ΡƒΡˆΠΊΠΎΠΉ, Π²Ρ‹Π·Ρ‹Π²Π°Π΅Ρ‚ Π­Π”Π‘ ΠΈ, ΡΠ»Π΅Π΄ΠΎΠ²Π°Ρ‚Π΅Π»ΡŒΠ½ΠΎ, Ρ‚ΠΎΠΊ Π² Π½ΠΈΠΆΠ½Π΅ΠΉ ΠΊΠ°Ρ‚ΡƒΡˆΠΊΠ΅. Когда ΠΏΠ΅Ρ€Π΅ΠΊΠ»ΡŽΡ‡Π°Ρ‚Π΅Π»ΡŒ Ρ€Π°Π·ΠΎΠΌΠΊΠ½ΡƒΡ‚ ΠΈ Π·Π°ΠΌΠΊΠ½ΡƒΡ‚, Π³Π°Π»ΡŒΠ²Π°Π½ΠΎΠΌΠ΅Ρ‚Ρ€ рСгистрируСт Ρ‚ΠΎΠΊΠΈ Π² ΠΏΡ€ΠΎΡ‚ΠΈΠ²ΠΎΠΏΠΎΠ»ΠΎΠΆΠ½Ρ‹Ρ… направлСниях.Когда ΠΏΠ΅Ρ€Π΅ΠΊΠ»ΡŽΡ‡Π°Ρ‚Π΅Π»ΡŒ остаСтся Π·Π°ΠΌΠΊΠ½ΡƒΡ‚Ρ‹ΠΌ ΠΈΠ»ΠΈ Ρ€Π°Π·ΠΎΠΌΠΊΠ½ΡƒΡ‚Ρ‹ΠΌ, Ρ‡Π΅Ρ€Π΅Π· Π³Π°Π»ΡŒΠ²Π°Π½ΠΎΠΌΠ΅Ρ‚Ρ€ Π½Π΅ Ρ‚Π΅Ρ‡Π΅Ρ‚ Ρ‚ΠΎΠΊ.

ЭкспСримСнт, ΠΊΠΎΡ‚ΠΎΡ€Ρ‹ΠΉ Π»Π΅Π³ΠΊΠΎ выполняСтся ΠΈ часто проводится Π² физичСских лабораториях, ΠΏΠΎΠΊΠ°Π·Π°Π½ Π½Π° (Рисунок). Π­Π”Π‘ индуцируСтся Π² ΠΊΠ°Ρ‚ΡƒΡˆΠΊΠ΅, ΠΊΠΎΠ³Π΄Π° стСрТнСвой ΠΌΠ°Π³Π½ΠΈΡ‚ вставляСтся ΠΈ Π²Ρ‹Ρ…ΠΎΠ΄ΠΈΡ‚ ΠΈΠ· Π½Π΅Π΅. Π­Π”Π‘ ΠΏΡ€ΠΎΡ‚ΠΈΠ²ΠΎΠΏΠΎΠ»ΠΎΠΆΠ½Ρ‹Ρ… Π·Π½Π°ΠΊΠΎΠ² ΡΠΎΠ·Π΄Π°ΡŽΡ‚ΡΡ Π΄Π²ΠΈΠΆΠ΅Π½ΠΈΠ΅ΠΌ Π² ΠΏΡ€ΠΎΡ‚ΠΈΠ²ΠΎΠΏΠΎΠ»ΠΎΠΆΠ½Ρ‹Ρ… направлСниях, ΠΈ Π­Π”Π‘ Ρ‚Π°ΠΊΠΆΠ΅ ΠΌΠ΅Π½ΡΡŽΡ‚ΡΡ Π½Π° ΠΏΡ€ΠΎΡ‚ΠΈΠ²ΠΎΠΏΠΎΠ»ΠΎΠΆΠ½Ρ‹Π΅ Π·Π° счСт измСнСния полюсов. Π’Π΅ ΠΆΠ΅ Ρ€Π΅Π·ΡƒΠ»ΡŒΡ‚Π°Ρ‚Ρ‹ Π±ΡƒΠ΄ΡƒΡ‚ ΠΏΠΎΠ»ΡƒΡ‡Π΅Π½Ρ‹, Ссли ΠΏΠ΅Ρ€Π΅ΠΌΠ΅Ρ‰Π°Ρ‚ΡŒ ΠΊΠ°Ρ‚ΡƒΡˆΠΊΡƒ, Π° Π½Π΅ ΠΌΠ°Π³Π½ΠΈΡ‚ – Π²Π°ΠΆΠ½ΠΎ ΠΎΡ‚Π½ΠΎΡΠΈΡ‚Π΅Π»ΡŒΠ½ΠΎΠ΅ Π΄Π²ΠΈΠΆΠ΅Π½ΠΈΠ΅.Π§Π΅ΠΌ быстрСС Π΄Π²ΠΈΠΆΠ΅Π½ΠΈΠ΅, Ρ‚Π΅ΠΌ большС Π­Π”Π‘, ΠΈ ΠΊΠΎΠ³Π΄Π° ΠΌΠ°Π³Π½ΠΈΡ‚ Π½Π΅ΠΏΠΎΠ΄Π²ΠΈΠΆΠ΅Π½ ΠΎΡ‚Π½ΠΎΡΠΈΡ‚Π΅Π»ΡŒΠ½ΠΎ ΠΊΠ°Ρ‚ΡƒΡˆΠΊΠΈ, Π­Π”Π‘ отсутствуСт.

Π”Π²ΠΈΠΆΠ΅Π½ΠΈΠ΅ ΠΌΠ°Π³Π½ΠΈΡ‚Π° ΠΎΡ‚Π½ΠΎΡΠΈΡ‚Π΅Π»ΡŒΠ½ΠΎ ΠΊΠ°Ρ‚ΡƒΡˆΠΊΠΈ создаСт Π­Π”Π‘, ΠΊΠ°ΠΊ ΠΏΠΎΠΊΠ°Π·Π°Π½ΠΎ Π½Π° рисункС. Π’Π°ΠΊΠΈΠ΅ ΠΆΠ΅ Π­Π”Π‘ Π²ΠΎΠ·Π½ΠΈΠΊΠ°ΡŽΡ‚ ΠΏΡ€ΠΈ ΠΏΠ΅Ρ€Π΅ΠΌΠ΅Ρ‰Π΅Π½ΠΈΠΈ ΠΊΠ°Ρ‚ΡƒΡˆΠΊΠΈ ΠΎΡ‚Π½ΠΎΡΠΈΡ‚Π΅Π»ΡŒΠ½ΠΎ ΠΌΠ°Π³Π½ΠΈΡ‚Π°. Π§Π΅ΠΌ большС ΡΠΊΠΎΡ€ΠΎΡΡ‚ΡŒ, Ρ‚Π΅ΠΌ большС Π²Π΅Π»ΠΈΡ‡ΠΈΠ½Π° Π­Π”Π‘, Π° ΠΏΡ€ΠΈ отсутствии двиТСния Π­Π”Π‘ Ρ€Π°Π²Π½Π° Π½ΡƒΠ»ΡŽ.

ΠœΠ΅Ρ‚ΠΎΠ΄ ΠΈΠ½Π΄ΡƒΠΊΡ†ΠΈΠΈ Π­Π”Π‘, ΠΈΡΠΏΠΎΠ»ΡŒΠ·ΡƒΠ΅ΠΌΡ‹ΠΉ Π² Π±ΠΎΠ»ΡŒΡˆΠΈΠ½ΡΡ‚Π²Π΅ элСктричСских Π³Π΅Π½Π΅Ρ€Π°Ρ‚ΠΎΡ€ΠΎΠ², ΠΏΠΎΠΊΠ°Π·Π°Π½ Π½Π° (Рисунок). ΠšΠ°Ρ‚ΡƒΡˆΠΊΠ° вращаСтся Π² ΠΌΠ°Π³Π½ΠΈΡ‚Π½ΠΎΠΌ ΠΏΠΎΠ»Π΅, создавая Π­Π”Π‘ ΠΏΠ΅Ρ€Π΅ΠΌΠ΅Π½Π½ΠΎΠ³ΠΎ Ρ‚ΠΎΠΊΠ°, которая зависит ΠΎΡ‚ скорости вращСния ΠΈ Π΄Ρ€ΡƒΠ³ΠΈΡ… Ρ„Π°ΠΊΡ‚ΠΎΡ€ΠΎΠ², ΠΊΠΎΡ‚ΠΎΡ€Ρ‹Π΅ Π±ΡƒΠ΄ΡƒΡ‚ исслСдованы Π² ΡΠ»Π΅Π΄ΡƒΡŽΡ‰ΠΈΡ… Ρ€Π°Π·Π΄Π΅Π»Π°Ρ….ΠžΠ±Ρ€Π°Ρ‚ΠΈΡ‚Π΅ Π²Π½ΠΈΠΌΠ°Π½ΠΈΠ΅, Ρ‡Ρ‚ΠΎ Π³Π΅Π½Π΅Ρ€Π°Ρ‚ΠΎΡ€ ΠΎΡ‡Π΅Π½ΡŒ ΠΏΠΎΡ…ΠΎΠΆ ΠΏΠΎ конструкции Π½Π° Π΄Π²ΠΈΠ³Π°Ρ‚Π΅Π»ΡŒ (другая симмСтрия).

ΠŸΡ€ΠΈ Π²Ρ€Π°Ρ‰Π΅Π½ΠΈΠΈ ΠΊΠ°Ρ‚ΡƒΡˆΠΊΠΈ Π² ΠΌΠ°Π³Π½ΠΈΡ‚Π½ΠΎΠΌ ΠΏΠΎΠ»Π΅ Π²ΠΎΠ·Π½ΠΈΠΊΠ°Π΅Ρ‚ Π­Π”Π‘. Π­Ρ‚ΠΎ основная конструкция Π³Π΅Π½Π΅Ρ€Π°Ρ‚ΠΎΡ€Π°, Π² ΠΊΠΎΡ‚ΠΎΡ€ΠΎΠΌ Ρ€Π°Π±ΠΎΡ‚Π°, выполняСмая ΠΏΠΎ Π²Ρ€Π°Ρ‰Π΅Π½ΠΈΡŽ ΠΊΠ°Ρ‚ΡƒΡˆΠΊΠΈ, прСобразуСтся Π² ΡΠ»Π΅ΠΊΡ‚Ρ€ΠΈΡ‡Π΅ΡΠΊΡƒΡŽ ΡΠ½Π΅Ρ€Π³ΠΈΡŽ. ΠžΠ±Ρ€Π°Ρ‚ΠΈΡ‚Π΅ Π²Π½ΠΈΠΌΠ°Π½ΠΈΠ΅, Ρ‡Ρ‚ΠΎ Π³Π΅Π½Π΅Ρ€Π°Ρ‚ΠΎΡ€ ΠΎΡ‡Π΅Π½ΡŒ ΠΏΠΎΡ…ΠΎΠΆ ΠΏΠΎ конструкции Π½Π° Π΄Π²ΠΈΠ³Π°Ρ‚Π΅Π»ΡŒ.

Π˜Ρ‚Π°ΠΊ, ΠΌΡ‹ Π²ΠΈΠ΄ΠΈΠΌ, Ρ‡Ρ‚ΠΎ ΠΈΠ·ΠΌΠ΅Π½Π΅Π½ΠΈΠ΅ Π²Π΅Π»ΠΈΡ‡ΠΈΠ½Ρ‹ ΠΈΠ»ΠΈ направлСния ΠΌΠ°Π³Π½ΠΈΡ‚Π½ΠΎΠ³ΠΎ поля Π²Ρ‹Π·Ρ‹Π²Π°Π΅Ρ‚ Π­Π”Π‘. ЭкспСримСнты ΠΏΠΎΠΊΠ°Π·Π°Π»ΠΈ, Ρ‡Ρ‚ΠΎ сущСствуСт критичСская Π²Π΅Π»ΠΈΡ‡ΠΈΠ½Π°, называСмая ΠΌΠ°Π³Π½ΠΈΡ‚Π½Ρ‹ΠΌ ΠΏΠΎΡ‚ΠΎΠΊΠΎΠΌ, опрСдСляСмая ΡΠΎΠΎΡ‚Π½ΠΎΡˆΠ΅Π½ΠΈΠ΅ΠΌ

.

, Π³Π΄Π΅ – Π½Π°ΠΏΡ€ΡΠΆΠ΅Π½Π½ΠΎΡΡ‚ΡŒ ΠΌΠ°Π³Π½ΠΈΡ‚Π½ΠΎΠ³ΠΎ поля Π² области ΠΏΠΎΠ΄ ΡƒΠ³Π»ΠΎΠΌ ΠΊ ​​пСрпСндикуляру ΠΊ области, ΠΊΠ°ΠΊ ΠΏΠΎΠΊΠ°Π·Π°Π½ΠΎ Π½Π° (Рисунок). Π›ΡŽΠ±ΠΎΠ΅ ΠΈΠ·ΠΌΠ΅Π½Π΅Π½ΠΈΠ΅ ΠΌΠ°Π³Π½ΠΈΡ‚Π½ΠΎΠ³ΠΎ ΠΏΠΎΡ‚ΠΎΠΊΠ° Π²Ρ‹Π·Ρ‹Π²Π°Π΅Ρ‚ Π­Π”Π‘. Π­Ρ‚ΠΎΡ‚ процСсс опрСдСляСтся ΠΊΠ°ΠΊ элСктромагнитная индукция. Π•Π΄ΠΈΠ½ΠΈΡ†Ρ‹ ΠΌΠ°Π³Π½ΠΈΡ‚Π½ΠΎΠ³ΠΎ ΠΏΠΎΡ‚ΠΎΠΊΠ° Π°Ρ€. Как Π²ΠΈΠ΄Π½ΠΎ Π½Π° (Рисунок), βŠ₯ , ΠΊΠΎΡ‚ΠΎΡ€Ρ‹ΠΉ являСтся ΡΠΎΡΡ‚Π°Π²Π»ΡΡŽΡ‰Π΅ΠΉ , пСрпСндикулярной области . Π’Π°ΠΊΠΈΠΌ ΠΎΠ±Ρ€Π°Π·ΠΎΠΌ, ΠΌΠ°Π³Π½ΠΈΡ‚Π½Ρ‹ΠΉ ΠΏΠΎΡ‚ΠΎΠΊ являСтся ΠΏΡ€ΠΎΠΈΠ·Π²Π΅Π΄Π΅Π½ΠΈΠ΅ΠΌ ΠΏΠ»ΠΎΡ‰Π°Π΄ΠΈ ΠΈ ΡΠΎΡΡ‚Π°Π²Π»ΡΡŽΡ‰Π΅ΠΉ ΠΌΠ°Π³Π½ΠΈΡ‚Π½ΠΎΠ³ΠΎ поля, пСрпСндикулярной Π΅ΠΉ.

ΠœΠ°Π³Π½ΠΈΡ‚Π½Ρ‹ΠΉ ΠΏΠΎΡ‚ΠΎΠΊ связан с ΠΌΠ°Π³Π½ΠΈΡ‚Π½Ρ‹ΠΌ ΠΏΠΎΠ»Π΅ΠΌ ΠΈ ΠΏΠ»ΠΎΡ‰Π°Π΄ΡŒΡŽ, Π½Π° ΠΊΠΎΡ‚ΠΎΡ€ΠΎΠΉ ΠΎΠ½ сущСствуСт. ΠŸΠΎΡ‚ΠΎΠΊ связан с ΠΈΠ½Π΄ΡƒΠΊΡ†ΠΈΠ΅ΠΉ; любоС ΠΈΠ·ΠΌΠ΅Π½Π΅Π½ΠΈΠ΅ Π²Ρ‹Π·Ρ‹Π²Π°Π΅Ρ‚ Π­Π”Π‘.

Вся индукция, Π²ΠΊΠ»ΡŽΡ‡Π°Ρ ΠΏΡ€ΠΈΠ²Π΅Π΄Π΅Π½Π½Ρ‹Π΅ Π΄ΠΎ сих ΠΏΠΎΡ€ ΠΏΡ€ΠΈΠΌΠ΅Ρ€Ρ‹, Π²ΠΎΠ·Π½ΠΈΠΊΠ°Π΅Ρ‚ ΠΈΠ·-Π·Π° Π½Π΅ΠΊΠΎΡ‚ΠΎΡ€ΠΎΠ³ΠΎ измСнСния ΠΌΠ°Π³Π½ΠΈΡ‚Π½ΠΎΠ³ΠΎ ΠΏΠΎΡ‚ΠΎΠΊΠ° . НапримСр, Π€Π°Ρ€Π°Π΄Π΅ΠΉ ΠΈΠ·ΠΌΠ΅Π½ΠΈΠ» ΠΈ, ΡΠ»Π΅Π΄ΠΎΠ²Π°Ρ‚Π΅Π»ΡŒΠ½ΠΎ, ΠΏΡ€ΠΈ Ρ€Π°Π·ΠΌΡ‹ΠΊΠ°Π½ΠΈΠΈ ΠΈ Π·Π°ΠΌΡ‹ΠΊΠ°Π½ΠΈΠΈ ΠΏΠ΅Ρ€Π΅ΠΊΠ»ΡŽΡ‡Π°Ρ‚Π΅Π»Ρ Π² своСм устройствС (ΠΏΠΎΠΊΠ°Π·Π°Π½ΠΎ Π½Π° (Рисунок)). Π­Ρ‚ΠΎ Ρ‚Π°ΠΊΠΆΠ΅ Π²Π΅Ρ€Π½ΠΎ для стСрТнСвого ΠΌΠ°Π³Π½ΠΈΡ‚Π° ΠΈ ΠΊΠ°Ρ‚ΡƒΡˆΠΊΠΈ, ΠΏΠΎΠΊΠ°Π·Π°Π½Π½Ρ‹Ρ… Π½Π° (Рисунок). ΠŸΡ€ΠΈ Π²Ρ€Π°Ρ‰Π΅Π½ΠΈΠΈ ΠΊΠ°Ρ‚ΡƒΡˆΠΊΠΈ Π³Π΅Π½Π΅Ρ€Π°Ρ‚ΠΎΡ€Π° ΡƒΠ³ΠΎΠ» ΠΈ, ΡΠ»Π΅Π΄ΠΎΠ²Π°Ρ‚Π΅Π»ΡŒΠ½ΠΎ, измСняСтся. Насколько Π²Π΅Π»ΠΈΠΊΠ° Π­Π”Π‘ ΠΈ ΠΊΠ°ΠΊΠΎΠ΅ Π½Π°ΠΏΡ€Π°Π²Π»Π΅Π½ΠΈΠ΅ ΠΎΠ½Π° ΠΏΡ€ΠΈΠ½ΠΈΠΌΠ°Π΅Ρ‚, зависит ΠΎΡ‚ измСнСния Π² ΠΈ ΠΎΡ‚ Ρ‚ΠΎΠ³ΠΎ, ΠΊΠ°ΠΊ быстро это ΠΈΠ·ΠΌΠ΅Π½Π΅Π½ΠΈΠ΅ Π±ΡƒΠ΄Π΅Ρ‚ Π²Ρ‹ΠΏΠΎΠ»Π½Π΅Π½ΠΎ, ΠΊΠ°ΠΊ Π±ΡƒΠ΄Π΅Ρ‚ рассмотрСно Π² ΡΠ»Π΅Π΄ΡƒΡŽΡ‰Π΅ΠΌ Ρ€Π°Π·Π΄Π΅Π»Π΅.

ΠšΠΎΠ½Ρ†Π΅ΠΏΡ‚ΡƒΠ°Π»ΡŒΠ½Ρ‹Π΅ вопросы

Каким ΠΎΠ±Ρ€Π°Π·ΠΎΠΌ ΠΌΠ½ΠΎΠ³ΠΎΠΏΠ΅Ρ‚Π»Π΅Π²Ρ‹Π΅ ΠΊΠ°Ρ‚ΡƒΡˆΠΊΠΈ ΠΈ ΠΆΠ΅Π»Π΅Π·Π½ΠΎΠ΅ ΠΊΠΎΠ»ΡŒΡ†ΠΎ Π² вСрсии Π°ΠΏΠΏΠ°Ρ€Π°Ρ‚Π° ЀарадСя, ΠΏΠΎΠΊΠ°Π·Π°Π½Π½ΠΎΠΉ Π½Π° (Рисунок), ΡƒΠ»ΡƒΡ‡ΡˆΠ°ΡŽΡ‚ наблюдСниС Π½Π°Π²Π΅Π΄Π΅Π½Π½ΠΎΠΉ Π­Π”Π‘?

Когда ΠΌΠ°Π³Π½ΠΈΡ‚ вставляСтся Π² ΠΊΠ°Ρ‚ΡƒΡˆΠΊΡƒ, ΠΊΠ°ΠΊ ΠΏΠΎΠΊΠ°Π·Π°Π½ΠΎ Π½Π° (Рисунок) (Π°), Π² ΠΊΠ°ΠΊΠΎΠΌ Π½Π°ΠΏΡ€Π°Π²Π»Π΅Π½ΠΈΠΈ ΠΊΠ°Ρ‚ΡƒΡˆΠΊΠ° воздСйствуСт Π½Π° ΠΌΠ°Π³Π½ΠΈΡ‚? НарисуйтС Π΄ΠΈΠ°Π³Ρ€Π°ΠΌΠΌΡƒ, ΠΏΠΎΠΊΠ°Π·Ρ‹Π²Π°ΡŽΡ‰ΡƒΡŽ Π½Π°ΠΏΡ€Π°Π²Π»Π΅Π½ΠΈΠ΅ Ρ‚ΠΎΠΊΠ°, ΠΈΠ½Π΄ΡƒΡ†ΠΈΡ€ΡƒΠ΅ΠΌΠΎΠ³ΠΎ Π² ΠΊΠ°Ρ‚ΡƒΡˆΠΊΠ΅, ΠΈ создаваСмоС Сю ΠΌΠ°Π³Π½ΠΈΡ‚Π½ΠΎΠ΅ ΠΏΠΎΠ»Π΅, Ρ‡Ρ‚ΠΎΠ±Ρ‹ ΠΎΠ±ΠΎΡΠ½ΠΎΠ²Π°Ρ‚ΡŒ Π²Π°ΡˆΡƒ Ρ€Π΅Π°ΠΊΡ†ΠΈΡŽ. Как Π²Π΅Π»ΠΈΡ‡ΠΈΠ½Π° силы зависит ΠΎΡ‚ сопротивлСния Π³Π°Π»ΡŒΠ²Π°Π½ΠΎΠΌΠ΅Ρ‚Ρ€Π°?

ΠžΠ±ΡŠΡΡΠ½ΠΈΡ‚Π΅, ΠΊΠ°ΠΊ ΠΌΠ°Π³Π½ΠΈΡ‚Π½Ρ‹ΠΉ ΠΏΠΎΡ‚ΠΎΠΊ ΠΌΠΎΠΆΠ΅Ρ‚ Π±Ρ‹Ρ‚ΡŒ Ρ€Π°Π²Π΅Π½ Π½ΡƒΠ»ΡŽ, ΠΊΠΎΠ³Π΄Π° ΠΌΠ°Π³Π½ΠΈΡ‚Π½ΠΎΠ΅ ΠΏΠΎΠ»Π΅ Π½Π΅ Ρ€Π°Π²Π½ΠΎ Π½ΡƒΠ»ΡŽ.

НавСдСна Π»ΠΈ Π­Π”Π‘ Π² ΠΊΠ°Ρ‚ΡƒΡˆΠΊΠ΅ (рисунок), ΠΊΠΎΠ³Π΄Π° ΠΎΠ½Π° растягиваСтся? Если Π΄Π°, ΡƒΠΊΠ°ΠΆΠΈΡ‚Π΅ ΠΏΡ€ΠΈΡ‡ΠΈΠ½Ρƒ ΠΈ ΡƒΠΊΠ°ΠΆΠΈΡ‚Π΅ Π½Π°ΠΏΡ€Π°Π²Π»Π΅Π½ΠΈΠ΅ ΠΈΠ½Π΄ΡƒΡ†ΠΈΡ€ΠΎΠ²Π°Π½Π½ΠΎΠ³ΠΎ Ρ‚ΠΎΠΊΠ°.

ΠšΡ€ΡƒΠ³Π»Π°Ρ ΠΊΠ°Ρ‚ΡƒΡˆΠΊΠ° с ΠΏΡ€ΠΎΠ²ΠΎΠ»ΠΎΠΊΠΎΠΉ натянута Π² ΠΌΠ°Π³Π½ΠΈΡ‚Π½ΠΎΠΌ ΠΏΠΎΠ»Π΅.

Π—Π°Π΄Π°Ρ‡ΠΈ ΠΈ упраТнСния

КакоС Π·Π½Π°Ρ‡Π΅Π½ΠΈΠ΅ ΠΌΠ°Π³Π½ΠΈΡ‚Π½ΠΎΠ³ΠΎ ΠΏΠΎΡ‚ΠΎΠΊΠ° Π² ΠΊΠ°Ρ‚ΡƒΡˆΠΊΠ΅ 2 (рисунок) ΠΈΠ·-Π·Π° ΠΊΠ°Ρ‚ΡƒΡˆΠΊΠΈ 1?

(a) ΠŸΠ»ΠΎΡΠΊΠΎΡΡ‚ΠΈ Π΄Π²ΡƒΡ… ΠΊΠ°Ρ‚ΡƒΡˆΠ΅ΠΊ пСрпСндикулярны. (Π±) ΠŸΡ€ΠΎΠ²ΠΎΠ»ΠΎΠΊΠ° пСрпСндикулярна плоскости ΠΊΠ°Ρ‚ΡƒΡˆΠΊΠΈ.

КакоС Π·Π½Π°Ρ‡Π΅Π½ΠΈΠ΅ ΠΌΠ°Π³Π½ΠΈΡ‚Π½ΠΎΠ³ΠΎ ΠΏΠΎΡ‚ΠΎΠΊΠ°, проходящСго Ρ‡Π΅Ρ€Π΅Π· ΠΊΠ°Ρ‚ΡƒΡˆΠΊΡƒ Π½Π° (Рисунок) (b), обусловлСно ΠΏΡ€ΠΎΠ²ΠΎΠ΄ΠΎΠΌ?

Глоссарий

ΠΌΠ°Π³Π½ΠΈΡ‚Π½Ρ‹ΠΉ ΠΏΠΎΡ‚ΠΎΠΊ
Π²Π΅Π»ΠΈΡ‡ΠΈΠ½Π° ΠΌΠ°Π³Π½ΠΈΡ‚Π½ΠΎΠ³ΠΎ поля, проходящСго Ρ‡Π΅Ρ€Π΅Π· ΠΊΠΎΠ½ΠΊΡ€Π΅Ρ‚Π½ΡƒΡŽ ΠΎΠ±Π»Π°ΡΡ‚ΡŒ, вычисляСмая ΠΏΠΎ Ρ„ΠΎΡ€ΠΌΡƒΠ»Π΅, Π³Π΄Π΅ – Π½Π°ΠΏΡ€ΡΠΆΠ΅Π½Π½ΠΎΡΡ‚ΡŒ ΠΌΠ°Π³Π½ΠΈΡ‚Π½ΠΎΠ³ΠΎ поля Π² области ΠΏΠΎΠ΄ ΡƒΠ³Π»ΠΎΠΌ ΠΊ ​​пСрпСндикуляру ΠΊ области
элСктромагнитная индукция
процСсс навСдСния Π­Π”Π‘ (напряТСния) с ΠΈΠ·ΠΌΠ΅Π½Π΅Π½ΠΈΠ΅ΠΌ ΠΌΠ°Π³Π½ΠΈΡ‚Π½ΠΎΠ³ΠΎ ΠΏΠΎΡ‚ΠΎΠΊΠ°

Π­Π”Π‘ ΠΈ напряТСниС Π½Π° ΠΊΠ»Π΅ΠΌΠΌΠ°Ρ…

Когда Π²Ρ‹ Π·Π°Π±Ρ‹Π²Π°Π΅Ρ‚Π΅ Π²Ρ‹ΠΊΠ»ΡŽΡ‡ΠΈΡ‚ΡŒ Π°Π²Ρ‚ΠΎΠΌΠΎΠ±ΠΈΠ»ΡŒΠ½Ρ‹Π΅ Ρ„Π°Ρ€Ρ‹, ΠΎΠ½ΠΈ ΠΌΠ΅Π΄Π»Π΅Π½Π½ΠΎ Ρ‚ΡƒΡΠΊΠ½Π΅ΡŽΡ‚ ΠΏΠΎ ΠΌΠ΅Ρ€Π΅ разрядки аккумулятора.ΠŸΠΎΡ‡Π΅ΠΌΡƒ ΠΎΠ½ΠΈ просто Π½Π΅ ΠΌΠΈΠ³Π°ΡŽΡ‚, ΠΊΠΎΠ³Π΄Π° батарСя разряТСна? Π˜Ρ… постСпСнноС Π·Π°Ρ‚Π΅ΠΌΠ½Π΅Π½ΠΈΠ΅ ΠΎΠ·Π½Π°Ρ‡Π°Π΅Ρ‚, Ρ‡Ρ‚ΠΎ Π²Ρ‹Ρ…ΠΎΠ΄Π½ΠΎΠ΅ напряТСниС Π±Π°Ρ‚Π°Ρ€Π΅ΠΈ ΡƒΠΌΠ΅Π½ΡŒΡˆΠ°Π΅Ρ‚ΡΡ ΠΏΠΎ ΠΌΠ΅Ρ€Π΅ разряда Π±Π°Ρ‚Π°Ρ€Π΅ΠΈ. ΠŸΡ€ΠΈΡ‡ΠΈΠ½Π° сниТСния Π²Ρ‹Ρ…ΠΎΠ΄Π½ΠΎΠ³ΠΎ напряТСния для разряТСнных ΠΈΠ»ΠΈ ΠΏΠ΅Ρ€Π΅Π³Ρ€ΡƒΠΆΠ΅Π½Π½Ρ‹Ρ… Π±Π°Ρ‚Π°Ρ€Π΅ΠΉ Π·Π°ΠΊΠ»ΡŽΡ‡Π°Π΅Ρ‚ΡΡ Π² Ρ‚ΠΎΠΌ, Ρ‡Ρ‚ΠΎ всС источники напряТСния состоят ΠΈΠ· Π΄Π²ΡƒΡ… основных частСй – источника элСктричСской энСргии ΠΈ Π²Π½ΡƒΡ‚Ρ€Π΅Π½Π½Π΅Π³ΠΎ сопротивлСния.

ЭлСктродвиТущая сила

ВсС источники напряТСния ΡΠΎΠ·Π΄Π°ΡŽΡ‚ Ρ€Π°Π·Π½ΠΎΡΡ‚ΡŒ ΠΏΠΎΡ‚Π΅Π½Ρ†ΠΈΠ°Π»ΠΎΠ² ΠΈ ΠΌΠΎΠ³ΡƒΡ‚ ΠΏΠΎΠ΄Π°Π²Π°Ρ‚ΡŒ Ρ‚ΠΎΠΊ, Ссли ΠΏΠΎΠ΄ΠΊΠ»ΡŽΡ‡Π΅Π½Ρ‹ ΠΊ ΡΠΎΠΏΡ€ΠΎΡ‚ΠΈΠ²Π»Π΅Π½ΠΈΡŽ.Π’ нСбольшом ΠΌΠ°ΡΡˆΡ‚Π°Π±Π΅ Ρ€Π°Π·Π½ΠΎΡΡ‚ΡŒ ΠΏΠΎΡ‚Π΅Π½Ρ†ΠΈΠ°Π»ΠΎΠ² создаСт элСктричСскоС ΠΏΠΎΠ»Π΅, ΠΊΠΎΡ‚ΠΎΡ€ΠΎΠ΅ воздСйствуСт Π½Π° заряды, вызывая Ρ‚ΠΎΠΊ. ΠœΡ‹ Π½Π°Π·Ρ‹Π²Π°Π΅ΠΌ эту Ρ€Π°Π·Π½ΠΎΡΡ‚ΡŒ ΠΏΠΎΡ‚Π΅Π½Ρ†ΠΈΠ°Π»ΠΎΠ² элСктродвиТущСй силой (сокращСнно Π­Π”Π‘). Π­Π”Π‘ – это Π²ΠΎΠΎΠ±Ρ‰Π΅ Π½Π΅ сила; это особый Ρ‚ΠΈΠΏ разности ΠΏΠΎΡ‚Π΅Π½Ρ†ΠΈΠ°Π»ΠΎΠ² источника ΠΏΡ€ΠΈ отсутствии Ρ‚ΠΎΠΊΠ°. Π•Π΄ΠΈΠ½ΠΈΡ†Ρ‹ измСрСния Π­Π”Π‘ – Π²ΠΎΠ»ΡŒΡ‚Ρ‹.

ЭлСктродвиТущая сила Π½Π°ΠΏΡ€ΡΠΌΡƒΡŽ связана с источником разности ΠΏΠΎΡ‚Π΅Π½Ρ†ΠΈΠ°Π»ΠΎΠ², Π½Π°ΠΏΡ€ΠΈΠΌΠ΅Ρ€ с ΠΊΠΎΠ½ΠΊΡ€Π΅Ρ‚Π½ΠΎΠΉ ΠΊΠΎΠΌΠ±ΠΈΠ½Π°Ρ†ΠΈΠ΅ΠΉ химичСских вСщСств Π² Π±Π°Ρ‚Π°Ρ€Π΅Π΅. Однако ΠΏΡ€ΠΈ ΠΏΡ€ΠΎΡ‚Π΅ΠΊΠ°Π½ΠΈΠΈ Ρ‚ΠΎΠΊΠ° Π­Π”Π‘ отличаСтся ΠΎΡ‚ Π²Ρ‹Ρ…ΠΎΠ΄Π½ΠΎΠ³ΠΎ напряТСния устройства.НапряТСниС Π½Π° Π²Ρ‹Π²ΠΎΠ΄Π°Ρ… Π±Π°Ρ‚Π°Ρ€Π΅ΠΈ, Π½Π°ΠΏΡ€ΠΈΠΌΠ΅Ρ€, мСньшС, Ρ‡Π΅ΠΌ Π­Π”Π‘, ΠΊΠΎΠ³Π΄Π° батарСя ΠΏΠΎΠ΄Π°Π΅Ρ‚ Ρ‚ΠΎΠΊ, ΠΈ ΠΎΠ½ΠΎ ΠΏΠ°Π΄Π°Π΅Ρ‚ дальшС, ΠΊΠΎΠ³Π΄Π° батарСя разряТаСтся ΠΈΠ»ΠΈ разряТаСтся. Однако, Ссли Π²Ρ‹Ρ…ΠΎΠ΄Π½ΠΎΠ΅ напряТСниС устройства ΠΌΠΎΠΆΠ½ΠΎ ΠΈΠ·ΠΌΠ΅Ρ€ΠΈΡ‚ΡŒ Π±Π΅Π· потрСблСния Ρ‚ΠΎΠΊΠ°, Ρ‚ΠΎ Π²Ρ‹Ρ…ΠΎΠ΄Π½ΠΎΠ΅ напряТСниС Π±ΡƒΠ΄Π΅Ρ‚ Ρ€Π°Π²Π½ΠΎ Π­Π”Π‘ (Π΄Π°ΠΆΠ΅ для сильно разряТСнной Π±Π°Ρ‚Π°Ρ€Π΅ΠΈ).

НапряТСниС Π½Π° ΠΊΠ»Π΅ΠΌΠΌΠ°Ρ…

схСматично ΠΏΠΎΠΊΠ°Π·Ρ‹Π²Π°Π΅Ρ‚ источник напряТСния. Π’Ρ‹Ρ…ΠΎΠ΄Π½ΠΎΠ΅ напряТСниС устройства измСряСтся Π½Π° Π΅Π³ΠΎ Π²Ρ‹Π²ΠΎΠ΄Π°Ρ… ΠΈ называСтся напряТСниСм Π½Π° Π²Ρ‹Π²ΠΎΠ΄Π°Ρ… Π’ .НапряТСниС Π½Π° ΠΊΠ»Π΅ΠΌΠΌΠ°Ρ… опрСдСляСтся ΡƒΡ€Π°Π²Π½Π΅Π½ΠΈΠ΅ΠΌ:

БхСматичСскоС ΠΈΠ·ΠΎΠ±Ρ€Π°ΠΆΠ΅Π½ΠΈΠ΅ источника напряТСния

Π›ΡŽΠ±ΠΎΠΉ источник напряТСния (Π² Π΄Π°Π½Π½ΠΎΠΌ случаС сухой ΡƒΠ³Π»Π΅Ρ€ΠΎΠ΄-Ρ†ΠΈΠ½ΠΊΠΎΠ²Ρ‹ΠΉ элСмСнт) ΠΈΠΌΠ΅Π΅Ρ‚ Π­Π”Π‘, ΡΠ²ΡΠ·Π°Π½Π½ΡƒΡŽ с источником разности ΠΏΠΎΡ‚Π΅Π½Ρ†ΠΈΠ°Π»ΠΎΠ², ΠΈ Π²Π½ΡƒΡ‚Ρ€Π΅Π½Π½Π΅Π΅ сопротивлСниС r, связанноС с Π΅Π³ΠΎ конструкциСй. (ΠžΠ±Ρ€Π°Ρ‚ΠΈΡ‚Π΅ Π²Π½ΠΈΠΌΠ°Π½ΠΈΠ΅, Ρ‡Ρ‚ΠΎ сцСнарий E ΠΎΠ·Π½Π°Ρ‡Π°Π΅Ρ‚ Π­Π”Π‘.) Π’Π°ΠΊΠΆΠ΅ ΠΏΠΎΠΊΠ°Π·Π°Π½Ρ‹ Π²Ρ‹Ρ…ΠΎΠ΄Π½Ρ‹Π΅ ΠΊΠ»Π΅ΠΌΠΌΡ‹, Π½Π° ΠΊΠΎΡ‚ΠΎΡ€Ρ‹Ρ… измСряСтся напряТСниС Π½Π° ΠΊΠ»Π΅ΠΌΠΌΠ°Ρ… V. ΠŸΠΎΡΠΊΠΎΠ»ΡŒΠΊΡƒ V = Π­Π”Π‘-Ir, напряТСниС Π½Π° ΠΊΠ»Π΅ΠΌΠΌΠ°Ρ… Ρ€Π°Π²Π½ΠΎ Π­Π”Π‘, Ρ‚ΠΎΠ»ΡŒΠΊΠΎ Ссли Ρ‚ΠΎΠΊ Π½Π΅ Ρ‚Π΅Ρ‡Π΅Ρ‚.

$ V = Π­Π”Π‘ – Ir $,

Π³Π΄Π΅ r – Π²Π½ΡƒΡ‚Ρ€Π΅Π½Π½Π΅Π΅ сопротивлСниС, Π° I – Ρ‚ΠΎΠΊ, ΠΏΡ€ΠΎΡ‚Π΅ΠΊΠ°ΡŽΡ‰ΠΈΠΉ Π²ΠΎ врСмя измСрСния.

ΠžΡΡ‚Π°Π²ΠΈΡ‚ΡŒ ΠΊΠΎΠΌΠΌΠ΅Π½Ρ‚Π°Ρ€ΠΈΠΉ